Vous êtes sur la page 1sur 226

Geometrı́a

Hugo Caerols
Rely Pellicer
Departamento de Ciencias
Universidad Adolfo Ibáñez
Índice General

Introducción 2

1. Razonamiento lógico. 6
1.1. El lenguaje de la Matemática . . . . . . . . . . . . . . . . . . . . . . . . . . . . . . 6
1.2. Conectivos y proposiciones . . . . . . . . . . . . . . . . . . . . . . . . . . . . . . . . 7
1.3. Álgebra proposicional . . . . . . . . . . . . . . . . . . . . . . . . . . . . . . . . . . 11
1.4. Cuantificadores . . . . . . . . . . . . . . . . . . . . . . . . . . . . . . . . . . . . . . 14
1.5. Elementos de un teorema . . . . . . . . . . . . . . . . . . . . . . . . . . . . . . . . 16
1.6. Métodos de demostración . . . . . . . . . . . . . . . . . . . . . . . . . . . . . . . . 18
1.6.1. Demostración directa . . . . . . . . . . . . . . . . . . . . . . . . . . . . . . . 18
1.6.2. Demostración por contrarrecı́proco . . . . . . . . . . . . . . . . . . . . . . . 19
1.6.3. Demostración por reducción al absurdo . . . . . . . . . . . . . . . . . . . . 19
1.7. Ejemplo de construción de una teorı́a . . . . . . . . . . . . . . . . . . . . . . . . . . 20

2. Geometrı́a plana 23
2.1. Elementos Iniciales de la Geometrı́a Plana. . . . . . . . . . . . . . . . . . . . . . . 23
2.2. Congruencia y Semejanza . . . . . . . . . . . . . . . . . . . . . . . . . . . . . . . . 30
2.3. Áreas y el Teorema de Tales. . . . . . . . . . . . . . . . . . . . . . . . . . . . . . . 41
2.4. Construcciones geométricas elementales. . . . . . . . . . . . . . . . . . . . . . . . . 48
2.5. El número π y el área de la circunferencia. . . . . . . . . . . . . . . . . . . . . . . . 53
2.6. División de un segmento. Razón áurea. . . . . . . . . . . . . . . . . . . . . . . . . . 61

3. Geometrı́a del espacio 74


3.1. Algunos postulados, definiciones y conceptos . . . . . . . . . . . . . . . . . . . . . 74
3.2. Problemas iniciales . . . . . . . . . . . . . . . . . . . . . . . . . . . . . . . . . . . . 75
3.3. Cuerpos en el espacio . . . . . . . . . . . . . . . . . . . . . . . . . . . . . . . . . . 79
3.4. Ángulos en los poliedros . . . . . . . . . . . . . . . . . . . . . . . . . . . . . . . . . 80
3.5. Prismas y cilindros . . . . . . . . . . . . . . . . . . . . . . . . . . . . . . . . . . . . 82
3.6. Pirámides y conos . . . . . . . . . . . . . . . . . . . . . . . . . . . . . . . . . . . . 83
3.7. Volumen y área de cuerpos en el espacio . . . . . . . . . . . . . . . . . . . . . . . . 85
3.8. Volumen y área de prismas y cilindros . . . . . . . . . . . . . . . . . . . . . . . . . 86
3.9. Volumen y área de pirámides y conos . . . . . . . . . . . . . . . . . . . . . . . . . . 88
3.10. Volumen y área de esferas y sectores . . . . . . . . . . . . . . . . . . . . . . . . . . 94

1
4. Trigonometrı́a 100
4.1. Medición de ángulos. . . . . . . . . . . . . . . . . . . . . . . . . . . . . . . . . . . . 100
4.2. Razones trigonométricas en el triángulo rectángulo. . . . . . . . . . . . . . . . . . . 105
4.3. Funciones trigonométricas. . . . . . . . . . . . . . . . . . . . . . . . . . . . . . . . . 111
4.4. Propiedades de las funciones trigonométricas. . . . . . . . . . . . . . . . . . . . . . 117
4.5. Construcción de gráficos de funciones trigonométricas. . . . . . . . . . . . . . . . . 121
4.6. Funciones trigonométricas inversas. . . . . . . . . . . . . . . . . . . . . . . . . . . . 123
4.7. Identidades trigonométricas. . . . . . . . . . . . . . . . . . . . . . . . . . . . . . . . 128
4.8. Ecuaciones tigonométricas. . . . . . . . . . . . . . . . . . . . . . . . . . . . . . . . 133
4.9. Ángulos y lados de un triángulo. . . . . . . . . . . . . . . . . . . . . . . . . . . . . 137

5. Vectores y geometrı́a analı́tica 141


5.1. Segmentos dirigidos y vectores . . . . . . . . . . . . . . . . . . . . . . . . . . . . . 141
5.2. Operaciones con vectores . . . . . . . . . . . . . . . . . . . . . . . . . . . . . . . . 143
5.3. Sistemas de coordenadas . . . . . . . . . . . . . . . . . . . . . . . . . . . . . . . . . 145
5.4. Propiedades fundamentales . . . . . . . . . . . . . . . . . . . . . . . . . . . . . . . 147
5.5. Ecuación de la recta . . . . . . . . . . . . . . . . . . . . . . . . . . . . . . . . . . . 152
5.6. Distancia entre puntos y norma de un vector . . . . . . . . . . . . . . . . . . . . . 157
5.7. Ángulo entre vectores y producto punto . . . . . . . . . . . . . . . . . . . . . . . . 158
5.8. Proyecciones . . . . . . . . . . . . . . . . . . . . . . . . . . . . . . . . . . . . . . . 161
5.9. Producto cruz y sus propiedades . . . . . . . . . . . . . . . . . . . . . . . . . . . . 163
5.10. El plano en R3 . . . . . . . . . . . . . . . . . . . . . . . . . . . . . . . . . . . . . . . 166
5.11. Distancias entre punto, recta y plano . . . . . . . . . . . . . . . . . . . . . . . . . . 168
5.12. Propiedades geométricas y vectores . . . . . . . . . . . . . . . . . . . . . . . . . . . 171

6. Secciones cónicas 176


6.1. La circunferencia . . . . . . . . . . . . . . . . . . . . . . . . . . . . . . . . . . . . . 176
6.2. La Elipse . . . . . . . . . . . . . . . . . . . . . . . . . . . . . . . . . . . . . . . . . 181
6.3. La parábola . . . . . . . . . . . . . . . . . . . . . . . . . . . . . . . . . . . . . . . . 186
6.4. La hipérbola . . . . . . . . . . . . . . . . . . . . . . . . . . . . . . . . . . . . . . . 192

7. Transformaciones de coordenadas 198


7.1. Cambio de sistema de coordenadas . . . . . . . . . . . . . . . . . . . . . . . . . . . 198
7.1.1. Traslación de ejes coordenados . . . . . . . . . . . . . . . . . . . . . . . . . 198
7.1.2. Rotación de ejes coordenados . . . . . . . . . . . . . . . . . . . . . . . . . . 201
7.2. Ecuación General de Segundo Grado. . . . . . . . . . . . . . . . . . . . . . . . . . . 204
7.3. Transformaciones en un mismo sistema coordenado . . . . . . . . . . . . . . . . . . 214
7.3.1. Traslación en el plano . . . . . . . . . . . . . . . . . . . . . . . . . . . . . . 214
7.3.2. Rotación en el plano . . . . . . . . . . . . . . . . . . . . . . . . . . . . . . . 214
7.3.3. Reflexión en el plano . . . . . . . . . . . . . . . . . . . . . . . . . . . . . . . 215
7.3.4. Homotecia en el plano . . . . . . . . . . . . . . . . . . . . . . . . . . . . . . 215
7.4. Matrices. . . . . . . . . . . . . . . . . . . . . . . . . . . . . . . . . . . . . . . . . . 217
7.5. Transformaciones del Plano. . . . . . . . . . . . . . . . . . . . . . . . . . . . . . . . 220

Bibliografı́a 225

2
Introducción.

La Geometrı́a constituye una disciplina esencial del saber humano por su utilidad para describir
el espacio en el que vivimos. Es quizás la parte de las matemáticas más conectada con la realidad
fı́sica. Múltiples actividades cotidianas la emplean de una manera u otra: desde el deporte y la
ingenierı́a hasta el arte y la industria.

La intuición geométrica es indispensable para orientarse reflexivamente en el espacio, para


estimar formas y distancias, para hacer cálculos relativos a la distribución de los objetos en el
espacio y representa un aspecto importante en el estudio de los elementos de la naturaleza. La
geometrı́a ofrece medios para describir, analizar y comprender el mundo y ver la belleza en sus
estructuras.

A través de la historia la geometrı́a ha sido esencial en la formación académica por su contribu-


ción al desarrollo del pensamiento lógico independiente a través de la resolución de problemas;
porque desarrolla la capacidad para pensar creativamente, formular, comprobar, generalizar y dis-
cutir argumentos de modo que favorece el desarrollo del lenguaje matemático y la visión cientı́fica
de la realidad.

Ya Galileo expresaba:

“El Universo está escrito en el lenguaje de las matemáticas y sus caracteres son triángulos,
cı́rculos y otras figuras geométricas, sin las cuales es humanamente imposible entender una sola de
sus palabras. Sin ese lenguaje, navegamos en un oscuro laberinto”.

La palabra geometrı́a se compone de los términos griegos: tierra (gea) y medida (metron). Lo
que hoy conocemos como una de las ramas de la matemática se remonta a la cultura babilónica,
pasando por el repertorio de técnicas con que los egipcios medı́an las crecidas del Nilo. Sin embargo
fue en la Antigua Grecia donde la geometrı́a se consolida. El aporte de los griegos consistió en
que no sólo utilizaron las técnicas geométricas para resolver asuntos prácticos, sino que también
reflexionaron sobre ellas. A partir de estas reflexiones aparece en el pensamiento griego el concepto
de demostración y la geometrı́a se convierte en una especie de modelo para el resto de los saberes,
precisamente por el modo en que sus enunciados pueden probarse.

En la historia posterior la investigación en geometrı́a ha sido estimulada gratamente por nuevas


ideas tanto desde el interior de las matemáticas como desde otras disciplinas. Particularmente
interesante es la estrecha relación de la geometrı́a con el arte del Renacimiento. En el presente, las

3
enormes posibilidades de las gráficas por computadoras y la relevancia del diseño por su potencia-
lidad de agregar valor a las creaciones humanas han aumentado la importancia de una adecuada
educación visual.

La formalización y diversificación de los enfoques de esta disciplina se pueden apreciar actual-


mente en diferentes tipos de geometrı́a. Entre las geometrı́as euclideanas se pueden mencionar la
clásica proveniente de los griegos, la trigonometrı́a, las geometrı́as analı́tica y diferencial, la ge-
ometrı́a proyectiva, ası́ como la geometrı́a vectorial; todas con enfoques, objetos y tratamientos
diferenciados. Aún más, existen geometrı́as no euclideanas que surgen a partir del siglo XIX como
las geometrı́as hiperbólica y elı́ptica, la topologı́a y, más recientemente, la geometrı́a fractal.

Este texto no está diseñado como un compendio enciclopédico de resultados geométricos con
sus demostraciones, sino como una visita a temas que a los autores nos parecen interesantes y
que creemos que serán útiles en la formación de una visión geométrica de la realidad. El lector
debe tomarlo como base de un estudio que requerirá una alta dosis de trabajo individual y de
creatividad, pues la manera en que están presentados los contenidos no agota las preguntas que
podrı́a hacerse una mente curiosa.

Se ha pretendido que el tratamiento de ciertos contenidos y las demostraciones de algunos resul-


tados incluyan ideas modernas y relacionadas con otras disciplinas matemáticas como el Cálculo
Diferencial y el Álgebra Lineal, vinculación que podrá ser más evidente al avanzar en estudios
posteriores. Las demostraciones deberán ser herramientas para el entendimiento personal, o para
explicar, clarificar y verificar.

Sin renunciar al fuerte carácter deductivo que tradicionalmente se le ha dado a la enseñanza de


la geometrı́a se presentan los temas recurriendo en gran medida a la intuición como instrumento
de acceso al conocimiento geométrico, tratando de lograr una interrelación dialéctica entre ambas
aproximaciones.

Los problemas que aparecen en cada sección de este libro deben ser abordados con la mayor
dedicación. Consideramos que este es un aspecto esencial del texto que se presenta, pues es en
esta instancia donde se verifica en parte la adquisición del conocimiento y su realización a través
de la resolución de problemas. Se pretende exponer a los lectores a diferentes niveles de rigor y
complejidad en los problemas, por lo que encontrará algunos cuya solución es más directa y otros
que requieren de mayor consolidación del conocimiento adquirido, ası́ como de la capacidad de
relacionar diferentes temas y de generar ideas creativas al buscar una solución.

En este libro se introducen enfoques cualitativamente diferentes en relación a las herramientas


matemáticas que emplean. Algunos temas requieren un uso más intensivo del cálculo numérico,
otros emplean las técnicas y procedimientos del álgebra, mientras que en ocasiones, la concep-
tualización de nuevos objetos geométricos se vuelve esencial al momento de enfrentar situaciones
problemáticas.

Queremos potenciar la habilidad de los lectores para elegir las herramientas adecuadas (concep-
tuales, operativas, tecnológicas) para resolver problemas geométricos especı́ficos y de ser capaces de
transitar de una representación del problema a otra que utilice diferentes tratamientos. Las repre-
sentaciones gráficas, algebraicas, vectoriales u otras, expresan de manera diferente las propiedades
y comportamiento de una situación geométrica y nos interesa que el lector sea capaz de ver en cada

4
una las caracterı́sticas que le resultan relevantes para su problema, seleccionando adecuadamente
las herramientas de tratamiento correspondientes a cada representación.

Este texto ha sido diseñado para un curso semestral de geometrı́a y cubre los enfoques y temas
tradicionalmente tratados en la enseñanza inicial de la geometrı́a. Sin embargo el texto puede
ser abordado parcialmente o complementado con textos más especı́ficos, pues sus capı́tulos son
más o menos independientes. El texto comienza con una introducción a la lógica matemática y
una discusión de las técnicas de demostración más comunes en geometrı́a, como base para la
construcción de cualquier teorı́a matemática. Posteriormente se trata la geometrı́a euclideana,
tanto plana como espacial recorriendo resultados clásicos y nociones esenciales en la formación de
una visión geométrica de la realidad (longitud, área, volumen,...). Se aborda la trigonometrı́a y
sus técnicas de tratamiento, haciendo la conexión con el cálculo y la teorı́a de funciones. Luego
se introduce el análisis vectorial y se muestra su relación con el álgebra lineal y la fı́sica. La
geometrı́a analı́tica ocupa el resto del texto, poniendo de manifiesto cómo el álgebra y la teorı́a de
ecuaciones enriquecen el análisis de situaciones geométricas; desde el estudio de lugares geométricos
importantes hasta el análisis de las transformaciones del plano y del espacio y su importancia en
la modelación de la realidad.

Queremos finalmente agradecer a todos los colegas que han estimulado la confección de este
texto y han participado de interesantes reflexiones acerca de la mejor forma de presentar ciertos
temas. Al lector, lo desafiamos a ingresar a este mundo de las formas con la mayor disposición a
enfrentar los retos que se presenten y a insistir en redescubrir y conquistar el conocimiento que
por miles de años ha quitado el sueño a tantos hombres ilustres.

Santiago de Chile, Marzo de 2007.

A Doris, Sofı́a, Diego y Bebé...


Para Adela...

5
Capı́tulo 1

Razonamiento lógico.

En este capı́tulo estudiamos en forma concreta el lenguaje de la matemática. Se presentan los


elementos básicos de la lógica binaria: proposiciones, conectivos y cuantificadores. Analizamos los
elementos de un teorema y los métodos usuales de demostración. Mostramos un ejemplo sencillo
de cómo se genera una teorı́a a partir de los elementos básicos.

1.1. El lenguaje de la Matemática


El lenguaje matemático, a diferencia del lenguaje con el que nos expresamos diariamente, es
extremadamente preciso. Entender cómo funciona es una gran ayuda para comprender el lenguaje
cientı́fico en general y es por eso, que en este primer capı́tulo hemos decidido aclarar al lector este
punto de vital importancia para que entienda cómo leer el resto del libro.

Las teorı́as matemáticas se construyen sobre elementos básicos que no se definen y que se
aceptan como fundamento de todo. Por ejemplo aceptamos los puntos, las rectas y los planos como
elementos iniciales de la geometrı́a sin cuestionarnos demasiado, sólo teniendo una idea de estos.
Lo mismo ocurre con los números naturales que expresan el concepto profundo de cantidad.

Dan cuerpo a la teorı́a un conjunto de definiciones y teoremas. La geometrı́a se puede deducir


a partir de algunos principios básicos llamados postulados que se asumen como verdaderos.

Un ejemplo de postulado: Dos puntos del plano determinan una única recta.

La utilización de esta estructura de razonamiento: deducir lógicamente a partir de un grupo de


postulados todas las propiedades siguientes, fue el aporte esencial de los griegos. Por el contrario,
la forma de razonar de las geometrı́as primitivas, que consideraban como prueba de una propiedad
la simple experimentación, los condujo a varios errores ahora conocidos.

La antigua fórmula Babilónica


(a + c)(b + d)
K= ,
4
para el área de un cuadrilátero cuyos lados consecutivos miden a, b, c, d es incorrecta y da un
resultado demasiado grande para todos los cuadriláteros no rectangulares.

6
El esquema de razonamiento de los griegos, plasmado en los Elementos de Euclides, que es
utilizado también en otras áreas se puede resumir como sigue:

a) Se dan explicaciones iniciales de ciertos términos técnicos básicos del discurso, siendo la
intención sugerir al lector lo que quieren decir estos términos básicos.
b) Algunos principios primarios relacionados con los términos básicos y que se suponen acep-
tables como verdades en base a las propiedades sugeridas por las explicaciones iniciales,
se enumeran. Estos principios primarios se llaman postulados o axiomas del discurso. Los
términos básicos iniciales y los postulados del discurso se consideran colectivamente como la
base del discurso.
c) Todos los términos técnicos del discurso se definen por medio de los básicos.
d) Todos los otros principios del discurso se deducen lógicamente de los axiomas o postulados.
Estos principios deducidos se llaman teoremas del discurso.

Dado que los teoremas conectan los elementos básicos, las definiciones y los postulados de una
teorı́a usando la lógica, pasamos ahora a explicar detalladamente los elementos que conforman la
lógica binaria.

1.2. Conectivos y proposiciones


Definición 1.2.1. Proposiciones. Son enunciados del lenguaje natural sobre los cuales es posible
afirmar que son verdaderos o falsos.

Por ejemplo la proposición p : dos es mayor que cinco. No es una proposición q : Hola ¿cómo
estás?

Definición 1.2.2. La veracidad o falsedad de una proposición se llama su valor de verdad.

Definición 1.2.3. Se llama función proposicional a una proposición cuyo valor de verdad depende
de una o más variables.

Por ejemplo, p(n) : “n es un número par”. El valor de verdad de p depende del valor asumido
por n. p(1) es falsa porque uno no es un número par, p(6) es verdadera por que seis es un número
par.

A partir de proposiciones simples, podemos construir otras proposiciones utilizando ciertos


sı́mbolos denominados conectivos.

Los conectivos más usados son:

Condicional (⇒), Conjunción (∧), Disyunción (∨), Bicondicional (⇔), textitNegación (¬).

7
Al conectar dos proposiciones dadas por algunos de estos conectivos producimos una nueva
proposición cuyo valor de verdad depende de los valores de las proposiciones originales.

Sean p y q dos proposiciones, como mencionábamos el valor de verdad de la proposición com-


puesta p2q dependerá de los valores de verdad tanto de p como de q. Las siguientes tablas resumen
todas las posibilidades con los conectivos antes mencionados, reemplazando 2 en p2q por el conec-
tivo respectivo.

p q p⇒q p q p∧q p q p∨q p q p⇔q


V V V V V V V V V V V V p ¬p
V F F V F F V F V V F F V F
F V V F V F F V V F V F F V
F F V F F F F F F F F V

Pasamos ahora a describir los diferentes conectivos.

La implicación p ⇒ q se lee p implica q y en general se traduce al lenguaje natural como si p


entonces q. El valor de verdad de p ⇒ q es falso sólo si p es verdadera y q es falsa. En los demás
casos p ⇒ q es verdadera. p se dice también antecedente y q consecuente.

Pensemos por ejemplo en las funciones proposicionales q(n) : “n es un número par”, p(n) : “n
es divisible por cuatro”. La proposición compuesta p(n) ⇒ q(n), se traduce al lenguaje natural
como:“ Si n es divisible por cuatro entonces n es un número par.”

Si la implicación p ⇒ q es verdadera p es una condición suficiente para q y q es una condición


necesaria para p. Esta terminologı́a es utilizada en diferentes materias y contextos, pero siempre
con el mismo significado. Aclararemos lo anterior con un ejemplo.

En el caso que analizábamos, “si n es divisible por cuatro entonces n es un número par”, que
es claramente verdadera para cualquier valor asignado a la variable n. Que n sea divisible por
cuatro es una condición suficiente para que n sea un número par. Que n sea un número par es una
condición necesaria para que n sea divisible por cuatro.

Ejemplo: Considere las funciones proposicionales: p(n) : n es divisible por tres, q(n) : n es
divisible por nueve. En el contexto de los números naturales. Responda lo siguiente:

1. ¿Es p(n) condición necesaria para q(n) ?

2. ¿Es p(n) condición suficiente para q(n) ?

Para responder la pregunta (1) nos preguntamos si la implicación q(n) ⇒ p(n) es verdadera para
todo natural n.

Supongamos que q(n) es verdadera, es decir, sea n ∈ N tal que n es divisible por 9.

8
Entonces n = 9a, para algún a ∈ N, luego lo que equivale a n = 3(3a), para algún a ∈ N. Por lo
anterior n es divisible por 3, la proposición q(n) ⇒ p(n) es verdadera y p(n) es condición necesaria
para q(n).

Pasamos ahora a responder la parte (b). Esta equivale a decidir si la implicación p(n) ⇒ q(n)
es verdadera para todo natural n.

La intuición nos dice que esta propiedad debe ser falsa para algunos naturales. Para confirmar lo
anterior debemos encontrar un n ∈ N tal que p(n) sea verdadera y q(n) sea falsa. Basta tomar por
ejemplo n = 3. p(3) es verdadera pues 3 es divisible por 3, sin embargo q(3) es falsa pues 3 no es
divisible por 9.

Luego el hecho que un número sea divisible por tres no es condición suficiente para que sea
divisible por nueve.

La conjunción p ∧ q de las proposiciones p y q se lee p y q. La conjunción es verdadera si y sólo


si, ambas proposiciones p y q son verdaderas y falsa en los demás casos.

Por ejemplo, sean p y q las proposiciones p :“ dos es par”, q :“ tres es múltiplo de cuatro.”p es
verdadera y q es falsa, luego p ∧ q es falsa.

La disyunción p ∨ q de las proposiciones p y q se lee p o q, esta es falsa sólo en el caso que ambas
proposiciones sean falsas y verdadera en los otros casos.

Por ejemplo, sean como antes p y q las proposiciones p :“ dos es par”, q :“ tres es múltiplo de
cuatro.”p es verdadera y q es falsa, luego p ∨ q es verdadera.

Analice desde el punto de vista de la lógica binaria la siguiente frase:


“La restricción vehicular afecta este viernes a los vehı́culos sin convertidor catalı́tico cuyas
patentes terminen en tres y cuatro.)) Cómo corregirı́a la proposición anterior ?

El bicondicional de las proposiciones p y q se denota p ⇔ q y se lee p si y sólo si q. Este es


verdadero sólo en el caso que ambas proposiciones tengan el mismo valor de verdad y falso en los
demás casos.

Por ejemplo, considere como antes las siguientes funciones proposicionales p(n): “n es divisible
por cuatro”, q(n): “n es un número par.”p(n) ⇔ q(n), en lenguaje natural dice “n es divisible por
cuatro si y sólo si “n es un número par. es claro que para n = 2 esta función es falsa porque p(2)
2

es falsa y q(2) verdadera.

Ejemplo: Escribamos en simbologı́a lógica la siguiente expresión:

“Si el sol no ilumina las plantas y no se riegan entonces los frutos no crecen”.

Sean p : “El sol ilumina las plantas”.

9
q : “Las plantas se riegan”.

s : “Los frutos de las plantas crecen”.

En forma simbólica nos queda


(¬p ∧ ¬q) ⇒ ¬s
Ejercicios

1. Exprese en lenguaje simbólico las siguientes frases:

a) Hoy es Lunes y Marı́a tiene frı́o.


b) Mañana voy a ir al cine o al teatro.
c) Si mañana no voy al cine entonces comeré papas fritas.

2. Determinar el valor de verdad de cada una de las siguientes proposiciones:

a) Si 3 < 5 entonces −3 < −5.


b) Si 2 + 2 = 4 , entonces 3 + 3 = 7 si y sólo si 1 + 1 = 1.
√ √
c) 16 = 4 ó 16 = −4.
√ √
d ) 6 + 4 = 10 y 2 · 2 = 2
e) 52 = 25 ó 2 + 2 = 5
f ) Si 2 + 2 = 4, entonces no es cierto que 2 + 1 = 3 y 5 + 5 = 10.
g) Si 3 − 1 = 1 entonces 2 + 2 = 5.

3. Si sabemos que “El dı́a está lluvioso” es una aseveración verdadera, pero que la aseveración
“El auto es nuevo” es falsa, ¿cuál es el valor de verdad de la aseveración “El dı́a está lluvioso
y el auto es nuevo”?

4. Considere las funciones proposicionales definidas en los enteros


t(x) : x(x + 1) es un número par,
f (x) : (x2 − x + 41) es un número primo.
Determine el valor de verdad de la proposición t(3) ⇒ f (2)

10
1.3. Álgebra proposicional
Definición 1.3.1. Proposiciones lógicamente equivalentes. Decimos que dos proposiciones com-
puestas m y n son equivalentes si entregan el mismo valor de verdad para cualquier valor de verdad
de las variables involucradas. Escribimos m ≡ n para indicar este hecho.

Por ejemplo las proposiciones m : p ⇔ q y n : (p ⇒ q) ∧ (q ⇒ p) son equivalentes. Si


consideramos una evaluación cualquiera, por ejemplo p = V, q = F, el valor de verdad de m es
falso y el de n también es falso. Esto no es casualidad, si analizamos todas las posibilidades de
evaluación que resumimos en la siguiente tabla:

p q p⇔q p⇒q q⇒p (p ⇒ q) ∧ (q ⇒ p)


V V V V V V
V F F F V F
F V F V F F
F F V V V V

vemos las columas que están bajo las proposiciones m y n, nos damos cuenta que coinciden, lo que
significa que las proposiciones asumen el mismo valor de verdad cualquiera sea el valor de verdad
de las variables involucradas, por lo que son equivalentes.

Anotamos
p ⇔ q ≡ (p ⇒ q) ∧ (q ⇒ p).
Note que de lo anterior se deduce que si p ⇔ q es verdadera p es una condición necesaria y
suficiente para q, q también es una condición necesaria y suficiente para p.

Definición 1.3.2. Tautologı́as. Una proposición cuyo valor de verdad es verdadero, cualquiera sea
el valor de verdad de las variables involucradas se llama una tautologı́a.

Una tautologı́a se denota con la letra V . Por ejemplo la proposición (p ∧ q) ⇒ p es verdadera


independientemente del valor de verdad de p o de q, como puede verificarse construyendo la tabla
de asignaciones.

Definición 1.3.3. Contradicciones. Una proposición cuyo valor de verdad es falso, cualquiera sea
el valor de verdad de las variables involucradas se llama una contradicción.

Una contradicción se denota por la letra F . Por ejemplo: p ∧ ¬p es una contradicción.

Colocamos a continuación un listado de proposiciones lógicamente equivalentes, que es de mucha


utilidad para simplificar expresiones en el álgebra lógica.

11
(1) ¬(¬p) ≡ p. (11)p ∧ (q ∧ r) ≡ (p ∧ q) ∧ r, asociatividad.
(2) p ∧ F ≡ F. (12) p ∧ (q ∨ r) ≡ (p ∧ q) ∨ (p ∧ r), distributividad.
(3) p ∨ F ≡ p. (13) p ∨ (q ∧ r) ≡ (p ∨ q) ∧ (p ∨ r), distributividad.
(4) p ∧ V ≡ p. (14) p ⇒ q ≡ ¬p ∨ q.
(5) p ∨ V ≡ V. (15) p ⇒ q ≡ ¬q ⇒ ¬p, contrapositivo.
(6) p ∧ p ≡ p. (16) ¬(p ∧ q) ≡ ¬p ∨ ¬q, ley de Morgan.
(7) p ∨ p ≡ p. (17)¬(p ∨ q) ≡ ¬p ∧ ¬q, ley de Morgan.
(8) p ∨ q ≡ q ∨ p, conmutatividad. (18) p ∨ (p ∧ q) ≡ p, absorción.
(9) p ∧ q ≡ q ∧ p, conmutatividad. (19)p ∧ (p ∨ q) ≡ p, absorción.
(10) p ∨ (q ∨ r) ≡ (p ∨ q) ∨ r, asociatividad. (20) p ∧ ¬p ≡ F
(21) ¬p ∨ p ≡ V

Las equivalencias anteriores nos permiten realizar álgebra en el conjunto de las proposiciones
y las operaciones que se realizan con los conectivos. Podemos simplificarlas, expandirlas y traba-
jar de forma similar a como lo hacemos con expresiones algebraicas reemplazando proposiciones
equivalentes.

Ejemplo: Simplifique al máximo la expresión [(p ∧ (p ⇒ q)] ⇒ q

(A la derecha de cada lı́nea se indica la propiedad utilizada.)

[ p ∧ (p ⇒ q) ] ⇒ q ≡ [ p ∧ (¬p ∨ q) ] ⇒ q (14)

≡ [ (p ∧ ¬p) ∨ (p ∧ q) ] ⇒ q (12)

≡ [ F ∨ (p ∧ q) ] ⇒ q (20)

≡ (p ∧ q) ⇒ q (3)

≡ ¬(p ∧ q) ∨ q (14)

≡ (¬p ∨ ¬q) ∨ q (16)

≡ ¬p ∨ (¬q ∨ q) (10)

≡ ¬p ∨ V (21)

≡ V. (5)

luego, la proposición es una tautologı́a.

Ejemplo: Se sabe que la proposición (p ∧ q) ⇒ ¬r es falsa. Determine el valor de verdad de

(¬[p ∧ q] ∨ ¬r) ⇒ ([[¬p ∨ ¬q] ∧ ¬r] ⇒ p)

12
Como la proposición (p ∧ q) ⇒ ¬r es falsa tenemos que (p ∧ q) debe ser verdadera y ¬r falsa,
lo que nos indica que r debe ser verdadera y tanto p como q deben ser verdaderas. Reemplazando
los valores de verdad en
(¬[p ∧ q] ∨ ¬r) ⇒ ([[¬p ∨ ¬q] ∧ ¬r] ⇒ p)
obtenemos que la proposición es verdadera.

Ejercicios

1. Determine si las siguientes proposiciones son tautologı́as, contradicciones o contingencias


(esto es, no tienen un valor de verdad fijo)

a) (p ∧ q) ∧ ¬(p ∧ q).
b) [(p ⇒ q) ∧ (p ∧ ¬q ∧ r)] ⇒ (¬p ∨ q).

2. Vı́ctor, que siempre dice la verdad, le ha contado a su hermana lo siguiente:


Me gusta Marı́a Elena o Liliana, pero no las dos. Además, si me gustara Liliana, también me
gustarı́a Marı́a Elena. ¿Quién le gusta a Vı́ctor realmente ?
3. Simplifique al máximo las siguientes expresiones:

a) [(p ∧ (p ⇒ q)] ⇒ p.
b) [(p ∨ q) ∧ ¬p] ⇒ q.

4. ¿Bajo qué condiciones de p y q la proposición [(p ⇔ q) ∧ ¬q] ⇒ (p ∧ ¬q), es falsa ?


5. Demostrar que las siguientes proposiciones son tautologı́as:

a) (p ∧ q) ⇒ ¬(¬p ∧ q).
b) p ∨ (p ∧ q) ⇔ p.

6. Demostrar las siguientes equivalencias:

a) p ⇒ (q ⇒ r) ≡ (p ∧ q) ⇒ r
b) p ∨ (p ∧ q) ≡ p, (propiedad de absorción)

7. Sean p, q, r proposiciones. Determine el valor de verdad de cada una de ellas sabiendo que
(p ∧ ¬q) ⇒ [r ∨ (p ⇔ q)] es falsa.
8. Usando las propiedades del álgebra de conectivos lógicos simplifique la proposición compuesta
(p ⇒ q) ⇒ [p ⇒ (¬q ∧ p)].
9. Demuestre sin usar tablas que las proposiciones ¬(p ∧ ¬q) ∨ (¬p ∧ q) y p ⇔ q son equiva-
lentes.

13
1.4. Cuantificadores
A partir de una función proposicional, se puede obtener una proposición anteponiendo un
cuantificador. Los cuantificadores más usados son el cuantificador universal y el existencial.

Definición 1.4.1. Cuantificador Universal. Se denota por el sı́mbolo ∀.


Sea A un conjunto y p(x) una función proposicional en el conjunto A. La proposición ∀x ∈ A :
p(x), se traduce al lenguaje natural como: “ Para todo x en el conjunto A se verifica p(x). Esta
proposición es verdadera si todos los elementos del conjunto A hacen que p(x) sea verdadera, o
dicho de otra forma, verifican la proposición p.

Por ejemplo, p(x) : “ x es positivo ”, la proposición ∀x ∈ R : p(x) se traduce al lenguaje natural


como “ para todo número real x se tiene que x es positivo”. La que es claramente falsa porque −1
es un número real que no es positivo.

Definición 1.4.2. Cuantificador Existencial. Se denota por el sı́mbolo ∃.


Sea A un conjunto y p(x) una función proposicional en el conjunto A. La proposición ∃x ∈ A :
p(x), se traduce al lenguaje natural como: “ Existe un elemento x en el conjunto A que verifica
p(x)”. Esta proposición es verdadera si al menos un elemento del conjunto A verifica la proposición
p.

Por ejemplo, p(x) : “ x es positivo ”, la proposición ∃x ∈ R : p(x) se traduce al lenguaje natural


como “ existe un número real x que es positivo”, la que es claramente verdadera porque 4 es un
número real positivo.

La negación de la proposición ∀x ∈ A : p(x), se denota ¬(∀x ∈ A : p(x)) y es equivalente a


∃x ∈ A : ¬p(x), esto es, existe un elemento del conjunto A que no verifica la proposición p(x).

La negación de la proposición ∃x ∈ A : p(x), se denota ¬(∃x ∈ A : p(x)) y es equivalente a


∀x ∈ A : ¬p(x), esto es, para todo elemento del conjunto A no se verifica la proposición p(x).

Ejemplo: Sean A = {1, 2, 3, 4, 5, 6, 7} y B = {−1, −2, 4, 5, 6, 9, 10}.

Determinemos el valor de verdad de la proposición

∀x ∈ (B − A), ∃y ∈ (A − B) tal que 2x + y ≤ 7

B − A = { −1, 9, 10 } A − B = {1, 3, 7}

∀x ∈ {−1, 9, 10}, ∃y ∈ {1, 3, 7} tal que 2x + y ≤ 7.

Para x = −1 basta tomar y = 1 y se cumple que 2x + y = −1 que es menor o igual que 7.

14
Si consideramos x = 9 debemos averiguar si existe un elemento y en el conjunto {1, 3, 7} que
verifique 18 + y ≤ 7, que equivale a y ≤ −11. Como ningún elemento del conjunto anterior satisface
esta condición, la proposición es falsa.

Ejercicios.

1. Exprese las siguientes proposiciones utilizando los sı́mbolos matemáticos y lógicos usuales:

a) No es cierto que, si el doble de cuatro es dieciséis entonces el cuadrado de cuatro es


treinta y dos.
b) El cuadrado de menos tres es nueve y es mayor que siete.
c) Existe un número entero mayor que dos.
d ) Dos es menor que tres y un real x es mayor o igual que 7.

2. Escriba la negación de cada una de las afirmaciones anteriores en lenguaje natural.

3. Dada la función proposicional p(x) : “x es un número mayor o igual que −2 y menor que 3”,
determine el valor de verdad de las proposiciones siguientes:

a) ∀ x ∈ E : p(x), donde E = {−2, −1, 0}.


b) ∃ x ∈ W : p(x), donde W = {3, 4, 5}.

4. Considerando N como universo escriba en sı́mbolos cada una de las siguientes expresiones:

a) Todo número multiplicado por cero es cero.


b) Todo número es menor o igual que sı́ mismo.
c) Hay un número que es mayor o igual que los demás.
d ) Si x es menor que 2 entonces es menor que 5.
e) Hay un número tal que su cuadrado es igual al número.
f ) No hay ningún número entero positivo y negativo a la vez.

5. Considere las proposiciones

p : ∀ x ∈ {1, 3, 5}, ∃ y ∈ {2, 4} : x > y.

q : ∃ y ∈ {2, 4}, ∀ x ∈ {1, 3, 5} : x > y.

a) Traduzca p y q al lenguaje natural.


b) Determine los valores de verdad de p y q.
c) Exprese simbólicamente las negaciones de p y q.

6. Escribir las negaciones de las siguientes proposiciones:

15
a) ∀x ∈ R, ∃y ∈ R : (x + y = 1 ⇒ x = −y).
b) ∀x ∈ R, ∀y ∈ R : (x + y es par ⇒ x es par ∧ y es par ).
c) ∃x ∈ R, ∀y ∈ R : (x < y ∧ x2 ≥ y).
d ) ∀x ∈ R, ∀y ∈ R, ∃ z ∈ R : (x < y ⇒ x + z = y).

7. Sabiendo que la proposición ∀x ∈ U : ¬p(x) es verdadera, determine el valor de verdad de la


proposición ∃x ∈ U : P p(x) ⇒ ∃x ∈ U : ¬p(x)

8. Sean A = {−1, 2, 3, 4, 1} y B = {1, π}. Determine el valor de verdad de la afirmación:


∃x ∈ A, ∀y ∈ B : x + y < y, justifique.

9. Sean U = {−2, − 12 , 0, 1}, V = {−2, 1, 2}. Determine el valor de verdad de las siguientes
proposiciones:

a) ∀u ∈ U, ∃v ∈ V : (uv + 1 < 0) ∨ (u2 − v 2 = 0).


b) ∀u ∈ U, ∀v ∈ V : (uv + 1 < 0) ∨ (u2 − v 2 = 0).

1.5. Elementos de un teorema


Definición 1.5.1. Teorema. Un teorema es un enunciado que relaciona elementos de una teorı́a
y que se desprende de postulados o de otros teoremas.

Los teoremas en matemática, a diferencia de los postulados y las definiciones, tienen que ser
establecidos a través de una prueba rigurosa, que está basada en razonamientos lógicos.

Conseguir una demostración adecuada de un teorema siempre es un acto creativo, las ideas que
conducen a la prueba no son siempre naturales. Bajo un enunciado muy simple puede encontrarse
una propiedad muy difı́cil de probar.

El lector debe recordar posiblemente el interés público causado por la demostración del último
teorema de Fermat, enunciado que fue atacado por los matemáticos más connotados sin llegar a
una prueba favorable por más de tres siglos, hasta que el matemático británico Andrew Wiles
dió una prueba completa, en un artı́culo de 98 páginas publicado recién en el año 1995.

Muchos de los teoremas están redactados de manera bastante general en la forma p ⇒ q.


Pensemos por ejemplo en el que pareciera ser uno de los más conocidos teoremas: el teorema de
Pitágoras. Un enunciado de éste establece lo siguiente:

“En un triángulo rectángulo cuyos catetos miden a y b e hipotenusa c, se verifica la relación


c2 = a2 + b2 .”

16
Es claro que hay elementos que es necesario conocer para entender el teorema, como qué es un
triángulo rectángulo, cuáles son sus catetos e hipotenusa. Estas son definiciones relacionadas con
elementos que aparecen en el teorema.

Notemos que el enunciado anterior puede ponerse en la forma

“Si un triángulo es rectángulo con catetos que miden a y b e hipotenusa c, entonces se verifica
la relación c2 = a2 + b2 .”

En todo teorema de la forma p ⇒ q, p se dice la hipótesis y q la tesis, en este caso la hipótesis


es que el triángulo es rectángulo, también que la hipotenusa mide c, y los catetos a y b. La tesis es
que c2 = a2 + b2 .

Muchas veces se aprende mucho de los teoremas preguntándose si es cierta la proposición


recı́proca. La recı́proca de p ⇒ q, es q ⇒ p.
En nuestro ejemplo la proposición recı́proca dice:
“Si en un triángulo con lados que miden a, b y c, se verifica la relación c2 = a2 + b2 , entonces
el triángulo es rectángulo.”

La proposición recı́proca en este caso es cierta pero, para otros teoremas, no lo es en general.
Encontrará ejemplos de esto en los ejercicios que aparecen más adelante.

Existen además las proposiciones inversa y contrarrecı́proca. Resumimos esto en la siguiente


tabla:

Directa p ⇒ q.
Recı́proca q ⇒ p.
Inversa ¬p ⇒ ¬q.
Contrarrecı́proca ¬q ⇒ ¬p.

Ejercicios

1. Enunciar y analizar el valor de verdad de las proposiciones recı́proca, inversa y contrarre-


cı́proca si la proposición directa es:
“Si un polı́gono es regular entonces todos sus lados tienen la misma medida. ”

2. Considerando como definición: Un triángulo equilátero es aquel que tiene sus tres lados de
igual longitud, y el siguiente teorema:
“Si un triángulo es equilátero entonces sus tres alturas miden exactamente lo mismo.”

17
a) Demuestre que cada altura mide la mitad de la longitud del lado por la raı́z de tres.
b) Demuestre el teorema.
c) Enuncie el enunciado recı́proco.
d ) ¿Es cierto el recı́proco ?
e) ¿Es cierto el contrarrecı́proco?

1.6. Métodos de demostración


Las técnicas de demostración más utilizadas son la demostración directa, la contrarrecı́proca y
reducción al absurdo. Pasamos ahora a describir cada uno de estos tipos de demostraciones.

1.6.1. Demostración directa


Suponga que un teorema es de la forma p ⇒ q. Una demostración directa del teorema es suponer
p y a partir de deducciones probar q.

Por ejemplo, “Si un número es impar entonces su cuadrado también es impar”.

Este enunciado tiene implı́cito un “para todo”, esto es su enunciado en términos de funciones
proposicionales serı́a más o menos ası́:

Considere las proposiciones p(n) :“ n es impar”, q(n) :“ n2 es impar”.

“Para todo número natural n, se tiene que si n es impar entonces n2 es impar.”

Usando notación simbólica:


∀n ∈ N : p(n) ⇒ q(n).
Muchas personas en sus primeras demostraciones omiten este hecho y creen que una de-
mostración de lo anterior es algo como lo siguiente:

“3 es impar y 32 = 9, que es impar.”

Las personas que razonan de esta forma han omitido el para todo y luego no han probado más
que un caso particular. El lector debe evitar este error bastante común entre las personas que se
inician en el estudio de la matemática.

La demostración directa serı́a:

Si n es impar entonces n = 2k − 1 para algún natural k, luego n2 = (2k − 1)2 = 4k 2 − 4k + 1 =


2(2k 2 − 2k) + 1, que es claramente un impar.

Es decir en una demostración directa de p ⇒ q, se supone p verdadero y se prueba q.

18
1.6.2. Demostración por contrarrecı́proco
Esta demostración se basa en la siguiente equivalencia lógica p ⇒ q ≡ ¬q ⇒ ¬p. Por ejemplo
supongamos que debemos probar la siguiente proposición:

“Si un número a2 es par entonces a es par.”

Escribimos la proposición anterior como:


∀a ∈ N : p(a) ⇒ q(a),
donde p(a) : “ a2 es par 2
q(a) : “ a es par ”. El contrarrecı́proco será:

∀a ∈ N : ¬q(a) ⇒ ¬p(a)
la que traducimos como:

“Si un número es a es impar entonces a2 es impar.”

que es la proposición que probamos anteriormente.

1.6.3. Demostración por reducción al absurdo


Esta se basa en el hecho de que la negación de p ⇒ q es p ∧ ¬q. En este caso para probar p ⇒ q
suponemos que se cumple p y que no se cumple q y debemos llegar a alguna contradicción.

Por ejemplo, probar el siguiente teorema:

“Si un número c verifica que c2 = 2 entonces c es irracional.”

Suponga que c2 = 2 y que no se verifica que c es irracional, entonces c es racional. Lo anterior


significa que existen enteros a y b tal que c = a/b donde esta fracción es irreducible, es decir a y b
no tienen factores comunes.
a2
Como c2 = b2 = 2, tenemos que a2 = 2b2 luego a2 es par. Por lo anterior deducimos que a es
par.

Como a es par a = 2d, reemplazando en la igualdad anterior 4d2 = 2b2 luego b2 = 2d2 , de
donde b2 es par, luego b es par.

Lo anterior prueba que a y b tienen el factor 2 en común, contrario a lo que se habı́a supuesto.
Esta contradicción prueba el teorema.

Note que hemos probado que los números ± 2 son irracionales.

Presentamos a continuación un conjuto inicial de postulados y definiciones, que tienen por


objeto que el lector se haga una idea más acabada de cómo se construye una teorı́a a partir de
elementos básicos, definiciones, postulados y teoremas, repasando de paso nuevamente los tipos de
demostraciones antes mencionadas.

19
1.7. Ejemplo de construción de una teorı́a
Consideremos una colección finita no vacı́a de personas S y los clubes formados por estas
personas donde, por un club entendemos un conjunto de personas organizadas según un cierto fin
común.

Tenemos entonces términos básicos que no se definen, sólo se tiene una idea de lo que significan y
se trabaja con ellos. En este caso: el conjunto de personas S y los clubes a los que éstas pertenecen.

Considere el siguiente conjunto de postulados y definiciones.

Postulado 1. Toda persona de S es miembro de al menos un club.

Postulado 2. Para cada par de personas de S hay uno y sólo un club al que ambas pertenecen.

Definición 1. Dos clubes que no tienen elementos en común se llaman conjugados.

Postulado 3. Para cada club hay uno y sólo un club conjugado.

Notación: Al conjugado del club C lo denotaremos por C ⊥ .

Con este conjunto de elementos básicos, postulados y definiciones es posible comenzar a es-
tablecer algunos teoremas.

Teorema 1. Toda persona de S es miembro de al menos dos clubes.

Demostración

Sea p1 una persona cualquiera de S, por el postulado 1, esta persona pertenece al menos a algún
club digamos C1 . Para el club C1 existe, por el postulado 3, un club conjugado digamos C2 = C1⊥ .
Este club está formado por alguna persona distinta de p1 digamos p2 . Por el postulado 2 la pareja
de personas {p1 , p2 } pertenecen a un único club C3 . Ahora C1 6= C3 porque p2 no pertenece a C1 .

Hemos probado ası́ que la persona p1 pertenece a los clubes C1 y C3 , luego pertenece al menos
a dos clubes.

Teorema 2. Todo club tiene al menos dos miembros.

Demostración

Por reducción al absurdo. Supongamos que existe un club C1 con exactamente un miembro p1 .
Esto es C1 = {p1 } por el teorema 1, p1 pertenece al menos a otro club digamos C2 . Como el club
C2 es distinto del club C1 éste tiene al menos a otra persona p2 . Sea C3 = C2⊥ , el conjugado de C2 .

C3 y C1 no tienen elementos en común, luego son conjugados. Para el club C3 hay dos clubes
conjugados, el club C2 y el club C1 , lo que contradice el postulado 3.

20
Esta contradicción nace de suponer que existe un club que tiene un solo elemento, luego todos
los clubes tienen al menos dos elementos.

Teorema 3. S contiene al menos cuatro personas.

Demostración

Como S es no vacı́o, S tiene al menos una persona p1 . Esta persona, por el postulado 1, es
miembro de algún club C1 . El club C1 tiene al menos dos personas por el teorema 2. Consideremos
el club C1⊥ , que existe por el postulado 3. Estos clubes tienen al menos dos personas cada uno y
no tienen elementos en común, luego en S hay al menos cuatro personas.

Dejamos la demostración del teorema 4 al lector.

Teorema 4. Hay al menos seis clubes.

Ejercicios

1. Considere la siguiente afirmación ∀ > 0, ∃δ > 0 tal que δ 2 +δ = . Determine si la afirmación


es verdadera o falsa. Justifique su respuesta y escriba la negación de la afirmación.

2. Dé dos ejemplos de Definiciones, Teoremas (con nombre y sin nombre), Lemas y Axiomas.
Explique las diferencias entre éstos.

3. Demuestre usando el método de reducción al absurdo que si x es un número real que cumple
x 6= 0 entonces x−1 6= 0.

4. Demuestre que p y q son soluciones de la ecuación cuadrática x2 + bx + c si y sólo si


p + q = −b y pq = c , (suponemos que q, p, b y c son números reales ).

5. Demuestre usando la axiomática real que (−x)(−y) = xy.

6. Determine el valor de verdad de las siguientes proposiciones. Demuéstrelas en caso de ser


verdaderas o dé un contraejemplo para probar que son falsas:

a) ∀x ∈ R(x2 + 1 > 0).


b) ∀x ∈ R(x2 + x + 1 > 0).
c) Si x es un real positivo entonces x2 < x.
1
d ) Si x es un número real y x < 1 entonces x > 1.

7. Demuestre que si el número natural a cumple que a2 es divisible por tres, entonces a es
divisible por tres.
Indicación: pruebe el contrarrecı́proco.

21

8. Demuestre por reducción al absurdo que el número 3 es irracional.

Indicación: Suponga que 3 = ab donde la fracción a
b ya esta simplificada al máximo.)

9. Considere las proposiciones, p : “En un cuadrilátero las diagonales se dimidian q : “El2

cuadrilátero es un paralelógramo”. Decida si:

a) p es suficiente para q.
b) q es suficiente para p.
c) q es necesario para p.
d ) p es necesario para q.
e) p es necesario y suficiente para q.

10. Encuentre la falla en el siguiente argumento.


Supongamos que x = y entonces x2 = xy. Sumando −y 2 a ambos lados obtenemos x2 − y 2 =
xy−y 2 . Factorizando inferimos de lo anterior que (x+y)(x−y) = (x−y)y luego simplificando
por x − y tenemos x + y = y. Como x = y de lo anterior podemos deducir que 2x = x y
simplificando por x tenemos 2 = 1. (¿Sorpresa ?)

11. Busque un enunciado del teorema de Thales de Mileto, determine los elementos involucrados:
definiciones, hipótesis, tesis, tanto del directo como del recı́proco.

12. Se sabe que la proposición p es verdadera, p : “No es cierto que un paı́s africano ha sido
ganador de alguna Copa del Mundo (fútbol)”.
Determine el valor de verdad de la siguiente proposición
“Si un paı́s africano fue ganador de una Copa del Mundo, entonces Chile será vencedor de la
próxima Copa del Mundo”.

13. Analice el siguiente planteamiento y comente si la conclusión es válida.


“O la lógica es difı́cil o no le gusta a muchos estudiantes. Si la matemática es fácil, entonces
la lógica no es difı́cil. Por tanto, si a muchos estudiantes les gusta la lógica, la matemática
no es fácil.”

14. Un Ingeniero civil industrial fallece y se encuentra con San Pedro quien le dice: hermano mı́o,
tus acciones no han sido del todo correctas en la tierra por lo que deberı́as ir al Infierno, sin
embargo he decidido darte una última oportunidad.
Estás viendo dos puertas, una es la puerta del Cielo, la otra la del Infierno. Frente a cada
puerta hay un guardia. Uno de los guardias siempre miente y el otro siempre dice la verdad.
Tienes derecho a hacer una pregunta a cualquiera de los guardias y luego escoger la puerta.
Como el ingeniero conocı́a bastante lógica hizo la pregunta correcta y pudo escoger la puerta
del Cielo.
¿Qué pregunta hizo ?

22
Capı́tulo 2

Geometrı́a plana

En este capı́tulo presentamos los resultados iniciales de la geometrı́a plana, obtenidos a partir
de algunos postulados básicos. Se utilizan los conceptos de congruencia, semejanza y área para
demostrar algunos teoremas importantes en la resolución de problemas geométricos. Se estudia la
circunferencia y el número π ası́ como la división de segmentos y el número φ.

2.1. Elementos Iniciales de la Geometrı́a Plana.


Como explicábamos, de una forma similar a la anterior, partiendo de un conjunto de eleméntos
básicos y postulados es posible construir toda la geometrı́a que conocemos.

Estudiaremos de un modo más formal la geometrı́a euclidiana, sin llegar a ser tan estrictos
como para demostrar todos los teoremas enunciados, pero sı́ dando una idea razonable de sus
fundamentos y sus consecuencias.

Aceptaremos como elementos básicos los puntos y las rectas según las nociones usuales que el
lector tiene de ellos. Asumiremos también hechos como que dos rectas distintas que se cortan lo
hacen en un único punto.

Los postulados que aceptaremos están, en un principio, muy relacionados con el lugar donde
viven las figuras: el plano. Por ejemplo, para nosotros es casi natural, si entendemos por rectas
paralelas aquellas que no se cortan, aceptar lo siguiente:

Postulado 5: Por un punto exterior a una recta existe una única paralela a ella.

Esta afirmación es enunciada de otra manera en la enseñanza secundaria:

Postulado 5’: Dos rectas distintas son paralelas si al ser cortadas por una secante forman ángulos
correspondientes de igual medida.

23
En la figura L1 // L2 si y sólo si α = β.

Si asumimos la existencia de la paralela a una recta dada por un punto dado, el postulado
anterior nos garantiza la unicidad de ésta. En efecto, supongamos que existen dos paralelas distintas
por el punto P a la recta L1 , digamos L2 y L3 , como en la figura

como L2 // L1 se tiene que α = β. Como L3 // L1 β = γ, de donde α = γ por lo que L2 coincide


con L3 .

Lo anterior se puede reducir a

Postulado 5’+ existencia de la paralela ⇒ unicidad de la paralela.

Note que del postulado 5’ también se deducen las siguientes relaciones.

Teorema 2.1.0.1. Ángulos entre paralelas. Suponga que las rectas paralelas L1 y L2 son cortadas
por una secante L, se tienen las siguientes relaciones angulares:

α1 = α5 , α2 = α6 , α4 = α8 , α3 = α7 .
α1 = α7 , α4 = α6 , α3 = α5 , α2 = α8 .
α2 + α5 = 180◦ α3 + α8 = 180◦ .

24
La demostración es directa del postulado 5’, del hecho que las medidas de los ángulos adyacentes
suplementarios suman 180◦ y que ángulos opuestos por el vértice tienen igual medida.

Por ejemplo: como L1 // L2 tenemos que α1 = α5 pero α1 + α2 = 180◦ , luego α5 + α2 = 180◦ .


Por otro lado, α3 + α2 = 180◦ , luego α3 = α5 . Es un buen ejercicio para el lector que no encuentre
triviales estas relaciones tratar de demostrarlas.

También es claro que de los teoremas anteriores se deduce el siguiente conocido teorema.

Teorema 2.1.0.2. En todo triángulo la suma de los ángulos interiores es 180◦ .

Considere el triángulo ABC de la figura. Como el punto C es exterior a la recta determinada


←→
por los puntos A y B, existe una única paralela por C a la recta AB. Tracemos esta paralela que
llamaremos L.

Por el teorema 2.1.0.1 es claro que α = α1 y que β = α2 . Como α1 + γ + α2 = 180◦ tenemos


que α + γ + β = 180◦ .

La relación anterior debe haber sido utilizada por el lector cientos de veces durante su educación
media, pero algo que tal vez no ha notado es la dependencia de este resultado con el Postulado 5,
por ser demasiado natural.

Para precisar la idea de la “naturalidad”de los postulados de la geometrı́a euclidiana pensemos


por un momento que hemos nacido en un planeta pequeño en que podemos ver su curvatura, por
lo que la superficie más razonable e intuitiva en la que se dibujan los trazos no es un plano sino
una esfera. Una pregunta inicial es: ¿qué consideraremos como “rectas.en esta superficie?

25
Dados dos puntos en la superficie de la esfera la geodésica es la lı́nea más corta que los une, la
que debe estar contenida en la superficie de la esfera por supuesto. Este serı́a el análogo de nuestros
trazos en el plano. Puede probarse, aunque no es fácil, que tales geodésicas corresponden a trazar
cı́rculos máximos que pasen por los dos puntos dados, es decir: circunferencias cuyo centro coincide
con el centro de la esfera y que pasan por los puntos en cuestión. Podrı́amos pensar en triángulos
que resultan de trazar las geodésicas que unen tres puntos de la superficie de la esfera.

Nos preguntamos si será verdad que la suma de las medidas de sus ángulos interiores sea 180◦ ,
como ocurre en el plano y antes que eso ¿cómo se medirı́an ángulos en este contexto ?

Aceptando que la medida de un ángulo corresponde a la medida de su proyección en un plano


tangente a la esfera que pasa por el vértice del ángulo, podemos verificar que existen triángulos
esféricos cuya suma de ángulos es 270◦ , como el de la figura. De hecho, la suma de los ángulos
interiores de cualquier triángulo en la esfera es mayor que 180◦ .

Es interesante que el lector analice lo siguiente. Si considera un punto P en la esfera, exterior a


una “recta”L (que en este contexto es un cı́rculo máximo en la esfera), no existen “rectas”paralelas
a L pasando por P, entendiendo por rectas paralelas dos rectas que no se cortan. Esto es por que
dos circunferencias máximas siempre se cortan en esta superficie.

Es claro que en este caso el Postulado 5 no parece para nada natural, de hecho, es falso.

Existen también otras superficies, como el hiperboloide que se muestra en la figura, donde por
un punto exterior a una “recta”pasan infinitas paralelas a la “recta”dada y donde la suma de los
ángulos interiores de cualquier triángulo es siempre menor a 180◦ .

26
¿Puede imaginar las geodésicas y completar el triángulo ABC en la figura anterior?

Concluimos entonces que el tipo de postulados desarrollados por los griegos y aceptados por
nosotros para construir la geometrı́a euclidiana, tiene ı́ntima relación con la superficie en la que se
dibujan las figuras: el plano.

Reconociendo lo anterior, iremos presentando en este capı́tulo algunos resultados de la geome-


trı́a plana que quizás sean conocidos por el lector, pero que en lo adelante deberán ser demostrados
como consecuencia de los postulados y teoremas que les preceden. Este ejercicio lógico es esencial
para desarrollar la intuición geométrica y el pensamiento cientı́fico en general.

En este camino, demostremos la siguiente afirmación.

Teorema 2.1.0.3. En todo triángulo a mayor lado se opone mayor ángulo.

Consideremos un triángulo ABC como en la figura, y supongamos que a es mayor que b.


Debemos probar que α es mayor que β.

Escojamos un punto D en el lado CB de manera que el segmento CD mida b. El triángulo


ADC es isósceles. Note que si admitimos que los ángulos basales de un triángulo isósceles son
iguales, llamando x a la medida del ángulo BAD, tenemos que β + x = α − x de donde se deduce
que α = β + 2x, luego α es mayor que β que es lo que se pedı́a demostrar.

¿Que pasa si analizamos el recı́proco del enunciado anterior?

Teorema 2.1.0.4. En todo triángulo a mayor ángulo se opone mayor lado.

Ahora nuestra hipótesis es que el ángulo α es mayor que el ángulo β y nuestra tesis es que a es
mayor que b.

27
Demostraremos lo anterior por reducción al absurdo. Supongamos que α > β y que no se tiene
que a > b entonces o bien a = b ó a < b. Si a = b entonces α = β y tenemos una contradicción. Si
a < b entonces, por el teorema anterior α < β que también es una contradicción. Luego se tiene el
teorema.

Los sencillos enunciados anteriores nos permiten probar el siguiente resultado importante cono-
cido como la desigualdad triangular.

Teorema 2.1.0.5. Desigualdad triangular. En todo triángulo la suma de dos de sus lados es siempre
mayor que el tercer lado

Prolonguemos el lado AC en el triángulo de la figura de modo que el segmento CD mida a.


Tenemos entonces el triángulo isósceles BDC. En el triángulo ADB, el ángulo opuesto al lado
AD mide β + x y es menor que el ángulo opuesto al lado AB, que mide x luego por los teoremas
anteriores
a + b > c.

Una idea muy conectada con el anterior resultado es la siguiente: si A, B y C son puntos de una
misma recta, con B situado entre A y C, es natural asumir como postulado que AB + BC = AC.
Aún más, el recı́proco de esta afirmación se obtiene como consecuencia de la desigualdad triangular
(basta probarlo por reducción al absurdo). Luego podemos decir que:

Teorema 2.1.0.6. Tres puntos del plano A, B y C son colineales, con B entre A y C si y sólo si
AB + BC = AC.

28
El lector interesado habrá notado que todos los resultados anteriores se basan en una propie-
dad, que es que “un triángulo isósceles tiene dos ángulos de igual medida”. Este hecho no ha sido
demostrado de ningún otro postulado. La primera tentación es pedir que este sea uno más de ellos.
Esto, sin embargo, nos plantea el riesgo de poner como postulado toda propiedad que no pueda ser
demostrada, con lo que el conjunto de éstos podrı́a crecer innecesariamente. Vamos sin embargo a
establecer un único postulado que nos permitirá obtener muchos otros principios y teoremas, este
será ası́ una de las piedras angulares sobre las que descansa la teorı́a que estamos desarrollando.

Postulado de Hilbert: Si dos lados y el ángulo comprendido de un triángulo tienen las mismas
medidas que, dos lados y el ángulo comprendido de otro triángulo, entonces cada uno de los ángulos
y el lado restantes del primer triángulo tienen respectivamente las mismas medidas que los ángulos
y el lado correspondientes del segundo.

Veamos como, usando sólo este postulado, probamos que los ángulos basales de un triángulo
isósceles tienen la misma medida.

Teorema 2.1.0.7. Los ángulos basales de un triángulo isósceles tienen la misma medida.

Para probar esto consideremos un triángulo isósceles ABC de base AB. Nuestra hipótesis es
que los lados AC y BC miden lo mismo, debemos probar que α = β.

Prolonguemos los lados AC y BC determinando puntos D y E de modo que los segmentos


AD y BE midan lo mismo. Los triángulos CDB y CEA verifican el postulado de Hilbert. De esto
obtenemos la igualdades angulares (∗) α + δ2 = β + δ1 , φ1 = φ2 y la igualdad de las longitudes de
los lados AE y BD. Ahora los triángulos DBA y EAB verifican también el postulado de Hilbert,
luego δ1 = δ2 . Reemplazando en (∗) obtenemos que α = β que es lo que tenı́amos que demostrar.

Esto ya da un fundamento sólido a las desigualdades anteriores. Analicemos el recı́proco del


enunciado anterior.

Teorema 2.1.0.8. Si un triángulo tiene dos ángulos de igual medida, entonces es isósceles.

Probaremos lo anterior por reducción al absurdo. Supongamos que los ángulos basales del
triángulo ABC son iguales pero que los lados a y b son distintos. Sin perder generalidad supongamos
que a > b. Tracemos sobre el lado CB el punto D de modo que el segmento CD mida b.

29
Como el triángulo ADC es isósceles por el enunciado directo el ángulo ADC mide α − β.
Sumando ángulos α + β = α − β por lo que el ángulo β = 0, de modo que los puntos D y B
coinciden, luego a = b. Esto contradice el supuesto que a > b y esta contradicción prueba el
teorema.

Ejercicios

1. Demostrar que la suma de las diagonales de un cuadrilátero convexo es mayor que el semi-
perı́metro del cuadrilátero.
2. Demostrar que en todo triángulo, cada transversal de gravedad es menor que la semisuma
de los lados adyacentes.
3. Demostrar que en todo triángulo, a mayor lado, corresponde una menor transversal de
gravedad.

2.2. Congruencia y Semejanza


En la Geometrı́a estos conceptos son fundamentales, pues permiten reducir el estudio de nu-
merosas figuras a otras ya estudiadas previamente y para las cuales se conocen sus propiedades.

Desde pequeños entendimos a dos figuras como iguales si podı́amos mover una y llevarla “sin de-
formarla”sobre la otra de modo que coincidieran. Incluso muchas veces recortamos y superpusimos
figuras como una forma de expresar este concepto.

Por figuras semejantes tenemos la idea de aquellas figuras que tienen la misma “forma”pero
no necesariamente el mismo “tamaño”. Una imagen que posiblemente nos indicaron para recordar
esto es realizar una figura en una hoja y pedir una ampliación o una reducción de esta figura en
un centro de fotocopiado. La figura original y la que nos entregan de la fotocopiadora son figuras
semejantes.

Dos figuras geométricas se dicen congruentes si tienen la misma forma y el mismo tamaño, por
ejemplo: dos cuadrados con lados de la misma longitud, dos circunferencias de igual radio, dos
segmentos del mismo largo, etc. Nótese que no importa la posición relativa de cada par de figuras

30
ni su orientación en el plano, sino que pueda llevarse una figura a la otra sin deformarla de manera
que se superpongan exactamente.

Dos figuras geométricas se dicen semejantes si tienen la misma forma, pero no necesariamente
el mismo tamaño por ejemplo: dos cuadrados cualquiera, dos circunferencias, dos segmentos, etc.
Nuevamente no importa la orientación de cada par de figuras en el plano. En este caso una de las
figuras es una ampliación de la otra.

Estudiaremos con mayor detenimiento la aplicación de los conceptos que presentamos anteri-
ormente al caso de los triángulos.

Los triángulos son polı́gonos de tres lados que, por su simplicidad, resultan de suma importancia
en la Geometrı́a al estudiar figuras más complejas. De manera general denotaremos los vértices,
lados y ángulos de un triángulo de la manera que se indica en la figura:

Cuando comparemos dos triángulos, resultará importante precisar cuáles son los elementos
correspondientes de cada figura, pues puede que las orientaciones de éstas no coincidan. Note que
si recorremos el contorno del triángulo en sentido antihorario partiendo del vértice A, la orientación
A − B − C es distinta de la orientación A − C − B.

Los conceptos descritos, aplicados a los triángulos, nos permiten enunciar las siguientes propie-
dades que se desprenden directamente de las nociones de semejanza y congruencia:

• Si dos triángulos son semejantes o congruentes, entonces sus tres pares de ángulos correspon-
dientes son respectivamente iguales.

• Si dos triángulos son congruentes, entonces sus tres pares de lados correspondientes son
respectivamente iguales.

• Si dos triángulos son semejantes, entonces sus tres pares de lados correspondientes son res-
pectivamente proporcionales.

Sin embargo, lo que trataremos a continuación es el problema inverso al anterior, es decir:


¿cómo saber si dos triángulos cualquiera son congruentes o semejantes? ¿qué elementos de ambos
triángulos debemos comparar para asegurar lo anterior? Las respuestas a estas preguntas son los
que se conocen como Criterios de Congruencia o Semejanza de triángulos.

Criterios de Congruencia. En vez de mostrar precisamente la demostración de cada teorema,


presentaremos algunos ejemplos con la idea de ilustrar cómo, a partir de estos teoremas, es posibe

31
probar muchas propiedades de las figuras planas. El lector debe tratar de ejercitar la deducción
utilizando sólo los postulados y teoremas precedentes.

Teorema 2.2.0.9. (Criterio LLL) Si dos triángulos tienen sus tres lados respectivamente iguales,
entonces los triángulos son congruentes.

Ejemplo. En un triángulo isósceles 4ABC, de base AB, el segmento que une el vértice C con
el punto medio de la base AB es bisectriz del ángulo ∠ACB y perpendicular a la base.

En el triágulo isósceles 4ABC sea M el punto medio de la base AB. Tenemos como hipótesis
que AC = BC y también que AM = BM. Esto significa que los triángulos 4ACM y 4BCM
tienen sus tres pares de lados respectivamente iguales lo que, por el Teorema 2.2.0.9, significa que
son congruentes.

Si 4ACM ∼ = 4BCM , entonces tienen sus tres pares de ángulos correspondientes respectiva-
mente iguales.

∠ACM = ∠BCM ⇒ CM es bisectriz del ∠ACB

∠AM C = ∠BM C y ∠AM C + ∠BM C = 180◦ ⇒ CM es perpendicular a AB.

Teorema 2.2.0.10. (Criterio LAL) Si dos triángulos tienen dos lados y el ángulo comprendido entre
ellos respectivamente iguales, entonces los triángulos son congruentes.

Ejemplo. Si las diagonales de un cuadrilátero convexo se dimidian, entonces es un parale-


lógramo.

En el cuadrilátero ABCD se tiene que AM = MC y BM = MD. Además ∠AM D = ∠BM C


y ∠AM B = ∠DM C por ser opuestos por el vértice.

32
Los triángulos 4AM D y 4BM C tienen dos lados y el ángulo comprendido entre ellos respec-
tivamente iguales luego, por el Teorema 2.2.0.10 son congruentes. Esto implica que los ángulos
correspondientes de ambos triángulos tienen la misma amplitud, por lo que ∠DAM = ∠BCM , lo
que significa que AD k BC. Por un argumento similar se demuestra que 4AM B ∼ = 4CM D y que
los lados AB y DC son paralelos.

Teorema 2.2.0.11. (Criterio ALA) Si dos triángulos tienen un lado y los dos ángulos adyacentes a
él respectivamente iguales, entonces los triángulos son congruentes.

Ejemplo. Los lados opuestos de un paralelógramo son iguales.

En el paralelógramo ABCD, AB k DC y BC k AD.

Tenemos que ∠BAC = ∠DCA y ∠DAC = ∠BCA por ser alternos internos entre paralelas. Los
triángulos 4ABC y 4ACD tienen un lado común y sus dos ángulos adyacentes respectivamente
iguales luego, por el Teorema 2.2.0.11 son congruentes. Esto implica que los lados correspondientes
de ambos triángulos tienen la misma longitud, por lo que AB = DC y BC = AD.

Teorema 2.2.0.12. (Criterio LLA) Si dos triángulos tienen dos lados y el ángulo opuesto al mayor
de ellos respectivamente iguales, entonces los triángulos son congruentes.

Los criterios de congruencia descritos permiten que, si queremos verificar cuándo dos triángulos
son congruentes, no tengamos que medir todos los lados y todos los ángulos de cada triángulo, sino
que basta con algunos de ellos, escogidos de cierta manera para comprobar la congruencia. Por eso
decimos que son condiciones suficientes para la congruencia de triángulos.

Una primera mirada a estos criterios de congruencia pareciera indicar que de los seis elementos
que hay en un triángulo (tres lados y tres ángulos) basta comprobar la igualdad de cualquier trı́o
de ellos en ambos triángulos para que se verifique la congruencia. Sin embargo, eso no es cierto; ya
en el Criterio LLA es necesario imponer una condición adicional y es que el ángulo seleccionado se
oponga al mayor de los lados seleccionados, pues de lo contrario, la congruencia no necesariamente
se verifica. Mostraremos un caso donde dos triángulos tienen dos lados y un ángulo respectivamente
iguales y sin embargo no son congruentes.

33
El siguiente ejemplo requerirá del lector la aplicación de algún criterio de congruencia (no
se especifica cuál), pero no de manera directa. Lo interesante de la Geometrı́a es desarrollar la
capacidad de modelar el problema de forma que se puedan utilizar las herramientas que conocemos.
Obviamente, esto no siempre es posible, pero ampliar estas habilidades es uno de los objetivos de
su estudio.
Ejemplo. Considere un punto en el interior de un ángulo, formado por dos rectas `1 y `2 .
Determine un punto Q en `1 y un punto R en `2 de modo que P sea punto medio de QR.

El lector que se enfrenta a un problema de este tipo por primera vez podrá hacer sin dificultad
el esquema que se muestra más arriba, con lo que tendrá superada la primera etapa: encontrar un
modelo visual que facilite la solución. ¿Y luego, qué hacer? se preguntará. He aquı́ el verdadero
salto cualitativo que se requiere para resolver, en general, cualquier problema geométrico: no basta
con “dibujar.el enunciado.

Un alternativa podrı́a ser considerar resuelto el problema y ver qué argumento geométrico puede
utilizarse. Por ejemplo, si por Q y R se trazan paralelas a las rectas originales, el punto P resulta
ser el punto de intersección de las diagonales del paralelógramo resultante.

Ahora es más fácil intentar una estrategia de solución. Bastará construir el paralelógramo de la
figura. Para esto basta trazar el segmento que une a P con el vértice del ángulo y copiarlo sobre la
misma recta, al otro lado de P . Por el extremo resultante se trazan rectas paralelas a las origiales.
Donde se corten estas rectas con `1 y `2 , estarán los puntos Q y R buscados.

Sugerimos al lector que trate de resolver los siguientes ejercicios como una aplicación de los
criterios antes aprendidos.

Ejercicios.

1. Demostrar que los puntos de la simetral de un trazo equidistan de los extremos del trazo.

34
2. Probar el recı́proco del enunciado anterior.
3. Probar que las simetrales de los lados de un triángulo concurren en un punto (llamado
circuncentro o centro de la circunferencia circunscrita, esto es que pasa por los tres vértices
del triángulo).
4. Demostrar que los puntos de la bisectriz de un ángulo equidistan de los lados del ángulo.
5. Probar el recı́proco del enunciado anterior.
6. Demostrar que las bisectrices de los ángulos interiores de un triángulo concurren en un único
punto (llamado incentro o centro de la circunferencia inscrita).
7. Probar que en un paralelógramo los ángulos interiores opuestos son de igual medida.
8. Probar que si en un cuadrilátero convexo cada par de ángulos interiores opuestos son de igual
medida entonces es un paralelógramo.

9. Si en un cuadrilátero convexo hay un par de lados paralelos e iguales, entonces es un par-


alelógramo.
10. Probar que las diagonales de un rectángulo son congruentes.
11. Si en un paralelógramo las diagonales son congruentes entonces es un rectángulo.
12. Probar que en un rombo las diagonales se dimidian y son perpendiculares.
13. Demostrar que las transversales de gravedad de un triángulo concurren en un punto (llamado
centro de gravedad) y que se cortan en razón 2 : 1.

Criterios de Semejanza. De la misma manera que en el caso del criterio LLA, no basta
que dos triángulos tengan los tres ángulos respectivamente iguales para ser congruentes, como se
muestra en la siguiente figura.

Sin embargo, la figura anterior nos sugiere el siguiente teorema:

Teorema 2.2.0.13. (Criterio AAA) Si dos triángulos tienen sus tres ángulos respectivamente iguales,
entonces los triángulos son semejantes.

35
Teorema 2.2.0.14. (Criterio AA) Si dos triángulos tienen dos ángulos respectivamente iguales,
entonces los triángulos son semejantes.

Es claro que si en dos triángulos hay dos ángulos iguales, el tercero también coincidirá, pues la
suma de los tres ángulos debe ser 180◦ , por lo que se aplica el Teorema 2.2.0.13.

Ejemplo. En un triángulo rectángulo, la altura trazada desde el ángulo recto divide al triángulo
en dos triángulos semejantes al original.

El triángulo 4ABC es rectángulo en C, sea H el punto donde la altura trazada desde C corta
a la base AB. Entonces CH ⊥ AB y se tiene que ∠ACB = ∠AHC = 90◦ .

Por otro lado ∠CAB = ∠CAH. Esto indica que los triángulos 4ABC y 4AHC tienen dos
pares de ángulos respectivamente iguales lo que, por el Teorema 2.2.0.14, implica que son seme-
jantes.

Análogamente se puede probar que 4ABC ∼ 4BHC, por lo que se obtiene el resultado pedido:

4AHC ∼ 4ABC ∼ 4BHC.

El resultado anterior proporciona tres importantes teoremas que se refieren al triángulo rectángu-
lo.

Lema 2.2.1. Lema de la altura. En todo triángulo rectángulo el cuadrado de la altura correspon-
diente a la hipotenusa es igual al producto de los segmentos que ésta determina sobre aquella.

Lema 2.2.2. Lema del cateto. En todo triángulo rectángulo el cuadrado de un cateto es igual al
producto entre la hipotenusa y la proyección del cateto sobre ella.

Teorema 2.2.0.15. Teorema de Pitágoras. Si un triángulo rectángulo tiene catetos de longitudes a


y b e hipotenusa de longitud c entonces se cumple la relación c2 = a2 + b2 .

Demostraremos los tres resultados como consecuencia del ejemplo presentado antes. Dada la
semejanza de los triángulos 4ABC, 4BHC y 4AHC se tiene, según se observa en la figura, que
los lados correspondientes de cada par de triángulos son proporcionales:

36
h q
4AHC ∼ 4BHC ⇒ =
p h

c b
4AHC ∼ 4ABC ⇒ =
b p
c a
4BHC ∼ 4ABC ⇒ =
a q

De la primera igualdad se tiene el resultado del Lema 2.2.1: h2 = pq.

El Lema 2.2.2 se obtiene directamente de la segunda y tercera igualdad: b2 = cp y a2 = cq.

Ahora, sumando estas últimas igualdades tenemos que a2 + b2 = cp + cq = c(p + q) = c2 , que


es la tesis del Teorema de Pitágoras.

Volviendo al tema de la semejanza, sabemos que dos triángulos semejantes tienen sus tres
ángulos correspondientes respectivamente iguales y sus tres lados correspondientes respectivamente
proporcionales, pues la semejanza amplı́a o reduce todas las longitudes por el mismo factor k. Esta
idea permite obtener otros tres criterios de semejanza de triángulos a partir de los teoremas de
congruencia ya demostrados.

Teorema 2.2.0.16. (Criterio LLLP) Si dos triángulos tienen tres lados respectivamente propor-
cionales, entonces los triángulos son semejantes.

Teorema 2.2.0.17. (Criterio LALP) Si dos triángulos tienen dos lados respectivamente propor-
cionales y el ángulo comprendido respectivamente igual, entonces los triángulos son semejantes.

Teorema 2.2.0.18. (Criterio LLAP) Si dos triángulos tienen dos lados respectivamente propor-
cionales y el ángulo opuesto al mayor de estos lados respectivamente igual, entonces los triángulos
son semejantes.

Reuniendo lo discutido hasta el momento acerca de congruencia y semejanza de triángulos,


resumiremos en el siguiente esquema las posibilidades que existen al seleccionar tres elementos
de un triángulo para comparar sus medidas y las conclusiones que se derivan para dos triángulos
distintos.

37
Para ejercitarse en el uso de la congruencia y de la semejanza sugerimos resolver los siguientes
ejercicios.

Ejercicios

1. Demostrar que la suma de las diagonales de un cuadrilátero convexo es mayor que el semi-
perı́metro del cuadrilátero.
2. Demostrar que al unir los puntos medios de los lados de un cuadrilátero se forma un par-
alelógramo.
3. Demostrar que el segmento que une el vértice del ángulo recto con el punto medio de la
hipotenusa de un triángulo rectángulo mide la mitad de la hipotenusa.
4. Construir la bisectriz de un ángulo cuyo vértice es inaccesible.
5. Considere puntos P y Q exteriores y a un mismo lado de a una recta L. Determinar un punto
R en la recta L que verifique que la suma de las distancias PR + RQ sea mı́nima.

38
6. Demostrar que las alturas de un triángulo concurren en un punto llamado ortocentro del
triángulo. Demostrar que la distancia desde el vértice hasta el punto de intersección de sus
alturas, es dos veces mayor que la distancia desde el centro del cı́rculo circunscrito hasta el
lado opuesto.

7. Demostrar que el radio de la circunferencia inscrita en un triángulo rectángulo se determina


por la fórmula r = a+b−c
2 , donde a y b son los catetos y c la hipotenusa.
8. Los catetos de un triángulo rectángulo son a y b. Encontrar la distancia desde el vértice del
ángulo recto hasta el punto de la circunferencia inscrita más próximo a él.

9. Los lados de un triángulo ABC son BC = a, CA = b, AB = c. Determinar la razón en la


que el punto de intersección de las bisectrices divide a la bisectriz del ángulo en B.
10. Sobre la base AC de un triángulo isósceles se toma un punto M de manera que AM =
a, MC = b. En los triángulos ABM y CBM están inscritas circunferencias. Hallar la
distancia entre los puntos de tangencia del lado BM con estas circunferencias.
11. Un triángulo rectángulo tiene trazada la bisectriz del ángulo recto. Hallar la distancia entre
los puntos de intersección de las alturas de los dos triángulos formados si los catetos del
triángulo son a y b.
12. En una circunferencia de centro O se trazan por los extremos de un diámetro AB dos cuerdas
paralelas AC y BD. Demostrar que los ángulos ∠ACO y ∠BDO son iguales.
13. En una circunferencia de centro O se traza un diámetro AB y una cuerda AC tales que
∠CAB = 30◦ . La tangente en el punto C intersecta a la propongación de AB en el punto
D. Demostrar que el triángulo ACD es isósceles.
14. En una circunferencia de centro O se trazan dos radios OA y OB y una cuerda MN per-
pendicular a la bisectriz del ángulo ∠AOB. La cuerda MN intersecta a los radios OA y OB
en los puntos C y D. Demostrar que MC = DN y CA = DB.
15. Demostrar que la suma de las perpendiculares trazadas a los lados desde un punto interior
de un triángulo equilátero, es igual a la altura del triángulo.
16. Demostrar que la recta que une los pies de las alturas iguales de un triángulo isósceles es
paralela a la base.
17. Dado un triángulo ABC, rectángulo en C, se prolonga el cateto AC en una longitud CD
igual a AC. Luego se traza la perpendicular desde D a la hipotenusa AB, la cual intersecta
al cateto BC en E. Demostrar que AE y BD son perpendiculares.

18. En un rectángulo ABCD (AB > BC), se trazan CE perpendicular a BD y CF bisectriz del
ángulo ∠ACE (con F en AB). Demostrar que FB = BC.
19. Construir las tangentes desde un punto dado a una circunferencia dada.
20. Demostrar que las distancias desde el vértice A de un triángulo ABC hasta el punto de
tangencia de los lados AB y AC con la circunferencia inscrita son iguales a s − a donde 2s
es el perı́metro del triángulo ABC.

39
21. En el lado de un ángulo recto con vértice en el punto O se toman dos puntoa A y B, siendo
las medidas OA = a y OB = b. Hallar el radio de la circunferencia que pasa por A, por B y
que es tangente al otro lado del ángulo.

22. Demostrar que la suma de las distancias de cualquier punto de la base de un triángulo isósceles
hasta sus lados, es igual a la altura del triángulo correspondiente a uno de sus lados.
23. Construir las tangentes comunes a dos circunferencias dadas.
24. Determinar el lugar geométrico de los puntos del plano desde los cuales se observa un trazo
con un ángulo constante. Note que si el ángulo en cuestión mide π/2, entonces el lugar
geométrico es la circunferencia de diámetro igual al trazo.
25. Probar que la bisectriz de un ángulo interior de un triángulo divide al lado opuesto en la
razón de los otros dos lados del triángulo.
←→
26. Sobre una recta AB se toma un punto P entre A y B. Luego por A y B se trazan dos
paralelas cualesquiera sobre las cuales se toma AM = AP, BN = BP, de un mismo lado de
←→
AB. Sea C el punto medio de MN. Demostrar que el triángulo ABC es rectángulo en C.
27. Demostrar que si en un cuadrilátero ABCD los ángulos en B y D son rectos, entonces las
bisectrices de los ángulos en A y C son paralelas.
28. Dado un paralelógramo ABCD, donde AB = 2BC, sea M el punto medio de DC. Demostrar
que el ángulo ∠AM B es recto.
←→ ←→
29. Demostrar que las rectas HF y GE que unen los puntos medios de los lados opuestos de un


cuadrilátero y la recta IJ que une los puntos medios de las diagonales son concurrentes.
30. Dado un triángulo ABC; M , N y P son puntos medios de los lados AB, BC y CA, respec-
tivamente y CH es altura. Demostrar que el cuadrilátero M HN P es un trapecio isósceles.
31. Si la circunferencia inscrita a un triángulo ABC toca a los lados BC y AC en los puntos X
e Y , respectivamente, demostrar que CX + CY = BC + CA − AB.
32. En el triángulo ABC rectángulo en C pitagórico 3, 4, 5 se traza la altura CE. Sean r1 y r2
los radios de las circunferencias inscritas en los triángulos AEC y ECB y r3 el radio de la
circunferencia inscrita al triángulo ABC. Probar que con r1 , r2 y r3 puede construirse un
triángulo rectángulo.
33. Demostrar que la proyección del punto medio de un cateto sobre la hipotenusa, determina
sobre ella dos trazos, cuya diferencia de cuadrados es igual al cuadrado del otro cateto.
34. Probar que el ángulo inscrito en una circunferencia mide la mitad del ángulo del centro que
subtiende el mismo arco.
35. Desde un punto P exterior a la circunferencia, se traza una secante PBC y la tangente PA.
Determinar el lugar geométrico del ortocentro del triángulo ABC cuando la secante varı́a.
36. En un cuadrilátero ABCD se conocen los ángulos ∠DAB = 90◦ , ∠DBC = 90◦ , además las
medidas DB = a y DC = b. Hallar las distancias entre los centros de las circunferencias,
una de las cuales pasa por los puntos D, A, B y la otra por los puntos B, C, D.

40
←→ ←→
37. Se da una circunferencia y un punto A fuera de ésta. AB y AC son tangentes a la circun-
ferencia (B y C son los puntos de tangencia). Demostrar que el centro de la circunferencia
inscrita en el triángulo ABC se halla en la circunferencia dada.

38. El triángulo equilátero ABC está inscrito en una circunferencia y en el arco BC se toma un
punto arbitrario M. Demostrar que AM = BM + CM.
39. Expresar el lado del decágono regular a través de R, que es el radio de la circunferencia
circunscrita.

2.3. Áreas y el Teorema de Tales.

El concepto de área de una región puede entenderse como una medida del espacio que la región
ocupa en el plano. Por supuesto que esto ha sido desarrollado desde temprana edad, aunque pocas
veces internalizado por el estudiante. Posiblemente una idea de este concepto fue lograda al dibujar
regiones sobre el papel milimetrado y contar cuántos cuadrados de un milı́metro de lado estaban
contenidos en la región. Si en la región entraban exactamente 20 de estos cuadrados, decı́amos que
la región ocupaba un área de 20 centı́metros cuadrados.

Ocurrı́a a veces que no era posible determinar un número exacto de cuadrados contenidos en la
región debido a la forma de su frontera, como pasa por ejemplo con un cı́rculo. En este caso, ¿cómo
determinar el área ? ¿Existen regiones del plano para las que no sea posible asociar un área ? Éstas
son preguntas bastante profundas que el lector quizás aclare algún dı́a. Partiremos formalizando el
concepto intuitivo de área, a través de algunos postulados, para las regiones más simples que son
los polı́gonos.

Postulados de áreas

Suponemos definida una función a del conjunto P de todos los polı́gonos cerrados en los números
reales positivos, que verifica los siguientes postulados.

Postulado 1: Regiones poligonales adyacentes tienen áreas aditivas.

41
En el caso de la figura, las regiones R1 y R2 son adyacentes porque comparten un lado común.
El postulado establece que:

a(R1 ∪ R2 ) = a(R1 ) + a(R2 )

Postulado 2: Polı́gonos congruentes tienen igual área.

Postulado 3: El área de un rectángulo es igual al producto de las longitudes de dos lados


contiguos.

Teorema 2.3.0.19. Área del paralelógramo. El área de un paralelógramo es igual al producto de


las longitudes de su base y su altura.

Considere el paralelógramo ABCD con base AB y altura DE de medidas b y h respectivamente.


Debemos probar que el área de este paralelógramo es bh.

a(#ABCD) = a(4AED) + a(EBCD) = a(4BF C) + a(EBCD) = a(EF CD)


Lo anterior es consecuencia de la congruencia de los triángulos AED y BF C. Luego por el
postulado 3 como el área del rectángulo EF CD es bh el área del paralelógramo es bh.

Teorema 2.3.0.20. Área del triángulo. El área de un triángulo es igual al semiproducto de las
longitudes de una base por la altura correspondiente.

Considere el triángulo ABC de base y altura de medidas c y h respectivamente. Debemos probar


que su área puede ser calculada como a(4ABC) = ch/2. Para probar esto, la idea es utilizar el
área anteriormente demostrada. Tracemos por el punto C una paralela a la base AB y por el

42
punto B una paralela a el lado AC. Estas dos rectas se cortan en el punto D, como se muestra en
la figura. El lector puede verificar que los triángulos ABC y DCB son congruentes, luego por el
postulado 2 tienen igual área.

a(4ABC) + a(4BDC) = a(#ABDC) = ch,


luego
2a(4ABC) = ch,
de donde se deduce claramente el resultado.

Teorema 2.3.0.21. Área del trapecio. El área de un trapecio es igual al producto de la semisuma
de las longitudes de sus bases y la longitud de su altura.

Considere un trapecio ABCD con lados paralelos, las bases AB y CD de medidas a y b


respectivamente. Por el punto medio del lado AD tracemos la paralela al lado BC, que corta a las
bases en los puntos P y Q como se aprecia en la figura.

El lector puede verificar que los triángulos AQM y DP M son congruentes, por lo que tienen
la misma área.

a(ABCD) = a(4AQM ) + a(QBCDM ) = a(4DP M ) + a(QBCDM ) = a(#QBCP ).

Luego sólo falta calcular la longitud de la base del paralelógramo QBCP. Para ello sea AQ =
PD = x. Como los lados opuestos de un paralelógramo miden lo mismo a − x = b + x de donde
x = a2 − 2b . De lo anterior

a b (a + b)
QB = PC = b + x = b + − = .
2 2 2
(a+b)
Luego el área del paralelógramo QBCP es 2 h, lo que demuestra el teorema.

Teorema 2.3.0.22. Teorema de Tales. Si por un punto de un lado de un triángulo trazamos la


paralela a otro de sus lados, entonces se determinan segmentos proporcionales.

43
En el triángulo ABC, por el punto P del lado AC se ha trazado la paralela al lado AB. Si las
medidas de los segmentos CP, PA, CQ y QB son x, y, z y w respectivamente y las medidas de
los trazos paralelos PQ y AB son a y b, el teorema establece que
x z a x z
= y que = = .
y w b x+y z+w
Para demostrar el teorema utilizaremos dos lemas que pueden ser verificados directamente por
el lector.

Lema 2.3.1. Si dos triángulos tienen igual base, la razón de sus áreas es la razón de las alturas
correspondientes.

Lema 2.3.2. Si dos triángulos tienen igual altura, la razón de sus áreas es la razón de las bases
correspondientes.

Pasamos ahora a demostrar el teorema.

Note primero que los triángulos AQP y P QC tienen la misma altura h respecto a las bases x
e y. Por lo que la razón entre sus áreas es la razón entre sus bases, esto es

a(4P QC) x
= .
a(4AQP ) y

44
Si hacemos el mismo análisis anterior, pero ahora mirando el lado derecho, los triángulos BP Q
y QP C tienen la misma altura h2 respecto a los lados w y z respectivamente, luego la razón entre
sus áreas, es la razón entre sus bases, esto es

a(4P QC) z
= .
a(4BQP ) w
x z
Luego para probar que = sólo necesitamos verificar que las áreas de los triángulos AQP
y w
y BP Q son iguales. Como notará, ambos triángulos tienen la misma base “a”, sus áreas serán
iguales si sus alturas tienen la misma medida. Esto sucede sólo si las rectas son paralelas, que es
nuestra hipótesis inicial. Lo anterior demuestra el teorema.

La relación entre las longitudes de los segmentos paralelos resulta de trazar la paralela al lado
AC, en el punto Q.

Utilizando el resultado anterior tenemos que

45
b−a w b z+w
= Sumando 1 a ambos lados = .
a z a z

Que es lo que tenı́amos que probar.

Note que en realidad, de la demostración del teorema directo se deduce el recı́proco del teorema
de Tales.

Teorema 2.3.0.23. Recı́proco del teorema de Tales. Si una recta que corta dos lados de un triángulo
determina sobre estos segmentos proporcionales, entonces esta recta es paralela al tercer lado.

Esto es por que si xy = wz , entonces por el análisis anterior tenemos que los triángulos AQP
y BP Q tienen la misma área. Como ambos comparten la base “a”, de esto se deduce que tienen
alturas de la misma medida respecto a esa base por lo que los segmentos AB y PQ son paralelos.

Ejemplo:

Considere el hexágono regular ABCDEF de la figura, donde la medida del lado AB es a.

1. Probar que AC ∼
= BF.

Solución:

Hipótesis: ABCDEF hexágono regular

FA = AB = BC = a

En los triángulos ∆ABF y ∆ABC se cumple que :

FA ∼= BC y AB es lado común a ambos triángulos, luego por hipótesis: ∠F AB ∼ = ∠ABC


porque los ángulos son todos iguales (en este caso a 120◦ ). Luego utilizando el criterio de
congruencia LAL se tiene que: ∆ABF ∼ = ∆ABC por tanto, por ser lados correspondientes
de dos triángulos congruentes se tiene :

Tesis : AC ∼
= BF

46
2. Calcular el área y el perı́metro del 4ABC en términos de a.

Solución:

Inscribimos el hexágono en una circunferencia cuyo radio será necesariamente igual a a.

En la figura, ∆AOB ∼
= ∆BOC y ambos son equiláteros.

la longitud AC = 2h, donde “h.es la altura del ∆AOB equilátero. Como


Luego se tiene que √
3 √
sabemos que h = a, luego AC = 3a. De lo anterior, el perı́metro del ∆ABC es p =
√ 2√
a + a + 3a = a(2 + 3)

AC · h
El área del ∆ABC = . Para calcular h, en la figura ABCO es un paralelógramo, luego
2
OB r a
sus diagonales se dimidian y la altura del ∆ABC resulta ser = = . Luego el área
√ √ 2 2 2
3a · a/2 3 2
del ∆ABC = = ·a .
2 4

También puede observar que los triángulos ∆ABC y ∆ABO tienen la misma base AB e
igual altura correspondiente a esa base, luego tienen igual área. De lo anterior se deduce
2√
inmediatamente que el área del ∆ABC es a4 3.

Ejercicios

1. Las diagonales de un cuadrilátero convexo son a y b y los segmentos que unen los puntos
medios de los lados opuestos son iguales. Hallar el área del cuadrilátero.
2. Alrededor de una circunferencia está circunscrito un trapecio isósceles con lado igual a l,
siendo una de las bases de éste igual a a. Hallar el área del trapecio.
3. Los vértices de un hexágono regular con lado a son los centros de circunferencias cuyos
radios son iguales a √a2 . Hallar el área de la parte del hexágono dispuesta fuera de estas
circunferencias.

47
4. Se dan tres circunferencias de radio r que se rozan de dos en dos. Hallar el área del triángulo
formado por tres rectas, cada una de las cuales es tangente a dos circunferencias y no toca a
la tercera.

5. Tenemos dos triángulos con un vértice A común, los demás vértices se encuentran en dos
rectas que pasan por A. Demostrar que la razón entre las áreas de estos triángulos es igual
a la razón entre los productos de los dos lados de cada triángulo que tienen al vértice A.
6. La hipotenusa de un triángulo rectángulo es igual a c y uno de los ángulos agudos es igual
a 30◦ . Encontrar el radio de la circunferencia con centro en el vértice del ángulo de 30◦ . que
divide al triángulo dado en dos partes equivalentes.
7. La transversal de gravedad de un triángulo rectángulo es igual a m y divide al ángulo recto
en razón 1 : 2. Hallar el área del triángulo.
8. Si el área de un triángulo equilátero es A, calcular el área del cuadrado inscrito en la circun-
ferencia circunscrita al triángulo.
9. Dado un cuadrilátero ABCD. ¿Qué punto E debe elegirse sobre la diagonal AC de modo
que la lı́nea BED divida al cuadrilátero en dos partes equivalentes.
10. Dos rectas paralelas a las bases de un trapecio dividen cada uno de sus lados en tres partes
iguales. Todo el trapecio se encuentra dividido por ellas en tres partes. Hallar el área de la
parte media, si las áreas de las partes extremas son S1 y S2 .
11. La base AB de un trapecio ABCD es a y la base CD es b. Hallar el área del trapecio si se
sabe que sus diagonales son las bisectrices de los ángulos ∠DAB y ∠ABC.
12. En los lados AB y AD de un rombo ABCD se eligen puntos M y N de manera que las
←−→ ←→
rectas M C y N C dividen al rombo en tres partes de áreas iguales. Hallar MN si la longitud
de BD = d.
13. En el lado AB de un triángulo ABC se toman dos puntos M y N de manera que AM :
MN : NB = 1 : 2 : 3. Por los puntos M y N se trazan paralelas al lado AC. Hallar el área de
la parte del triángulo comprendida entre estas rectas, si el área del triángulo ABC es igual
a S.
14. El área de un rombo es S y la suma de sus diagonales m. Hallar el lado del rombo.

2.4. Construcciones geométricas elementales.


Esta sección resultará eminentemente técnica para el lector, incluso pueden considerarse las
construcciones presentadas como ejemplos concretos donde se aplican los teoremas demostrados
en secciones anteriores. La idea es contar con herramientas bien fundamentadas que permitan
entender y desarrollar con mayor consistencia los temas que se tratarán más adelante.

Entenderemos por construcciones geométricas aquellas que pueden ser realizadas solamente
utilizando dos instrumentos: la regla y el compás. Esto limitará nuestro accionar pues, por ejemplo,

48
no podremos medir la amplitud de ángulos (no contamos con un transportador) ni podremos medir
cualquier longitud (nuestra regla tendrá siempre limitaciones de precisión). De este modo, sólo
podremos trazar segmentos con la regla ası́ como trazar circunferencias y transportar longitudes
(desconocidas en general) con el compás.

El primer ejemplo de construcción lo vimos al estudiar el postulado de medida de ángulos


en la sección 3.3. En esa ocasión y, a través de ese procedimiento constructivo adquirimos una
herramienta para copiar un ángulo dado sobre una semirrecta dada, sin necesidad de conocer la
amplitud del ángulo.

A continuación presentaremos algunas construcciones geométricas elementales y su justificación


teórica a partir de los teoremas demostrados con antelación.

Simetral de un trazo.

Sea dado el segmento AB:


AB
- Trace una circunferencia C1 con centro en A y radio mayor que .
2
- Con el mismo radio, trace una circunferencia C2 con centro en B.
- Las circunferencias C1 y C2 se cortan en los puntos P y Q.
- Trace la recta s que determinan los puntos P y Q y corta al segmento AB en M .

AB
Como el radio de ambas circunferencias es mayor que , éstas se cortan en P y Q. Note
2
que el cuadrilátero AQBP es un rombo, por lo que sus diagonales se dimidian en el punto
M y son perpendiculares, lo que significa que la recta s es perpendicular al segmento AB y
lo dimidia.

Note que esta construcción también sirve para encontrar el punto medio de un segmento
dado.

Perpendicular a una recta por un punto de la recta.

Sea ` una recta dada y sea P un punto sobre ella:

49
- Trace una circunferencia C1 con centro en P y radio arbitrario.
- Sean A y B los puntos de intersección de la circunferencia C1 con la recta `.
- Trace la simetral s del segmento AB.

El punto P , por ser centro de la circunferencia, es punto medio del diámetro AB, luego la
simetral de este segmento pasará por P y será perpendicular a él.

Perpendicular a una recta por un punto exterior a ella.

Sea ` una recta dada y sea P un punto exterior a ella:

- Trace una circunferencia C1 con centro en P y radio mayor que la distancia del punto P a
la recta.
- Sean A y B los puntos de intersección de la circunferencia C1 con la recta `.
- Trace la simetral s del segmento AB.

Los segmentos PA y PB, por ser radios de la circunferencia C1 son iguales, entonces el punto
P yace sobre la simetral s del segmento AB y ésta es perpendicular a `.

50
Paralela a una recta por un punto exterior a ella.

Sea ` una recta dada y sea P un punto exterior a ella. Hay varias construcciones posibles,
presentamos tres de ellas:

1. - Trace una recta cualquiera t que pase por P y corte a ` en un punto Q.


- Copie el ángulo α = ∠`QP sobre la recta t, con vértice en P y en el semiplano contrario al
que contiene a α.
- Trace la recta s que contiene al otro lado del ángulo copiado.

Como ∠`QP = ∠QP s, por el postulado de las paralelas, si se forman ángulos alternos internos
iguales, la recta s es paralela a la recta `.

2. - Trace la recta p que pasa por P y es perpendicular a `.


- Trace la recta s que pasa por P y es perpendicular a p.

Como ` ⊥ p y p ⊥ s, entonces: la recta s es paralela a la recta `.

3. - Tome un segmento AB sobre la recta `.


- Trace la circunferencia C1 de centro en P y radio AB.
- Trace la circunferencia C2 de centro en B y radio PA.
- C1 y C2 se cortan en Q. Trace la recta s que determinan P y Q.

51
Como AB = PQ y AP = BQ, el cuadrilátero ABQP es un paralelógramo y la recta s es
paralela a la recta `.

Bisectriz de un ángulo.

Sea α un ángulo de vértice A y lados `1 y `2 :

- Trace una circunferencia C1 con centro en A y radio arbitrario.


- Sean P y Q los puntos de intersección de la circunferencia C1 con las rectas `1 y `2 .
- Trace la simetral b del segmento PQ.

Los segmentos AP y AQ, por ser radios de la circunferencia C1 son iguales, entonces el
triángulo AP Q es isósceles y ya probamos anteriormente que la bisectriz del ángulo α coicide
con la simetral de la base del triángulo.

Triángulo equilátero de lado dado.

Sea AB un segmento dado:

52
- Trace una circunferencia C1 con centro en A y radio AB.
- Trace una circunferencia C2 con centro en B y radio AB.
- Las circunferencias C1 y C2 se cortan en los puntos P y Q.

Los segmentos AB, AP, BP, BQ y AQ, por ser radios de las circunferencias C1 y C2 , son
iguales; entonces los triángulos ABP y ABQ son equiláteros y sus ángulos interiores miden
60◦ .

2.5. El número π y el área de la circunferencia.


La circunferencia ha sido desde tiempos remotos objeto de admiración por la perfección y
simetrı́a de su forma. Al observar la Luna llena, el Sol en dı́as nublados y las trayectorias de las
estrellas durante la noche, todo parecı́a estar en perfecta armonı́a unido a través de esta “mági-
caçurva. Por lo anterior no es extraño que los primeros matemáticos se hallan interesado en estudiar
sus principales caracterı́sticas.

Entendemos como circunferencia al lugar geométrico de todos los puntos que equidistan de un
mismo punto llamado centro. La distancia constante es el radio de la circunferencia. Una cuerda
es un segmento que une dos puntos de la circunferencia. Si esta cuerda pasa por el centro recibe el
nombre de diámetro. Un ángulo que tiene su vértice en la circunferencia y cuyos lados son cuerdas
de ésta es un ángulo inscrito en la circunferencia.

53
Definimos la recta tangente a una circunferencia en un punto de ella como la perpendicular al
radio en el punto.

Lema 2.5.1. La recta tangente corta en un único punto a la circunferencia

Realizaremos la demostración por reducción al absurdo. Supongamos que la recta tangente a una
circunferencia en el punto P, corta a la circunferencia en otro punto Q. Por definición, la tangente
en P es perpendicular al radio en el punto de contacto, luego el ángulo OP Q es recto. Como el
triángulo OP Q es isósceles el ángulo OQP también es recto. Tendrı́amos entonces un triángulo
cuya suma de ángulos interiores es mayor que 180◦ . Esta contradicción prueba el teorema.

Existen muchas propiedades tanto angulares como métricas relacionadas con la circunferencia.
Algunas de estas propiedades están resumidas en los siguientes teoremas.

Teorema 2.5.0.24. Todo ángulo inscrito en una circunferencia mide la mitad del ángulo central que
subtiende el mismo arco.

Consideremos el ángulo inscrito ACB que subtiende el arco AB en la circunferencia de centro


O y radio r. En la demostración hay tres casos a considerar según el centro O de la circunferencia
esté en el interior del ángulo, en uno de los lados del ángulo o en el exterior del ángulo, casos que
se presentan en la siguiente figura.

54
Utilizando que los ángulos basales de un triángulo isósceles tienen igual medida, el teorema
resulta en todos los casos de sumar ángulos en triángulos.

Lo anterior prueba el siguiente resultado

Teorema 2.5.0.25. Dos ángulos inscritos en una circunferencia que subtienden el mismo arco tienen
igual medida.

Con un análisis similar al del teorema anterior, el lector puede demostrar los siguientes teoremas.

Teorema 2.5.0.26. Todo ángulo inscrito en una semicircunferencia es recto.

Teorema 2.5.0.27. El ángulo formado por una cuerda y la recta tangente en uno de los extremos
de ella, mide lo mismo que cualquier ángulo inscrito que subtienda esa cueda.

Consideremos el siguiente problema que está ı́ntimamente ligado con lo que hemos expuesto
anteriormente. Un futbolista acomoda la pelota para realizar un tiro libre. Observa el arco con un
ángulo α desde su posición ¿Desde qué otros puntos de la cancha, se observa el arco con la misma
amplitud α ?

55
Es claro que si el futbolista se aleja del arco en la dirección de la flecha, el ángulo con que lo
observa disminuirá. Si por el contrario se acerca en la dirección contraria, el ángulo aumentará.
Luego deberı́a moverse hacia el costado. Si el lector recuerda que los ángulos inscritos en una
circunferencia que subtienden el mismo arco miden lo mismo, se dará cuenta que lo único que tiene
que hacer es construir la circunferencia que pasa por los puntos A, B y P. Si coloca el balón en esa
circunferencia, seguirá observando el arco con el mismo ángulo α. Sin saberlo ha descubierto un
lugar geométrico muy interesante llamado arco capaz.

Definición 2.5.2. Dada una base fija AB y un ángulo α, El lugar geométrico de los vértices C de
todos los triángulos que tienen una base común AB y el ángulo opuesto de magnitud α es el arco
capaz de magnitud α del segmento AB.

Dado un segmento AB y un ángulo α describimos ahora la construcción del arco capaz de


magnitud α del segmento AB. Primero copiamos el ángulo α a partir del vértice A y hacia abajo
del segmento AB. El centro del arco se determina intersectando la simetral del trazo AB con la
perpendicular al lado del ángulo en el punto A como en la figura.

Ejercicios

1. Dados tres puntos A, B y C en un mismo plano, determinar un cuarto punto Q tal que unido
con los tres anteriores, las rectas QA, QB y QC formen dos ángulos dados α y β.

56
2. Dado un triángulo ABC hallar en su interior un punto Q tal que los ángulos ∠AQB, ∠BQC
y ∠CQA sean de igual medida. (Problema de Brocard)
3. Determinar el lugar geométrico de los puntos medios de los lados de un triángulo ABC cuya
base AB es fija y el ángulo γ opuesto a la base es constante.

También se producen algunas relaciones métricas importantes de mencionar.

Suponga que considera un punto P en el exterior de una circunferencia de centro O y radio r.


Trace desde P un rayo que corte a la circunferencia en los puntos A y B. Notará que el producto
PA · PB permanece constante independiente del rayo trazado. Intente verificar esto con Cabri.
Esto en realidad no es casualidad, este importante invariante recibe el nombre de “potencia del
punto P respecto a la circunferencia”.

Teorema 2.5.0.28. Si por un punto dado P se trazan secantes a una circunferencia dada, las
distancias desde P a cada uno de los puntos de corte de cada secante con la circunferencia tienen
productos iguales.

Para ver esto sea P un punto en el exterior a una circunferencia, tracemos una secante que corta
en los puntos A y B a la circunferencia. Sea T el punto en que la recta tangente desde P corta
a la circunferencia. Como los triángulos P T A y P BT tienen dos ángulos iguales, son semejantes,
entonces sus lados son proporcionales por lo que
PA PT
= .
PT PB
Lo anterior prueba que el producto PA · PB vale PT2 y no depende de la semirrecta trazada.
Asi, si trazamos otra secante que corta a la circunferencia en los puntos C y D, tenemos que

PA · PB = PC · PD = PT2 .

57
Puede probarse también que el producto P A · P B es constante para un punto P situado en el
interior de la circunferencia.

Teorema 2.5.0.29. Sea P un punto en el interior de una circunferencia, suponga que traza dos
secantes por P que cortan a la circunferencia en puntos A, B y C, D respectivamente, entonces
PA · PB = PC · PD.

La afirmación anterior se obtiene directamente de la semejanza de los triángulos P DA y P DC.

Pensemos ahora en el problema de asignar un área a una región circular. En este caso la forma
de la frontera es curva, lo que hace que el problema de asignar un área sea significativamente más
difı́cil.

El problema de las lúnulas de Hipócrates daba una esperanza de que era posible calcular áreas
de regiones curvas. Este resultado establece lo siguiente.

Sea ABC un triángulo rectángulo en A; las superficies comprendidas entre el semicı́rculo BAC
de diámetro BC y los semicı́rculos de diámetros AC y AB exteriores al triángulo, se llaman lúnulas
de Hipócrates. La suma de las áreas de estas dos lúnulas es igual al área del triángulo ABC.

58
El problema de construir un polı́gono cuya área sea igual a la de un cı́rculo dado, tuvo entonces
nuevas esperanzas dado que era posible, de acuerdo a lo anterior, hacer que la suma de áreas de
regiones de fontera curva con forma de luna, fuera igual al área de un simple triángulo rectángulo.

El primero en dar una luz para la solución del problema del cálculo del área de regiones de
frontera curva fue Arquı́medes quien utilizó un método llamado “Exhaución”para determinar el
área de algunas regiones. Este método consistı́a en inscribir polı́gonos de área, por supuesto cono-
cida, en el interior de la región y aumentar sus lados, de modo que el área de la región original
fuera aproximada sucesivamente por áreas de polı́gonos inscritos. Es decir la idea de lı́mite que
serı́a formalizada mucho más tarde por Leibnitz.

Consideremos dos circunferencias C1 y Cr , una de radio unitario y otra de radio r. Considere-


mos también dos polı́gonos regulares, Pn y P 0 n , con el mismo número de lados inscritos en ambas
circunferencias. Hemos mencionado anteriormente que la razón entre las áreas de estos polı́gonos
es el cuadrado de la razón entre elementos homólogos. Esto es

Área(P 0 n ) r2
= .
Área(Pn ) 1
Si hacemos crecer el número de lados, el área de cada polı́gono estará tan cerca del área del
cı́rculo como queramos, luego es de esperar que la misma relación anterior se mantenga para el
área de las circunferencias anteriores. Esto es

Área(Cr ) r2
=
Área(C1 ) 1
de donde deducimos que el área de una circunferencia es proporcional al cuadrado del radio, es
decir

59
Área(Cr ) = k · r2 ,
donde la constante k es el área del cı́rculo unitario, tardaron bastante tiempo en darse cuenta que
esta constante era en realidad un número irracional, esto significa que su representación decimal
es infinita no periódica. Este número fue bautizado por Euler como π.

Como el lector debe √ conocer, algunos números irracionales pueden ser construidos geométri-
camente, por ejemplo 5 se determina como la diagonal de un rectángulo de lados de una y dos
unidades respectivamente. La demostración de que era imposible construir un polı́gono con igual
área que un cı́rculo sólo se consiguió después de desarrollar una nueva teorı́a con mucha más
matemática de la que se estimaba inicialmente, lo que sepultó la esperanza griega de la cuadratura
del cı́rculo.

Los primeros mil decimales de π son

3.141592653589793238462643383279502884197169399375105820974944592307816406286
20899862803482534211706798214808651328230664709384460955058223172535940812848
11174502841027019385211055596446229489549303819644288109756659334461284756482
33786783165271201909145648566923460348610454326648213393607260249141273724587
00660631558817488152092096282925409171536436789259036001133053054882046652138
41469519415116094330572703657595919530921861173819326117931051185480744623799
62749567351885752724891227938183011949129833673362440656643086021394946395224
73719070217986094370277053921717629317675238467481846766940513200056812714526
35608277857713427577896091736371787214684409012249534301465495853710507922796
89258923542019956112129021960864034418159813629774771309960518707211349999998
37297804995105973173281609631859502445945534690830264252230825334468503526193
11881710100031378387528865875332083814206171776691473035982534904287554687311
59562863882353787593751957781857780532171226806613001927876611195909216420198

El poema que hemos puesto en los inicios de este texto tiene una estrecha relación con este
magnı́fico número. Desafiamos al lector a que encuentre dicha relación.

“Soy, y seré a todos definible


mi nombre tengo que daros
cociente diametral siempre inmedible
soy de los redondos aros.”

El análisis anteriormente expuesto prueba el siguiente teorema.

Teorema 2.5.0.30. El área de una circunferencia de radio r es πr2 .

La relación existente entre el área de un polı́gono regular y su perı́metro permite calcular el


perı́metro de la circunferencia conociendo su área.

60
Lema 2.5.3. El área de un polı́gono regular es igual al semiproducto de su perı́metro y apotema.

El apotema es el segmento que va desde el centro del polı́gono al punto medio de uno de sus
lados en la figura hemos designado su longitud por “a”. Si x1 , x2 , ..., xn denotan las longitudes
de los n lados del polı́gono, que por supuesto miden lo mismo, podemos calcular su área como la
suma de las áreas de los triángulos en que es descompuesto. Esto es,
a
Área = (x1 + x2 + ... + xn ) · .
2
Por lo anterior y suponiendo que al aumentar el número de lados, el perı́metro del polı́gono se
acerca al perı́metro del cı́rculo y su apotema a su radio r. Tenemos la interesante relación entre el
área A del cı́rculo y su perı́metro P .
r
A=P ·
2
Reemplazando la fórmula antes obtenida para el área tenemos el siguiente resultado.

Teorema 2.5.0.31. El perı́metro de una circunferencia de radio r es 2πr

2.6. División de un segmento. Razón áurea.


En la Geometrı́a el problema de la división de un todo en partes aparece repetidamente, ya sea
referido a longitudes, amplitudes, áreas o volúmenes. El lector reconocerá este tema si se detiene a
pensar en situaciones tan simples como la medición de distancias, el loteo de terrenos o el reparto
de una torta de cumpleaños.

Usualmente se aborda este problema a través de instrumentos de medición adecuados al tipo


de magnitud que se quiere dividir. Por ejemplo, si se quiere dividir un segmento de 3 metros de
longitud en tres partes iguales, se toma una lienza y se miden con ella segmentos consecutivos de 1
metro cada uno. El lector estará de acuerdo en que este procedimiento es extremadamente limitado,
porque basta que el segmento original mida 1 metro de longitud y nuestro instrumento se hace
inservible para esta tarea, pues su precisión es limitada para medir la longitud requerida (1/3 = 0.3
metros). No resulta muy difı́cil notar que, a pesar de que se cambie la lienza original por el más

61
sofisticado y preciso de los instrumentos disponibles, siempre habrá un lı́mite de exactitud en él.
Lo mismo ocurre con la medición de amplitudes de ángulos, podremos medir con un transportador
un ángulo recto, pero no podremos dividir con este instrumento (ni cualquier otro más preciso) un
ángulo recto en 7 partes iguales.

Estas limitaciones están relacionadas con la conmensurabilidad de ciertas magnitudes respecto


de la unidad de medida mı́nima del instrumento, es decir: si esta unidad “cabe.o no un número
entero de veces en el segmento a medir.

Aún ası́, es claro que hay métodos geométricos para construir (utilizando solamente regla y
compás) segmentos cuyas longitudes sean inconmensurables. Por ejemplo: si se construye un cuadra- √
do de lado 1 metro (magnitud que puede ser medida con una lienza común) su diagonal medirá 2
metros (magnitud inconmensurable con la lienza). Del mismo modo, si construimos
√ un triángulo
equilátero de lado 2 cm, entonces su altura será un segmento de longitud 3 cm. Inmediatamente
nos podrı́amos preguntar si se puede elaborar una construcción geométrica para obtener segmentos
de cualquier longitud. Lamentablemente, la respuesta a esta interrogante es negativa, no todos los
números reales son construibles geométricamente. Las magnitudes que pueden ser “construidas-
eciben el nombre de números algebraicos y entre ellas están los racionales y las raı́ces (de cualquier
orden). Existen números reales que no pueden ser obtenidos como longitud de ningún segmento a
través de una construcción geométrica que utilice solamente regla y compás, estos números reciben
el nombre de trascendentes y entre ellos están π, e y una cantidad infinita de otros números reales.
Si al lector le interesa profundizar en estas denominaciones y su relación con el álgebra puede
investigar sobre el tema en la bibliografı́a indicada en este libro.

De la misma manera, existen construcciones geométricas que permiten obtener ángulos de


ciertas magnitudes. Por ejemplo, en las construcciones elementales presentadas obtuvimos ángulos
de 60◦ y 90◦ , luego, copiando ángulos sucesivamente o construyendo bisectrices de ángulos dados,
podemos obtener ángulos de 30◦ , 45◦ , 75◦ , 15◦ , 105◦ , entre otros.

Si retornamos a una de las situaciones planteadas anteriormente, la de dividir un segmento de 1


metro de longitud en tres partes iguales, podremos profundizar en lo discutido. Presentaremos una
construcción geométrica particular que permite resolver este problema y veremos cómo, utilizando
los resultados demostrados en secciones anteriores, podemos justificar teóricamente lo obtenido a
través de la construcción.

División de un segmento en tres partes iguales.

Sea AB el segmento que se desea dividir. La idea es hacer que este segmento sea la transversal
de gravedad de un triángulo y el punto buscado su centro de gravedad. Para esto trazamos por
el extremo B una recta ` cualquiera y una circunferencia con centro en B y radio arbitrario, que
corta a ` en C y D. Sea M el punto medio del segmento AC. El segmento MD corta a AB en T .
Una circunferencia con centro en T y radio TB corta al segmento AB en S.

62
Discusión:

Es claro que DB = BC por ser radios de la circunferencia y que CM = MA por construcción.


Luego, en el triángulo ADC tenemos que DM y AB son transversales de gravedad, por lo que se
AB
cortan en razón 1 : 2. Eso indica que BT = . La posición del punto S queda ahora claramente
3
determinada.

Note que esta construcción puede hacerse sólo con regla y compás pues está basada en las
construcciones elementales que vimos en la sección anterior, además puede ser rigurosamente jus-
tificada a partir de los resultados con que contamos de geometrı́a plana. Como el procedimiento
es independiente de la longitud original del segmento AB, puede ser replicado para dividir en tres
partes iguales cualquier segmento.

Para extender lo que hemos visto describiremos y justificaremos una construcción geométrica
que permite dividir un segmento arbitrario en un número cualquiera n de partes iguales.

División de un segmento en n partes iguales.

Dado el segmento AB, se traza una semirrecta cualquiera con origen en A. Sobre la semirrec-
ta se construyen n segmentos iguales (basta con hacer un segmento con origen en A) y hacer
circunferencias iguales. Desde el extremo G del n-ésimo segmento, se traza el segmento GB. Basta
con hacer rectas paralelas a GB por los puntos intermedios C, D, E y F . Las intersecciones de
estas rectas con AB determinan la división del segmento.

63
Discusión:

La construcción puede hacerse sólo con regla y compás ya que está basada en las construcciones
elementales que vimos en la sección anterior.

La construcción se basa en el teorema de Tales aplicado al triángulo ABG. Como las transver-
sales son todas paralelas entre sı́, determinan sobre el lado AB segmentos en la misma proporción
de los que están en el lado AG. Como éstos son iguales, los del lado AB también son iguales.
Como hay n segmentos en total, cada uno de ellos es la n-ésima parte del segmento original.

División de un segmento en una razón dada.

Diremos que un punto P divide al segmento AB en una razón dada k > 0 si


AP
=k
PB
Si el punto P está entre A y B, decimos que divide interiormente al segmento, mientras que si
el punto está en la prolongación del segmento, decimos que lo divide exteriormente.

Note que en la división exterior, si k > 1 es porque B está entre A y P , mientras que si k < 1
es porque A está entre P y B.

Si P divide interiormente en razón 1 a AB, entonces es su punto medio, pues


AP
=1 ⇒ AP = PB.
PB
Un punto no puede dividir exteriormente a un segmento en razón 1.

Primero veamos el caso en que la razón de división es un número racional


AP m
= ∈ Q.
PB n
Describiremos una construcción a través de la que podremos encontrar los puntos P y P 0 que
dividen interior y exteriormente a un segmento AB en razón k = mn.

Sea dado el segmento AB. Tracemos por A y por B dos rectas paralelas `1 y `2 . Sobre `1
trace m segmentos de igual longitud u. Sobre la recta `2 trace n segmentos de igual longitud u y
trace la recta que pasa por los extremos de los últimos segmentos trazados sobre cada recta. Para
encontrar el punto P que divide interiormente a AB en razón m n , los segmentos trazados en `2
deben estar en el semiplano contrario a los trazados en `1 (respecto del segmento AB), mientras
que para encontrar el punto P 0 que divide exteriormente a AB en razón m n , los segmentos trazados
en `2 deben estar en el mismo semiplano que los trazados en `1 .

64
La justificación de esta construcción es inmediata del Teorema de Tales y el lector puede hacerla
por sı́ mismo.

Note que igualmente se puede utilizar la división de un segmento en partes iguales. Si se divide
el segmento AB en n+m partes, tomando P como el m-ésimo punto, se obtiene la división interior.
En cambio, si se divide el segmento AB en m − n partes y se toman en la prolongación por B
n partes adicionales, tomando como P 0 el extremo del último segmento, se obtiene la división
exterior.

Esta construcción funciona cuando la razón es un racional, pues equivale a√dividir el segmento
en un número entero de partes, pero ¿podrá dividirse un segmento en razón 2? ¿y en cualquier
otro número real?

Note que se puede reproducir la construcción siempre que sobre las rectas `1 y `2 se tracen
segmentos con la longitud adecuada. Si queremos que
AP
=r∈R
PB
basta trazar sobre `1 un segmento de longitud r y sobre `2 un segmento de longitud 1 en el semiplano
que corresponda para obtener los puntos P y P 0 de división interior y exterior del segmento.

65
Esto significa que todo dependerá de la posibilidad de construir con regla y compás un segmento
de longitud r, es decir: si r es un número algebraico se puede dividir el segmento AB interior y
exteriormente en razón r.

En tal caso podrı́amos preguntarnos ¿qué otros puntos del plano satisfacen la misma relación
que P y P 0 ? Es decir, queremos encontrar el lugar geométrico de los puntos P del plano para los
cuales
AP
=r
PB

Para responder la pregunta anterior, demostremos el siguiente resultado:

Teorema 2.6.0.32. En un triángulo, las bisectrices interior y exterior de un ángulo dividen al lado
opuesto interior y exteriormente en la misma razón de los otros dos lados.

En las figuras se aprecian las construcciones necesarias para hacer la demostración.

En el caso de la bisectriz interior, se traza BD paralela a la bisectriz, por suma de ángulos


se obtiene que el triángulo BCD es isósceles y se aplica el Teorema de Tales al triángulo ABD.
Luego:
AP m
= .
PB n
En el caso de la bisectriz exterior, se traza BD paralela al lado AC, por suma de ángulos
se obtiene que el triángulo BCD es isósceles y se aplica el Teorema de Tales al triángulo ACP 0 .
Luego:
AP0 m
0
= .
PB n

66
De este resultado se desprende que si un punto está en este lugar geométrico entonces, el ángulo
que subtiende con el segmento PP0 es recto. Es decir, por el Teorema 2.5.0.26, el lugar geométrico
buscado es una circunferencia de diámetro PP0 . Ésta recibe el nombre de circunferencia de Apolonio
del segmento AB con razón r y tiene propiedades muy interesantes que pueden ser estudiadas con
mayor detención por el lector en la bibliografı́a indicada.

Ejemplo

Calcule el radio R de la circunferencia de Apolonio de un segmento AB con razón r.

De la figura anterior tenemos las siguientes relaciones:


x x+y+z
=r =r x + y = AB
y z
y+z
y debemos encontrar el valor de R = .
2
x+y
De las dos primeras igualdades tenemos que x = ry, = r − 1, luego:
z
y r−1 y+z R r
= ⇒ = = .
z r+1 2z z r+1
AB AB
De las dos últimas igualdades tenemos: =r−1 ⇒ z= .
z r−1
r
Uniendo estas dos igualdades tenemos: R = AB.
r2 − 1

Notemos que a medida que la razón r se acerca a 1, el radio de la circunferencia de Apolonio


se hace cada vez más grande. Dejamos al lector que analice, utilizando Cabri, qué sucede en los
casos en que r es muy grande o muy cercano a cero.

67
Teoremas de Menelao y Ceva.

Como una aplicación importante de la división de un segmento en una razón dada se tienen
dos teoremas muy útiles al demostrar ciertas propiedades en la Geometrı́a y que presentamos a
continuación.

Teorema 2.6.0.33. Teorema de Menelao. Tres puntos X, Y , Z de los lados BC, CA y AB del
triángulo ABC son colineales, si y sólo si:
BX CY AZ
· · =1
CX AY BZ

Note que los puntos, en general, están sobre el lado del triángulo o en su prolongación y que
las proporciones involucradas en la expresión anterior son las razones en que los puntos X, Y, Z
dividen a los lados del triángulo.

Utilizando el Teorema de Tales repetidamente tenemos:


BX h2 CY h3 AZ h1
= , = , =
CX h3 AY h1 BZ h2
luego
BX CY AZ h2 h3 h1
· · = · · =1
CX AY BZ h3 h1 h2

El recı́proco también es cierto: si tres puntos X, Y , Z, cada uno de ellos sobre un lado del
triángulo (o su prolongación), cumplen la relación de Menelao, son colineales. En efecto, la recta
←→
XZ cortará al lado AC en un punto Y 0 . Por el teorema directo, sabemos que:
BX CY0 AZ
· · = 1.
CX AY0 BZ
Comparando esta igualdad con la que admitimos por hipótesis, tenemos que Y = Y 0 , y los
puntos X, Y , Z están en una misma recta.

68
Si el teorema de Menelao proporciona un criterio de colinealidad de tres puntos, el teorema de
Ceva proporciona un criterio de concurrencia de tres rectas.

El segmento que une un vértice de un triángulo con cualquier punto del lado opuesto se llama
ceviana en honor del matemático e ingeniero milanés Giovanni Ceva (1648-1734).

Teorema 2.6.0.34. Teorema de Ceva. Tres rectas que unen los vértices de un triángulo ABC con
tres puntos X, Y , Z situados en los lados opuestos son concurrentes si y sólo si:
BX CY AZ
· · =1
CX AY BZ

Para demostrar la necesidad, aplicamos el teorema de Menelao a los triángulos ABX y ACX
←→ ←→
(atravesado el primero por la recta ZC y el segundo por la recta BY ):
BC XP AZ AP BX CY
· · =1 · · =1
CX AP BZ XP BC AY

Multiplicando ambas expresiones tenemos el resultado que buscamos.

La demostración de la suficiencia del teorema de Ceva es idéntica a la del de Menelao. Sean tres
puntos X, Y , Z sobre los lados de un triángulo que obedecen a la relación de Ceva. Consideramos
←→ ←→ ←→ ←→
los puntos P (intersección de las rectas AX y ZC) e Y 0 (intersección de las rectas BP y AC). Por
el teorema directo, sabemos que:

BX CY0 AZ
· · =1
CX AY0 BZ
←→
Comparando esta igualdad con la hipótesis del teorema, vemos que Y = Y 0 y las rectas AX,
←→ ←→
BY y CZ son concurrentes.

Dejamos como un interesante ejercicio al lector que demuestre que las transversales de gravedad
y las bisectrices de los ángulos de un triángulo son concurrentes utilizando el Teorema de Ceva.

69
Razón Áurea. El número φ.

Dada cierta magnitud AB podemos dividirla interiormente de una infinidad de maneras difer-
entes. Entre todas estas divisiones ¿habrá una preferible a las otras desde el punto de vista de la
armonı́a de sus proporciones?

Para que un todo, dividido en dos partes desiguales parezca hermoso debe presentar entre la
parte menor y la mayor, la misma relación que entre ésta y el todo.

La expresión anterior puede considerarse como una medida geométrica de la Belleza. El lector
notará a través de los múltiples ejemplos que mencionamos cuáles son los fundamentos para esta
afirmación.

Si un punto P divide interiormente a un segmento AB de modo que:

AP AB
=
PB AP
la razón de proporcionalidad se denota por la letra φ y recibe el nombre de razón áurea, divina

proporción o número de Fidias.

Si observamos la figura podemos establecer una relación que nos permite calcular el valor
numérico de φ.

x x+y
En efecto, φ = = , luego: x2 − xy − y 2 = 0 por lo que, utilizando la fórmula general
y x
para las soluciones de la ecuación de segundo grado y tomando la raiz positiva (φ > 0) tenemos:
p √
y + y 2 + 4y 2 y(1 + 5)
x= =
2 2
luego √
x 1+ 5
φ= = .
y 2
Se verifica que este número φ es irracional, algebraico y sus primeros mil decimales son:

70
1.618033988749894848204586834365638117720309179805762862135448622705260462818
90244970720720418939113748475408807538689175212663386222353693179318006076672
63544333890865959395829056383226613199282902678806752087668925017116962070322
21043216269548626296313614438149758701220340805887954454749246185695364864449
24104432077134494704956584678850987433944221254487706647809158846074998871240
07652170575179788341662562494075890697040002812104276217711177780531531714101
17046665991466979873176135600670874807101317952368942752194843530567830022878
56997829778347845878228911097625003026961561700250464338243776486102838312683
30372429267526311653392473167111211588186385133162038400522216579128667529465
49068113171599343235973494985090409476213222981017261070596116456299098162905
55208524790352406020172799747175342777592778625619432082750513121815628551222
48093947123414517022373580577278616008688382952304592647878017889921990270776
90389532196819861514378031499741106926088674296226757560523172777520353613936

Los egipcios ya conocı́an esta proporción y la usaron en la arquitectura de la pirámide de Keops


(2600 años a.C.). Este número jugó un papel relevante en las artes y la estética de la antigua Grecia
y posteriormente en el Renacimiento. Fue considerado como la proporción perfecta entre los lados
de un rectángulo y se le atribuyó caracter “divino”por su repetida aparición en la Naturaleza.
Hay enigmáticas conexiones de la espiral de los nautilus (un tipo de caracola) y las espirales de
los girasoles con la razón áurea. También los cuerpos humanos exhiben proporciones cercanas a la
razón áurea, como puede verse comparando la altura total de una persona con la que hay hasta su
ombligo.

Fidias, Leonardo da Vinci, Dalı́, Bela Bartok entre otros artistas utilizaron esta proporción en
sus obras escultóricas, plásticas, arquitectónicas y musicales.

Antes hablamos de la posibilidad de dividir geométricamente un segmento en una cierta razón.


A continuación describiremos el procedimiento para dividir un segmento dado AB en razón áurea.

71
Se divide el segmento AB en dos partes iguales (M es su punto medio) y se transporta esta
longitud x, a través de C1 , sobre la perpendicular a AB que pasa por B, determinando el punto
C. Sobre el segmento AC, a partir del punto C se transporta la longitud x, por medio de C2 ,
determinando el punto Q y luego, la longitud AQ se transporta sobre el segmento AB a partir del
punto A, por medio de C3 , determinando el punto P .

Discusión:

Como el triángulo ABC es rectángulo en B, su √
hipotenusa AC mide 5 x. Al restarle x, el
segmento AQ y, por tanto el segmento AP, miden ( 5 − 1) x. Luego:
√ √ √
AP ( 5 − 1) x 5−1 1+ 5
= √ = √ = =φ
PB 2x − ( 5 − 1) x 3− 5 2

de la misma manera: √
AB 2x 2 1+ 5
= √ =√ = =φ
AP ( 5 − 1) x 5−1 2
Se llama rectángulo áureo a aquel cuyos lados están en la divina proporción. Ejemplos de
rectángulos áureos los podemos encontrar en las fachadas de numerosos edificios (como el Partenón
en Grecia o el edificio de la ONU en Nueva York), en las tarjetas de crédito, en nuestro carné de
identidad y también en las cajetillas de cigarro.

El lector podrá ahora comprobar cómo la construcción de la izquierda permite construir un


rectángulo áureo a partir de un cuadrado, sabiendo que M es el punto medio de un lado y que un
rectángulo áureo tiene la propiedad que se observa a la derecha.

Como ejercicios para el lector interesado dejamos las siguientes propiedades del número áureo:

1. φ = 1 + φ−1 , φn = φn−1 + φn−2 , ∀n ∈ N

1
2. φ = 1 +
1
1+
1
1+
1 + ···

72
r

q
3. φ = 1+ 1+ 1 + ···

4. φ = 2 cos 36◦ = 2 sen 54◦

Otra de las interesantes apariciones del número áureo ocurre al estudiar el pentágono regular.
El número φ es la relación entre varios segmentos que aparecen en el pentágono regular y en el
pentáculo construido dentro de éste. Estas relaciones y la posibilidad de reproducir estos patrones
infinitamente hacia dentro y hacia afuera de la figura hicieron que se le atribuyera propiedades
mágicas y fuera adoptado como sı́mbolo por grupos intelectuales y sectas.

e e c d b
Según la figura: = = = = = φ.
c d b b a
Con esta propiedad en mente se puede describir sin mayor dificultad la construcción geométrica
del pentágono regular ya que sabemos cómo encontrar dos segmentos cuyas longitudes estén en la
proporción áurea. Este es un ejercicio que dejamos al lector.

Una muestra de las razones por las cuales se le atribuye origen “divino.al número φ se tiene
cuando un rectángulo áureo se divide sucesivamente en cuadrados menores y en cada uno de ellos
se traza un cuarto de circunferencia de radio igual al lado del cuadrado. Juzgue por usted mismo
lo sorprendente del resultado que se obtiene.

73
Capı́tulo 3

Geometrı́a del espacio

En este capı́tulo presentamos los elementos fundamentales de la geometrı́a del espacio. Se


discuten los conceptos de vértices, aristas, caras y ángulos de poliedros. Se estudian las nociones
de área y volumen para los principales cuerpos geométricos.

3.1. Algunos postulados, definiciones y conceptos


Consideraremos como elementos básicos los puntos, las rectas y los planos y utilizaremos las
siguientes letras para referirnos a estos. Para los puntos: A, B, C, ..., P1 , P2 , ..., para las rectas:
l, l0 , l00 , ..., l1 , l2 , ..., y para los planos utilizaremos letras como Π, Π1 , Π2 , ...

Es un ejercicio interesante que el lector trate de imaginar y dibujar cada uno de los enunciados
que se exponen a continuación en el espacio.

1. Dos puntos distintos determinan una única recta.


2. Tres puntos son colineales, si existe una recta pasando por ellos.
3. Tres puntos distintos y no colineales, determinan un único plano.
4. Cuatro puntos se dicen coplanares si existe un plano pasando por ellos.
5. Si dos puntos distintos de una recta están en un plano, entonces toda la recta está contenida
en ese plano.
6. Si una recta tiene un solo punto en común con un plano, decimos que la recta corta al plano.
7. Si una recta no tiene puntos en común con un plano, decimos que la recta es paralela al
plano.
8. Una recta y un punto fuera de ella determinan un plano.
9. Dos rectas secantes distintas determinan un plano.
10. Dos rectas son paralelas si están en un mismo plano y no se cortan.

74
11. Dos rectas en el espacio que no están en un mismo plano y no se cortan se dicen alabeadas
(rectas que se cruzan).
12. Dos planos Π1 y Π2 tienen las siguientes posiciones relativas: Π1 es coincidente con Π2 , ambos
planos Π1 y Π2 son secantes (se cortan), Π1 es paralelo a Π2 .
13. Dos planos secantes se cortan en una recta.
14. Dados un plano y un punto fuera de él, existe un único plano que pasa por el punto y es
paralelo al plano dado.
15. El ángulo que forman dos rectas en el espacio, se define como el ángulo que forman las
paralelas a tales rectas trazadas desde un punto cualquiera del espacio.
16. Dos rectas se dicen ortogonales si el ángulo que forman es recto.

Desarrollaremos a continuación algunos problemas que ilustran cómo utilizar los conceptos
anteriores para generar nuevos resultados.

3.2. Problemas iniciales


Problema 1

Contruir una recta l que corte a una recta dada l1 , sea paralela a un plano dado Π y pase por
un punto dado A que no pertenece ni a l ni a Π.

Solución:

Hay tres posiciones posibles entre la recta l1 y el plano Π. l1 corta a Π en un solo punto, l1 es
paralela al plano Π o l1 pertenece a Π. Suponga en primer lugar que l1 corta a Π en el punto B
(ver la figura). Considere el plano Π1 que determinan el punto A y la recta l1 . Este plano Π1 corta
←→ ←→
a Π según una recta BC. Por A tracemos la paralela l2 a la recta BC, la que pertenece al plano
Π1 . La recta l2 pasa por A es paralela a Π y corta a l1 , por lo que es la recta pedida.

75
Dejamos al lector el análisis de los casos que no han sido considerados en la discusión anterior.

Problema 2

Trazar una recta que pase por un punto O dado y que corte a dos rectas dadas l1 y l2 . Discutir.

Solución:

En este problema también hay tres posibles posiciones relativas entre las rectas: que las rectas
se corten, que sean paralelas o que se crucen. Analizaremos el último caso y dejamos al lector los
restantes.

La recta l1 y el punto O determinan un plano. Supongamos que la recta l2 corta a ese plano
←→
en el punto Q. Consideremos la recta OQ. Esta recta es la recta pedida si corta a la recta L1 en
el punto R.

Discusión:

←→
Si la recta OQ es paralela a l1 entonces no hay solución. Esto es porque el plano que forma
O con l2 es paralelo a l1 .
Si l2 es paralela al plano que determinan O y l1 entonces no hay solución.
Si el punto O pertenece a alguna de las rectas l1 o l2 , la recta pedida se consigue uniendo
cualquier punto de la otra recta con O.

Definición 3.2.1. Una recta es perpendicular a un plano si y sólo si es perpendicular a todas las
rectas que pasan por el punto de intersección de la recta con el plano.

Note que de la definición anterior se obtiene fácilmente que si una recta es perpendicular a un
plano, entonces es perpendicular a todas las rectas del plano (incluso a las que no pasan por el
punto de intersección de la recta con el plano).

76
Teorema 3.2.0.35. Si una recta es secante a un plano y es ortogonal a dos rectas no paralelas del
plano, entonces es perpendicular al plano.

Demostración:

Considere la recta l, que corta al plano Π en el punto P y que es perpendicular a las dos rectas
l1 y l2 que pertenecen al plano Π y que pasan por P. Sea l3 una tercera recta cualquiera que pase
por P. Debemos demostrar que las rectas l y l3 son perpendiculares.

Considere los puntos D y E en la recta l pero a distintos lados del plano Π, que cumplan
DP ∼ = PE. Considere también una recta l4 cualquiera del plano Π que corte a las rectas l1 en A,
l2 en B y l3 en C.

Note que 4AP D ∼ = 4AEP y que 4BP D ∼ = 4BP E. Por lo anterior se tiene la siguiente
relación 4ABD ∼= 4ABE, de donde ∠CAE ∼ = ∠CAD. Luego se tiene que 4CAD ∼ = 4CAE, por
lo que CE ∼ = CD. Tenemos en consecuencia que 4P CD ∼ = 4P CE, de donde ∠CP E ∼ = ∠CP D
y como ambos ángulos son suplementarios, deben ser ambos rectos, de lo anterior las rectas l y l3
son perpendiculares.

Dejamos la demostración de los siguientes dos teoremas como ejercicio para el lector.

Teorema 3.2.0.36. Por un punto dado del espacio se puede trazar un sólo plano perpendicular a
una recta dada.

Teorema 3.2.0.37. Por un punto dado del espacio se puede trazar una sola recta perpendicular a
un plano dado.

77
Teorema 3.2.0.38. Teorema de las tres perpendiculares. Si una recta l es perpendicular a un plano
Π y desde un punto B de l se traza una perpendicular en C a una recta l1 del plano Π, entonces
la recta AC es perpendicular a l1 .

Demostración:

Considera el plano Π formado por la recta l y el punto C. La recta l1 es perpendicular a la recta


←→
CB. Note que l1 es perpendicular a l. Esto es porque si una recta es perpendicular a un plano,
es perpendicular a todas las rectas de ese plano. Por lo anterior la recta l1 es perpendicular a dos
rectas del plano Π, en cunsecuencia, por el Teorema 3.2.0.35, es perpendicular al plano Π. De lo
←→
anterior la recta AC es perpendicular a l1 .

Los siguientes ejercicios pondrán a prueba la imaginación y creatividad en 3D del lector.

Ejercicios

1. Dados un punto A y dos rectas que se cruzan l1 y l2 , pasar por A un plano Π paralelo a l1 y
a l2 .
2. Trazar una recta l que corte a tres rectas dadas l1 , l2 , l3 . ¿Cuántas rectas se pueden trazar ?
3. Demostrar que dos circunferencias de igual radio y del mismo centro no situadas en un mismo
plano tienen dos puntos en común.
4. Demostrar que 3 rectas que se cortan dos a dos y no situadas en un mismo plano son
concurrentes.

5. En el espacio se dan dos puntos O y O0 y dos rectas l y l0 . Trazar dos rectas paralelas entre
ellas l1 y l2 de modo que l1 pase por O y se apoye en l y la otra l2 pase por O0 y se apoye en
l0 . Discutir.
6. Sea P un punto variable de l. Encontrar el Lugar Geométrico de la recta intersección de los
planos (P, l1 ) y (P, l2 ) cuando P recorre la recta l. (l1 y l2 se cortan en O y l no las corta).
7. Un prisma recto tiene por base un hexágono regular de a cm de lado. Sus caras laterales son
cuadrados. Calcular la longitud de las tres diagonales que parten de uno de sus vértices.

78
8. Demostrar que si en un paralelepı́pedo una sección plana corta cuatro aristas laterales, dicha
sección es un paralelógramo.
9. Si por dos aristas opuestas de un cubo se hace pasar un plano. Determinar la naturaleza del
polı́gono de la sección y calcular el área si la arista del cubo mide a.
10. Demostrar que en un paralelepı́pedo las rectas que unen los puntos medios de dos aristas
paralelas situadas en distintas caras, se cortan en un mismo punto, que coincide con el punto
de concurrencia de las diagonales.
11. Si en los circuncentros de las caras de un tetraedro regular se levantan las perpendiculares
a los planos de las caras, demostrar que estas perpendiculares son concurrentes en un punto
que equidista de los vértices del tetraedro.
12. Demostrar que el plano determinado por los puntos medios de las aristas consecutivas de un
cubo y no situadas en un mismo plano corta al cubo según un hexágono regular.

3.3. Cuerpos en el espacio

Comenzamos esta sección haciendo un paralelo entre los poliedros, que son elementos de tres
dimensiones, con los polı́gonos, que son más familiares al lector y estan contenidos en un plano.

Los poliedros en 3D son los elementos que corresponden a lo que son los polı́gonos para la
dimensión dos. Pasamos ahora a definirlos.

Definición 3.3.1. Poliedro. Se denomina poliedro a la unión de varios polı́gonos planos adyacentes
y no coplanares, que limitan una región espacial finita(ver figura siguiente)

En el plano los polı́gonos están formados por elementos de dimensión uno (lados) y elementos
de dimensión cero (vértices.)

En un poliedro, existen elementos de dimensiones dos, uno y cero. Los polı́gonos que forman
las caras del poliedro son los elementos de dimensión dos y los lados y vértices, de dimensiones uno
y cero respectivamente, se conocen como aristas y vértices del poliedro.

79
Aquellos poliedros que verifiquen que cada plano que contiene a una de sus caras deja a todo el
resto del poliedro en el mismo semiespacio con respecto a ese plano, se llaman poliedros convexos.
Los que no cumplan esta propiedad para alguna de sus caras son poliedros no convexos.

Los análogos de los vértices de un polı́gono son por supuesto los vértices de un poliedro, sin
embargo si buscamos un análogo para los ángulos nos encontramos con alguna dificultad.

3.4. Ángulos en los poliedros


Distinguiremos dos tipos de ángulos en un poliedro. Los ángulos diedros que están asociados a
mirar el ángulo formado por dos planos con una arista comun, y los ángulos poliedros que están
asociados a cada vértice del poliedro.

Definición 3.4.1. Ángulo Diedro. Un ángulo diedro es la región del espacio comprendida entre la
←→
unión de dos semiplanos Π1 y Π2 de frontera común, en la figura la recta AB. Los semiplanos se
←→
llaman caras del ángulo diedro y la recta frontera AB se conoce como arista del ángulo diedro.

80
A cada ángulo de un polı́gono en el plano se le puede asociar un número que corresponde a su
medida angular, que tiene que ver con su amplitud. Esto es posible hacerlo para el ángulo diedro.

←→
Definición 3.4.2. Medida de un ángulo diedro. Dado un ángulo diedro de arista AB, y caras Π1
y Π2 sea O un punto de la arista. Si trazamos por el punto O y en cada semiplano la recta
perpendicular a la arista, tenemos un ángulo plano. La medida del ángulo diedro, se define como
la medida del ángulo plano ası́ construido.

Definición 3.4.3. Ángulo Poliedro. Dados un polı́gono A1 A2 ...An y un punto O fuera del plano
del polı́gono, trazamos por el punto O rayos que pasen por los puntos A1 , A2 , ..., An . La región
−−→ −−→
encerrada por los planos (O, A1 , A2 ), ..., (O, An , A1 ) se llama ángulo poliedro. Los rayos OA1 , OA2 ,
−−→
OAn se conocen como las aristas del ángulo poliedro. Los ángulos planos ∠A1 OA2 , ..., ∠An OA1 , se
denominan las caras del ángulo poliedro. Muchas veces se abusa de notación y se utiliza la palabra
caras para referirse a la medida del ángulo que la conforma.

Las dos propiedades siguientes se refieren a las caras de un ángulo poliedro.

Teorema 3.4.0.39. En todo ángulo poliedro cada cara es menor que la suma de las restantes.

Teorema 3.4.0.40. En todo ángulo poliedro convexo la suma de las caras es menor que 360◦ .

Este último teorema permite dar una demostración elemental del conocido hecho que sólo hay
cinco poliedros regulares, esto es, poliedros cuyas caras son polı́gonos regulares congruentes y cuyos
ángulos poliedros son todos iguales.

81
3.5. Prismas y cilindros
Definición 3.5.1. Superficie prismática. Dada una poligonal C que pertenece a un plano Π y una
recta g que corta al plano Π en un punto. Si por cada punto P de la poligonal C trazamos una recta
paralela a la recta g, originamos una superficie conocida como superficie prismática de generatriz
g y directriz la poligonal C.

Definición 3.5.2. Superficie cilı́ndrica. Si cambiamos la poligonal por una curva C que esté con-
tenida en el plano Π y repetimos el proceso realizado en la definición anterior, obtenemos una
superficie cilı́ndrica de generatriz g y directriz la curva C.

Algunas propiedades de las superficies prismáticas se resumen en los siguientes teoremas cuya
demostración dejamos al lector.

Teorema 3.5.0.41. Las aristas de una superficie prismática son rectas paralelas a la recta dada g.

Teorema 3.5.0.42. Las secciones de una superficie prismática paralelas al plano Π son polı́gonos
congruentes.

Definición 3.5.3. Prisma. El poliedro determinado por dos secciones paralelas al plano base de
una superficie prismática, con directriz un polı́gono cerrado, se llama Prisma. Las secciones para-
lelas se conocen como bases del prisma y los paralelógramos determinados por tales secciones en
la superficie prismática se llaman caras laterales del prisma. Si las caras son rectángulos, lo que
sucede cuando la generatriz es perpendicular al plano de la poligonal, el prisma se dice recto. La
distancia entre las bases del prisma se conoce como altura del prisma.

Definición 3.5.4. Cilindro. La superficie determinada por dos secciones paralelas al plano base de
una superficie cilı́ndrica con directriz una curva cerrada, se llama cilindro, las secciones paralelas
se llaman las bases del cilindro. Si la generatriz es perpendicular al plano de corte, el cilindro se
dice recto. Si la curva C es una circunferencia, el cilindro se dice circular. La distancia entre los
planos basales del cilindro se conoce como altura del cilindro.

82
Definición 3.5.5. Paralelepı́pedo. Un paralelepı́pedo es un prisma cuyas bases son paralelógramos.

Ejercicios

Probar las siguientes afirmaciones:

1. En un paralelepı́pedo dos caras opuestas cualesquiera pueden considerarse como bases.


2. Un paralelepı́pedo queda determinado si se conocen las tres aristas que pasan por un vértice
en dirección y longitud.
3. Las tres diagonales de un paralelepı́pedo concurren en un punto que las dimidia.
4. Las diagonales de un paralelepı́pedo recto tienen una longitud dada por la raı́z cuadrada de
la suma de los cuadrados de las longitudes de las tres aristas que concurren en un vértice.

3.6. Pirámides y conos


Definición 3.6.1. Pirámide. Una pirámide es el poliedro determinado por un ángulo poliedro y un
plano que corta todas las aristas de éste. El polı́gono de la sección es la base de la pirámide y los
triángulos son las caras laterales. La altura de la pirámide es la distancia del vértice al plano de la
base.Una pirámide se dice regular si la base es un polı́gono regular y la altura de la pirámide pasa
por el centro del polı́gono.

Definición 3.6.2. Tronco de pirámide. Un tronco de pirámide es un poliedro comprendido entre la


base y otra sección paralela a la base que corta a todas las aristas laterales de la pirámide.

83
Definición 3.6.3. Si consideramos una curva C en un plano Π y se unen todos los puntos P de la
curva C con un punto fijo O exterior a Π, obtenemos una superficie cónica. Si en vez de considerar
rectas, consideramos segmentos y pedimos que la curva C sea cerrada, obtenemos un cono, cuya
base es la región encerrada por C y altura la distancia entre el punto O y el plano basal Π. Si la
curva basal es una circunferencia y la altura cae en el centro de ella, el cono se dice circular recto.

El tronco de cono se define de manera análoga al tronco de pirámide.

Ejercicios

Probar las siguientes afirmaciones:

1. Todo plano paralelo a la base de una pirámide y que corta a las aristas laterales, determina
un polı́gono semejante a la base. Los perı́metros de tales polı́gonos están en la misma razón
que sus distancias al vértice y sus áreas en la razón del cuadrado de tales distancias.

84
2. Si dos pirámides de igual altura se cortan por planos paralelos a cada base y a la misma
distancia de los vértices, entonces la razón de las áreas de las secciones determinadas es
constante e igual a la razón entre las áreas basales. Si las bases de las pirámides tienen igual
área, las secciones también.
3. Todo poliedro puede ser descompuesto en pirámides.

3.7. Volumen y área de cuerpos en el espacio

Desde tiempos remotos el Hombre se ha encontrado con la necesidad de medir. Hasta ahora
hemos estudiado medidas asociadas a magnitudes lineales o planas, esto es lo que entendemos
como longitud de un segmento o área de una región. Asociamos ası́ a cada trazo AB un número
real que representa su longitud. Esto nos permite comparar distintos trazos. El perı́metro de un
polı́gono, que es la suma de las longitudes de sus lados, es una medida lineal. También asociamos
a un polı́gono en el plano otro número real (su área), de manera que sea una medida del lugar que
ocupa el polı́gono en el plano.

Como las caras de los poliedros son polı́gonos, cada una de estas tiene asociado un número real
que representa su área.

Definición 3.7.1. Área de un poliedro. El área de un poliedro es la suma de las áreas de sus caras.
A veces, según sea el caso, se distingue entre área lateral y área basal, en todo caso siempre la
suma de tales áreas es el área del poliedro asociado.

El concepto de volumen tiene relación con el lugar que ocupa un poliedro en el espacio.

Definición 3.7.2. Volumen de un poliedro. Asociamos a cada poliedro un número real, que tiene
relación con el lugar que ocupa el poliedro en el espacio. Como en el caso de las áreas, le pediremos
al volumen que cumpla los siguientes postulados.

Postulado 1: Poliedros adyacentes tienen volúmenes aditivos.

Postulado 2: Poliedros congruentes tienen igual volumen.

Postulado 3: Principio de Cavalieri. Si dos cuerpos comprendidos entre dos planos paralelos
tienen la propiedad de que cualquier plano paralelo a los planos anteriores, corta a los cuerpos en
dos regiones de igual área entonces los cuerpos tienen igual volumen.

Postulado 4: El volumen de un paralelepı́pedo rectangular de aristas de longitud a, b, y c es


V = abc.

En lo que sigue presentamos las fórmulas que permiten determinar las áreas y los volúmenes de
los cuerpos más conocidos. Será interesante para el lector entusiasta entender cómo se demuestran
estas fórmulas a partir de los postulados y utilizando procesos que pueden considerarse arraigados
en el fundamento del cálculo integral.

85
3.8. Volumen y área de prismas y cilindros
Teorema 3.8.0.43. Volumen de un prisma. El volumen de un prisma, es igual al área del polı́gono
basal por la altura.

Demostración:

En la figura anterior hay que probar que V = Área de (P)·h.

Consideremos un prisma cualquiera, con base el polı́gono P entre los planos paralelos Π1 y Π2 .
Consideremos en Π1 un rectángulo R que tenga la misma área que el polı́gono P. Contruyamos
el paralelepı́pedo rectangular con base R en el plano Π1 . Consideremos un plano cualquiera Π,
entre los planos Π1 y Π2 , este plano corta a ambos cuerpos en un polı́gono congruente a P y
un rectángulo congruente a R por lo que tienen igual área. Utilizando el principio de Cavalieri,
deducimos que ambos cuerpos tienen igual volumen. Como la altura del paralelepı́pedo es igual a
la del prisma, su volumen es el área basal por la altura, esto es V = Área de (R)·h, o lo que es lo
mismo, V = Área de (P)·h.

Teorema 3.8.0.44. Volumen de un paralelepı́pedo. El volumen de un paralelepı́pedo es igual al


producto del área de uno de sus paralelógramos basales por su altura correspondiente.

Demostración:

Es directa del teorema anterior aplicado a un prisma de base un paralelógramo.

Teorema 3.8.0.45. Volumen de un cilindro. El volumen de un cilindro es igual al producto de su


área basal por su altura.

Demostración:

86
Consideraremos regiones basales cuya área puede ser aproximada por áreas de polı́gonos. La
afirmación anterior tiene que ver con una pregunta fundamental: ¿qué tipo de regiones planas
admiten la asignación de un área ? Aunque el lector no vea en un inicio la razón de esta pregunta,
ella está en el fondo de la teorı́a que permite extender el concepto de área y desarrollada en los
fundamentos de la teorı́a matemática de la medida.

Supondremos entonces que el área de la región RC que encierra la curva C, verifica

Área(RC ) = lı́m Área(Pn ),


n→∞

donde los polı́gonos Pn están inscritos en la curva C y cada uno de ellos encierra una región
que está contenida en la que encierra el siguiente, de modo de aproximar cada vez mejor el área
encerrada por C.

El volumen del prisma inscrito en el cilindro asociado a cada polı́gono Pn , está dado por
VPn = Área(Pn ) · h, donde h es la altura del prisma. Haciendo crecer n, obtenemos el volumen del
cilindro VC ,
VC = lı́m VPn = lı́m Área(Pn ) · h = h · Área(RC ).
n→∞ n→∞

87
Teorema 3.8.0.46. Área de un prisma. El área lateral de un prisma puede calcularse como el
producto de la longitud de una arista lateral y el perı́metro de una sección recta. El área basal es
la suma de las áreas de los polı́gonos basales.

Demostración:

El área lateral de un prisma es igual a la suma de las áreas de los paralelógramos que forman sus
caras laterales. Estas caras son paralelógramos luego su área es base por altura. Si consideramos
en cada uno de ellos la arista como base, (dado que todas las aristas laterales miden lo mismo
digamos g). Si cortamos el prisma por un plano perpendicular a la arista, el segmento determinado
en cada paralelógramo es la altura del mismo. Sumando las áreas de los paralelógramos obtenemos
el resultado enunciado.

Teorema 3.8.0.47. Área de un cilindro. El área lateral de un cilindro puede calcularse como el
producto de la longitud del segmento generatriz lateral g y el perı́metro de una sección recta. El
área basal es la suma de las áreas de las regiones curvas basales.

Demostración:

Como en el caso del volumen del cilindro, el resultado se obtiene de aproximar el cilindro por
prismas.

¿Puede el lector completar los detalles ?

3.9. Volumen y área de pirámides y conos


Teorema 3.9.0.48. Volumen de una pirámide. El volumen de una pirámide es igual a un tercio del
producto de su área basal por su altura.

En la demostración utilizaremos el siguiente resultado.

88
Lema 3.9.1. Considere una pirámide de altura h con área basal A0 . Sea Ax el área de la sección
paralela a la base a distancia x del vértice entonces
Ax  x 2
=
A0 h
.

Demostración:

La demostración resulta de relacionar, utilizando semejanza, las áreas con lados homólogos de
los polı́gonos y éstos con las distancias al vértice de la pirámide, como se puede apreciar en la
figura anterior.

De esta forma  2
Ax bx  x 2
= = .
A0 b h
Probaremos el resultado para una pirámide de base triangular con una de sus aristas perpen-
dicular a la base.

La idea de la demostración es aproximar el volumen de la pirámide inferior y superiormente


por sumas de volúmenes de prismas, como se muestra en la figura.

89
Para ello utilizamos una técnica que resultará cada vez más familiar al lector y que se refiere al
fundamento del cálculo integral. Haremos una partición del lado perpendicular considerando n + 1
puntos equiespaciados, ubicados en la arista perpendicular a la base, partiendo en el vértice de la
pirámide:
x0 = 0, x1 = h/n, x2 = 2h/n, ..., xi = ih/n, xi+1 = (i + 1)h/n, ..., xn = h.
Consideremos el prisma que resulta de las secciones paralelas a la base en los puntos xi , xi+1 .
Llamemos vi al volumen del prisma interior y Vi al volumen del prisma exterior. Ai representa el
área del triángulo que resulta de cortar el prisma por un plano a distancia xi del vértice, como se
muestra en la figura.

De esta forma
h h
vi = Ai · , Vi = Ai+1 · .
n n
por el lema anterior
 x 2  2
i i
Ai = A0 = A0 .
h n

90
Note que si i = 0 entonces Ai = 0 , si i = n entonces Ai = A0 , el área basal como era de
esperar.

El volumen V de nuestra pirámide estará comprendido entre la suma de los volúmenes de los
prismas interiores y la de los prismas exteriores, esto es
n−1
X n−1
X
vi ≤ V ≤ Vi .
i=0 i=0
Reemplazando los valores de los volúmenes anteriores, tenemos que
n−1  2 n−1  2
X i h X i+1 h
A0 · ≤V ≤ A0 ,
i=0
n n i=0
n n
utilizando propiedades de las sumatorias,
n−1 n−1
h X 2 h X
A0 · i ≤ V ≤ A0 (i + 1)2 ,
n3 i=0 n3 i=0
Tenemos ası́ que

h(n − 1)n(2(n − 1) + 1) hn(n + 1)(2n + 1)


A0 · ≤ V ≤ A0 · ,
6n3 6n3
lo que es igual a

(1 − n1 )(2 − n1 ) (1 + n1 )(2 + n1 )
A0 · h ≤ V ≤ A0 · h .
6 6

Si hacemos crecer el número n de intervalos, es geométricamente claro que la suma de los


volúmenes de los prismas interiores aumentará y la de los prismas exteriores disminuirá. De he-
cho el término n1 en la expresión anterior puede hacerse arbitrariamente pequeño, escogiendo n
suficientemente grande luego

(1 − n1 )(2 − n1 ) h
A0 · h → A0 ·
6 3
cuando n → ∞, y también

(1 + n1 )(2 + n1 ) h
A0 · h → A0 · ,
6 3
cuando n → ∞, y como el volumen de la pirámide está atrapado entre estos dos valores, debe
cumplirse que
h
V = A0 · .
3
Esto es: el volumen de una pirámide es un tercio del área basal por la altura.

Note que de este resultado y del principio de Cavalieri se deduce la fórmula para cualquier
pirámide, porque si consideramos una pirámide cuya base es un polı́gono cualquiera basta conside-
rar una de base triangular, igual área basal y altura que la anterior , dado que por el lema el área

91
de la sección a distancia x del vértice depende del área basal y de x, por el principio de Cavalieri,
ambas pirámides tendrı́an igual volumen.

El resultado anterior puede extenderse para definir el volumen de un cono.

Teorema 3.9.0.49. Volumen de un cono. El volumen de un cono es igual a un tercio de su área


basal por su altura.

La demostración es similar a la del volumen de un cilindro, es decir un proceso de lı́mite de


volúmenes de pirámides cuyos polı́gonos basales están inscritos en la región basal del cono. ¿Puede
el lector completar los detalles?

Las demostraciones de los dos teoremas siguientes son sencillas y se dejan como ejercicio para
el lector.

Teorema 3.9.0.50. Área de una pirámide. El área lateral de una pirámide es igual a la suma de
las áreas de los triángulos que forman sus caras. En el caso de una pirámide regular, esto es su
base es un polı́gono regular y la altura pasa por su centro, el área lateral puede calcularse como el
producto entre el semiperı́metro del polı́gono basal s y el apotema lateral ρ, que corresponde a la
altura de los triángulos laterales.

A = ρs.

92
El área de un cono en general es difı́cil de calcular. Existe sin embargo una fórmula sencilla
cuando el cono es regular que, como en todos los casos anteriores, puede probarse por un proceso
de lı́mite de áreas de pirámides regulares inscritas.

Teorema 3.9.0.51. Área de un cono regular. El área lateral de un cono regular es igual al producto
del semiperı́metro basal y la generatriz g.

Ejercicios

1. El área lateral de un prisma recto de base pentagonal regular es 104 m2 . Calcular su altura
sabiendo que uno de los lados de la base es 2, 6 m.
2. La base de un prisma recto es un triángulo equilátero inscrito en una circunferencia de radio
3 m. Calcular la altura sabiendo que su área lateral es 135 m.
3. ¿Cuál es el área total de un prisma recto ortogonal regular , si la arista basal mide 4 m y su
altura 6 m ?
4. En una pirámide regular recta de base pentagonal, sus aristas laterales miden 9 cm. y la
arista basal 10 cm. ¿Cuál es el área total de la pirámide?
5. Calcular el área total de un tetraedro regular en función de su altura h.
6. Pruebe que el área lateral de un tronco de pirámide regular es igual al perı́metro de la sección
media paralela a las bases por el apotema lateral del tronco.
7. Pruebe que el volumen de un tronco de pirámide de bases paralelas es igual a un tercio del
producto de su altura, por la suma de sus dos áreas basales y de la media geométrica entre
ellas. Esto es:
1 p
V = h(B1 + B2 + B1 B2 ).
3
8. ¿Cuál es el área total de un tronco de pirámide cuadrangular regular cuyas aristas basales
miden respectivamente 4 m y 2 m y la altura 4 m?

9. Dividir la superficie lateral de una pirámide en dos partes equivalentes por medio de un plano
paralelo a la base.

93
10. Si las bases de un tronco de pirámide son B y b, calcular la superficie de la sección situada
a igual distancia de las bases.
11. Demuestre que todo segmento que tiene sus extremos sobre dos caras de un paralelepı́pedo
y que pasa por el punto de concurrencia de las diagonales queda dimidiado por dicho punto.
12. Probar que el área lateral de un tronco de cono regular de bases circulares de radios R y r
respectivamente y altura h está dada por
p
A = π(R + r) h2 + (R − r)2 .

13. Demuestre que el volumen de un tronco de cono de altura h cuyas bases circulares tienen por
radios r1 y r2 está dado por
1
V = πh(r12 + r22 + r1 r2 ).
3

3.10. Volumen y área de esferas y sectores


Teorema 3.10.0.52. Volumen de una esfera. El volumen de una esfera de radio R es V = 34 πR3 .

Utilizaremos la misma técnica aplicada para determinar la fórmula del volumen de la pirámide,
que es la idea más interesante sin duda de este capı́tulo.

Consideramos entonces una esfera de radio r. Tomamos un radio fijo cualquiera y hacemos una
partición de éste con n + 1 puntos equiespaciados, partiendo en el centro de la esfera:
R 2R iR (i + 1)R
x0 = 0, x1 = , x2 = , xi = , xi+1 = , ..., xn = R.
n n n n
Consideremos el cilindro que resulta de las secciones paralelas a la base en los puntos xi , xi+1 ,
como se muestra en la figura. Llamemos vi al volumen del cilindro interior, que se construye
utilizando la circunferencia de menor radio como base y Vi el volumen del cilindro exterior, en la
que se utiliza la circunferencia mayor.

94
Los volúmenes de estos cilindros son respectivamente

i2 R2 R
Vi = π(R2 − )
n2 n
(i + 1)2 R2 R
vi = π(R2 − )
n2 n
Luego si llamamos V al volumen de la esfera
n−1 n−1
X V X
vi ≤ ≤ Vi ,
i=0
2 i=0

lo que equivale a
n−1 n−1
X (i + 1)2 R2 R V X i2 R2 R
π(R2 − ) ≤ ≤ π(R 2
− ) ,
i=0
n2 n 2 i=0
n2 n
aplicando propiedades de las sumatorias, tenemos
n−1 n−1
R X 2 (i + 1)2 R2 V R X 2 i2 R2
π (R − ) ≤ ≤ π (R − 2 ),
n i=0 n2 2 n i=0 n
separando
n−1 n−1
R R2 X V R R2 X 2
π (nR2 − 2 (i + 1)2 ) ≤ ≤ π (nR2 − 2 i ),
n n i=0 2 n n i=0

R R2 V R R2
π (nR2 − 2 n(n + 1)(2n + 1)) ≤ ≤ π (nR2 − 2 (n − 1)n(2n − 1)),
n 6n 2 n 6n

R3 1 1 V R3 1 1
πR3 − π (1 + )(2 + ) ≤ ≤ πR3 − π (1 − )(2 − ),
6 n n 2 6 n n

Si aumentamos el número de puntos de la partición es claro geométricamente que la suma de


los volúmenes de los cilindros interiores aumentará y la de los exteriores disminuirá, de hecho como
1
n → 0 cuando n tiende a infinito,

R3 1 1 2
πR3 − π (1 + )(2 + ) → πR3 ,
6 n n 3
y también

R3 1 1 2
πR3 − π (1 − )(2 − ) → πR3 ,
6 n n 3
V
de esta forma como 2 está atrapado entre estas dos cantidades se concluye que

V 2
= πR3 ,
2 3

95
luego
4 3
V = πR
3
El lector puede modificar el procedimiento anterior para calcular el volumen de un casquete
esférico, que se define como cualquiera de los dos sólidos que resultan al cortar una esfera por un
plano que no pase por su centro.

Teorema 3.10.0.53. Volumen de un casquete esférico. El volumen de un casquete esférico de altura


h, en una esfera de radio R está dado por la fórmula

πh2 (3R − h)
V = .
3

Teorema 3.10.0.54. Área de una esfera. El área de una esfera de radio R está dada por

A = 4πR2 .

Para la demostración de esta propiedad utilizaremos un método que puede aplicarse a sólidos
simétricos. Este método está muy ligado al concepto de derivada de una función, aspecto que lo
hace más interesante.

Considere una esfera de radio R, inscriba en ésta una esfera que esté a una distancia h de
la anterior. Es claro que si h es pequeño las áreas de ambas esferas serán parecidas, ver figura
siguiente.

96
Considere ahora la cáscara entre las dos esferas cuyo volumen es
4
π(R3 − (R − h)3 )
3
después de pensarlo un rato, podemos aceptar que este volumen es aproximadamente el área de
alguna esfera entre ambas por h, luego al dividir por h la expresión inicial y hacer tender h a cero
nos estamos aproximando al área de la esfera.

4
π(R3 − (R − h)3 ) ≈ Ah · h
3
Lo anterior nos conduce a estudiar la expresión
4 (R3 − (R − h)3 )
π
3 h
cuando h se aproxima a cero.

Simplificando obtenemos

4(R2 + R(R − h) + (R − h)2 )


π.
3
Si h es cercano a cero, las expresión tiende a 4πR2 . que es el área de la esfera.

Teorema 3.10.0.55. Área de un casquete esférico.El área de un casquete esférico de altura h en una

esfera de radio R está dado por


A = 2πRh.

Problema

En un cono de revolución la sección plana que pasa por el vértice y su eje es un triángulo
equilátero. Calcular el área total en función de su altura h.

Solución:

97
Sea h la altura del cono, si x es el radio de la circunferencia basal, entonces el lado del triángulo
2h
equilátero es 2x. Utilizando el Teorema de Pitágoras x = √h3 y la generatriz es g = 2x = √ , luego
2 3
πh2

h
el área basal es π √ = . Por otra parte el área lateral de un cono recto es AL = πRg,
3 3
donde R es el radio de la circunferencia basal y g la generatriz. De lo anterior el área lateral es
2
AL = 2πx2 = πh2 . Luego el área total es
3
AT = πh2 .

Ejercicios

1. Un rectángulo de lados a y b gira en torno del lado a. Calcular el área total del cuerpo
generado.
2. Calcular la superficie total de un tronco de cono recto si su altura es 4.5 y el radio de la base
inferior es 9 y el de la superior es 3.
3. Determinar el volumen de un octaedro regular de arista a.
4. Demestre el teorema de Eudoxio: “Todo prisma triangular puede descomponerse en tres
pirámides de igual volumen”.
5. En un tronco de pirámide las áreas de la base inferior y superior son B y b respectivamente,
la altura de la pirámide entera h. Calcular el volumen del tronco.
6. Un cilindro y un cono están circunscritos a una esfera. Demostrar que el volumen del cilindro
es media proporcional geométrica entre los volúmenes de la esfera y del cono.
7. Probar que el área del casquete esférico es igual al área del cı́rculo que tiene por radio el trazo
que une el punto más alto M del casquete con un punto cualquiera P de su circunferencia
basal.
8. Calcular el volumen del cuerpo que se genera al hacer girar un cuadrado de lado a en torno
a:

98
a) una de sus diagonales.
b) la paralela a una de las diagonales que pasa por uno de los vértices del cuadrado.
c) la recta que une los puntos medios de dos lados contiguos.

9. Cortar una esfera por un plano de manera que la diferencia de las áreas de los casquetes
obtenidos sea equivalente al área de la sección plana.

10. Determinar el volumen comprendido entre dos esferas tangentes exteriormente y la superficie
cónica tangente a ellas, si sus radios miden 2 cm y 1 cm.
11. En una esfera de radio r se traza un plano a distancia 2r del centro. Determinar la razón en
que queda dividida el área de la esfera y en que razón queda dividido el volumen.

12. Demostrar que si se designan por C, B, A los volúmenes generados por un triángulo rectángulo
que rota sucesivamente en torno de la hipotenusa y de los catetos entonces se tiene:
1 1 1
2
= 2 + 2.
C A B

13. Dado un prisma triangular, encontrar el Lugar Geométrico de los puntos O tales que las
pirámides que tienen por vértice común O y por bases respectivas las caras laterales del
prisma tengan el mismo volumen.
14. Demuestre que el área de un sector esférico de una esfera de radio R, de altura h es A = 2πRh.
Note que esto indica que esta área no depende de dónde esté ubicado el sector en la esfera,
sólo depende de h.

99
Capı́tulo 4

Trigonometrı́a

En este capı́tulo se precisa la noción de ángulo y se introducen las razones y funciones tri-
gonométricas, estudiando sus principales propiedades. Se construyen los gráficos de las funciones
trigonométricas y de sus inversas, ası́ como se presentan las identidades y se estudia la resolución de
ecuaciones. Se aborda la aplicación de estas herramientas en la resolución de problemas geométricos.

4.1. Medición de ángulos.


Precisaremos las nociones de ángulos con las que hemos trabajado antes, reflexionando acerca
de lo que consideraremos “medida”de un ángulo y la forma de expresar este concepto.

En el plano la noción de ángulo puede describirse de varias formas. Consideraremos un ángulo


plano como aquella región que resulta de la intersección de dos semiplanos cerrados. La intersección
de las rectas que determinan los semiplanos se llama vértice del ángulo, y las semirrectas que
tienen origen en el vértice y están en ambos semiplanos, se denominan lados del ángulo. Note que
el ángulo es toda la región ası́ determinada y no solamente su frontera. También podrı́amos pensar
un ángulo desde el punto de vista dinámico como la región del plano que recorre una semirrecta
como resultado de un movimiento de rotación en torno a su origen.

En el espacio la noción es un poco más compleja, como vimos en el capı́tulo anterior. En las
definiciones 4.3.2 y 4.3.4 presentamos los ángulos diedros y poliedros que se generan al intersectar
dos planos en una recta o más de dos planos en un punto. Estos ángulos constituyen regiones
espaciales delimitadas por regiones planas y antes analizamos ciertas medidas asociadas con ellos.

100
En un sentido más general definiremos como ángulo sólido en el espacio a la región encerrada
por una semisuperficie cónica cuya directriz es una curva cerrada. Es decir, dado un punto V del
espacio (el vértice del ángulo sólido) y una curva cerrada plana D (la directriz del ángulo), si se
toman todas las semirrectas con origen en V y que pasan por algún punto de la curva D, queda
determinado dentro del semicono un ángulo sólido.

Note que en este caso si la curva directriz del ángulo es un polı́gono, el ángulo sólido es justa-
mente un ángulo poliedro como el que definimos anteriormente. Sin embargo, el ángulo diedro no
coincide con esta definición pues la directriz en ese caso corresponderı́a a un par de rectas paralelas,
que no forman una curva cerrada.

El problema de la medición de ángulos (planos o espaciales) es equivalente al problema de la


medición de longitudes, áreas o volúmenes que tratamos en distintos momentos anteriormente. Lo
que se busca es asociar con cada ángulo un número que exprese la magnitud de la región que
ocupa en el plano o en el espacio y que permita comparar estos objetos entre sı́. A la magnitud
de un ángulo se le denomina amplitud, analizaremos a continuación el problema de la medición de
amplitudes.

Cuando se mide cierto objeto geométrico una de las estrategias es compararlo con una cierta
unidad de medida predeterminada. Esta es la base de los sistemas sexagesimal y centesimal de
medición de ángulos. En ambos casos la unidad de medida se obtiene de la división de un ángulo
completo en una cierta cantidad de partes iguales.

En el sistema sexagesimal se fija como unidad de amplitud la 360-ava parte de un ángulo


completo, que recibe el nombre de grado sexagesimal y se denota: 1◦ . Para obtener subunidades
se procede de la siguiente manera:

1◦ = 600 (1 grado sexagesimal = 60 minutos sexagesimales)


10 = 6000 (1 minuto sexagesimal = 60 segundos sexagesimales)
100 = 10 dseg. (1 segundo sexagesimal = 10 décimas de segundo)
...

101
En el sistema centesimal, el ángulo completo se divide en 400 partes y cada una de ellas se
denota como 1 grad. Para obtener subunidades se procede de la siguiente manera:

1 grad = 100 min (1 grado centesimal = 100 minutos centesimales)


1 min = 100 seg (1 minuto centesimal = 100 segundos centesimales)
1 seg = 10 dseg. (1 segundo centesimal = 10 décimas de segundo)
...

Obviamente que la unidad de cada sistema puede obtenerse también a partir de un ángulo
extendido o de un ángulo recto (se divide un ángulo recto en 90 ó 100 partes iguales según sea el
caso). Además es claro cómo convertir unidades del sistema sexagesimal al centesimal o viceversa.

x◦ 360
= ⇔ x◦ = 0,9 y grad ⇔ y grad = 1.1 x◦
y grad 400

Luego, el proceso de medición de la amplitud de un ángulo dado (en estos y otros posibles
sistemas de medición) se reducirı́a a conmensurar la unidad de amplitud del sistema (y sus sub-
unidades) con el ángulo que se desea medir. En la figura, por ejemplo, se muestra un ángulo de
9◦ .

Describiremos a continuación otra manera de asociar un valor numérico a las amplitudes de


ángulos. Ésta consiste en asociar a cada ángulo plano la longitud de un arco de circunferencia.

En efecto, si se construyen circunferencias centradas en el vértice del ángulo y de radios distintos,


el ángulo determina arcos sobre cada una de ellas. La razón entre la longitud de los arcos y el radio
de las circunferencias correspondientes permanece constante, por lo que esta magnitud no depende
del radio, sino sólo del ángulo. Luego si tomamos, particularmente, la circunferencia de radio 1
podemos asociar con la amplitud θ del ángulo, la longitud del arco que este ángulo determina sobre
ella.

102
De ese modo se define un radián (1 rad) como la amplitud de un ángulo central de una circun-
ferencia de radio r, que determina un arco de longitud igual a r. De este modo el ángulo completo
tendrá una amplitud de 2π radianes, mientras que el ángulo recto medirı́a π2 . Note que los radianes
son números reales que, por ser cuociente de dos longitudes, no se expresan en unidades.

Ahora se podrá medir la amplitud de un ángulo en cualquiera de los tres sistemas de medición
estudiados y hacer las conversiones necesarias entre ellos.
2π 360 400 π 180 200
= ◦ = ó = ◦ =
x y z grad x y z grad

En efecto, 1 rad ≈ 57, 29578◦ ≈ 57◦ 170 4500 .

Utilizando la misma idea anterior se pueden medir ángulos sólidos en el espacio. En este caso
asociaremos la amplitud de un ángulo sólido con el área de una superficie esférica.

En efecto, si se construyen esferas centradas en el vértice del ángulo y de radios distintos, el


ángulo determina regiones cerradas sobre cada una de ellas. La razón entre el área de dichas regiones
esféricas y el cuadrado del radio de las esferas correspondientes permanece constante, por lo que
esta magnitud no depende del radio, sino sólo del ángulo. Luego si tomamos, particularmente, la
esfera de radio 1 podemos asociar con la amplitud θ del ángulo sólido, el área de la región que este
ángulo determina sobre ella.

De ese modo se define un estereorradián (1 srad) como la amplitud de un ángulo sólido que,
teniendo su vértice en el centro de una esfera de radio r, determina un área en la superficie de ésta

103
igual a r2 . De este modo, por ejemplo, el ángulo triedro que se muestra en la figura medirı́a π2 srad,
mientras que el ángulo sólido que determina una semiesfera tendrá una amplitud de 2π srad. Note
que los estereorradianes son también números reales que no se expresan en unidades.

Aquı́ debemos hacer notar que no contamos con todas las herramientas necesarias para calcular
áreas de regiones esféricas generales, pues para este fin se requieren resultados del cálculo diferencial
e integral que se estudiarán más adelante. Sin embargo, en vista de los resultados desarrollados en
el capı́tulo anterior podremos restringirnos a considerar ciertos ángulos sólidos poliedros (como el
de la figura) o aquellos generados por conos circulares rectos ya que las regiones esféricas que estos
últimos determinan son casquetes esféricos, para los cuales conocemos la manera de determinar su
área.

Esta forma de medición de ángulos planos (o sólidos) consiste en establecer una correspondencia
entre las amplitudes de ángulos y las longitudes de arcos de circunferencias (o áreas de superficies
esféricas).

Ejercicios

1. Exprese cada uno de los siguientes ángulos como fracción de un ángulo recto, en medida
sexagesimal, centesimal y en radianes, según corresponda.
a) 67◦ 300 b) 6 grad 25 min c) 0,2315 de un ángulo recto d) 5π
27 rad e) 0,3927 rad
2. El número de grados sexagesimales de un ángulo sumado al número de sus grados centesimales
es 152: ¿cuál es el ángulo?

3. Si s y t indican respectivamente el número de segundos sexagesimales y centesimales con-


tenidos en un ángulo, probar que
250s = 81t

4. Tomando π1 ≈ 0,31831 verifique que un radián tiene aproximadamente 206265 segundos


sexagesimales.
5. ¿Cuál es el radio del cı́rculo en el cual un arco de 1 cm subtiende un ángulo central de 1’ ?
6. Encuentre la medida en radianes de cada ángulo de un polı́gono regular.

104
7. Demostrar que la medida en radianes de cualquier ángulo central en una circunferencia se
expresa por la fracción
arco subtendido
radio
8. En una carrera a velocidad uniforme sobre una pista circular, un atleta recorre cada minuto
un arco que subtiende un ángulo en el centro de la pista de 20/7 radianes. Si cada vuelta de
la pista es de 400 m ¿cuánto tiempo tardará en recorrer 1 Km?
9. Hallar el radio de un globo esférico tal que la distancia medida sobre la superficie, entre dos
puntos cuyas latitudes difieran en 1◦ 100 , sea de 1 cm.
10. El volante de una máquina da 35 revoluciones por minuto, ¿qué tiempo tardará en girar 5
radianes?

11. El minutero de un reloj tiene 70 cm de largo, ¿cuántos centı́metros recorre su extremo en 20


minutos? ¿cuántos recorre su punto medio?
12. Suponiendo que la Tierra es una esfera de radio 12700 Km, encuentre la longitud de un arco
que subtienda 1 minuto en el centro de la Tierra.
13. Encuentre el √
ángulo de elevación del Sol cuando una torre de 18 m de altura proyecta una
sombra de 18 3 m de largo.
14. ¿Cuál es la medida en estereorradianes de un ángulo sólido completo?
15. Si un cono circular recto tiene ángulo en el vértice de θ radianes, ¿que amplitud en estereo-
rradianes tiene el ángulo sólido que este determina?

4.2. Razones trigonométricas en el triángulo rectángulo.

Consideremos dos triángulos rectángulos generales, semejantes entre sı́, cuyos elementos se
nombran como se muestra en la figura.

105
Según resultados anteriores sabemos que: 0 < α, β < π2 = 90◦ , α + β = π2 y que, por el
Teorema de Pitágoras, a2 + b2 = c2 y m2 + n2 = p2 . La semejanza de ambos triángulos asegura
que:
a b c
= =
m n p
por lo que podemos establecer diferentes relaciones entre elementos de un mismo triángulo que se
mantienen constantes por transformaciones de semejanza. Vemos que:
a m b n a m
= , = , =
c p c p b n
entre otras.

Note que, a pesar de que las longitudes de los lados de los triángulos varı́an, las razones an-
teriores permanecen constantes. Esto hace que tales razones sean asociadas con los ángulos del
triángulo, que son los que no varı́an por transformaciones de semejanza. De este modo, para un
ángulo α dado, podrı́amos construir a partir de él triángulos rectángulos (serı́an todos semejantes
entre sı́) y las razones que mencionamos serı́an iguales en todos ellos.

A estas razones se les denomina razones trigonométricas en el triángulo rectángulo y se detallan


a continuación:
cateto opuesto a m
seno α = sen α = = =
hipotenusa c p

cateto adyacente b n
coseno α = cos α = = =
hipotenusa c p

cateto opuesto a m
tangente α = tan α = = =
cateto adyacente b n

hipotenusa c p
cosecante α = csc α = = =
cateto opuesto a m

hipotenusa c p
secante α = sec α = = =
cateto adyacente b n

cateto adyacente b n
cotangente α = cot α = = =
cateto opuesto a m

106
Identidades Fundamentales. Razones y Corrazones Trigonométricas.

El lector no debe sentirse impresionado por la variedad de las anteriores definiciones pues,
como es usual en matemáticas hay varias relaciones que facilitan el manejo y la aplicación de estas
razones. De hecho mostraremos que bastará conocer una cualquiera de las razones trigonométricas
de un ángulo de un triángulo rectángulo para obtener cualquiera de las cinco restantes.

Por ejemplo, de las propias definiciones podemos ver que, para un ángulo α
sen α 1 cos α 1 1
tan α = , cot α = = , sec α = , csc α =
cos α tan α sen α cos α sen α

por lo que, conociendo el seno y el coseno del ángulo, se tienen el resto de las razones trigonomé-
tricas. Más aún, el conocido Teorema de Pitágoras permite obtener fácilmente, a partir de las
definiciones, las denominadas Identidades Fundamentales, que en sus distintas representaciones se
muestran a continuación y cuya demostración dejamos al lector como ejercicio.

sen2 α + cos2 α = 1, sec2 α − tan2 α = 1, csc2 α − cot2 α = 1


La combinación de las anteriores relaciones permite escribir cualquiera de las razones trigono-
métricas de un ángulo agudo en términos de cualquier otra, como se ve en la siguiente tabla y el
lector podrá verificar sin mayor dificultad.

sen α cos α tan α csc α sec α cot α


√ tan α 1 sec2 α − 1 1
sen α sen α 1 − cos2 α √ √
1 + tan2 α csc α sec α 1 + cot2 α


√ 1 csc2 α − 1 1 cot α
cos α 1 − sen2 α cos α √ √
1 + tan2 α csc α sec α 1 + cot2 α

sen α cos α 1 √ 1
tan α √ √ tan α √ sec2 α − 1
1 − sen2 α 1 − cos2 α csc2 α − 1 cot α


1 1 1 + tan2 α sec α √
csc α √ csc α √ 1 + cot2 α
sen α 1 − cos2 α tan α sec2 α − 1


1 1 √ csc α 1 + cot2 α
sec α √ 1 + tan2 α √ sec α
1 − sen2 α cos α csc2 α − 1 cot α


1 − sen2 α cos α 1 √ 1
cot α √ csc2 α − 1 √ cot α
sen α 1 − cos2 α tan α sec2 α − 1

107
Hasta ahora hemos utilizado sólo uno de los ángulos agudos del triángulo rectángulo (α) para
definir sus razones trigonométricas, pero el lector notará que el proceso es el mismo para su ángulo
complementario β. De hecho, este es el motivo por el cual las razones trigonométricas se agrupan
por pares, agregando el prefijo “co.al nombre: seno y coseno; tangente y cotangente; secante y
cosecante. Estos pares se llaman corrazones trigonométricas debido a que una razón trigonométrica
de un ángulo es igual a la correspondiente corrazón trigonométrica de su ángulo complementario,
es decir: π 
sen α = cos β = cos −α ,
2
π 
tan α = cot β = cot −α ,
2
π 
sec α = csc β = csc −α
2

y viceversa. Note que todas las definiciones anteriores sólo son válidas para ángulos de un triángulo
rectángulo (por tanto, agudos), por lo que no pueden establecerse estas relaciones en triángulos
generales. Esto no deberı́a ser un impedimento, ya que vimos que, dado cualquier ángulo agudo
se pueden construir triángulos rectángulos sobre él y definir con las longitudes de sus lados las
razones trigonométricas que estudiamos.

Ángulos Notables.

El lector comenzará a notar que en lo adelante, expresaremos las amplitudes de ángulos en


radianes y no en grados sexagesimales como se hace en la enseñanza media. Las razones para la
elección de este sistema de medición son muy importantes y están relacionadas con la esencia del
tratamiento funcional que daremos a estas razones y por tanto, con las herramientas del cálculo
diferencial e integral. Más adelante retomaremos esta discusión; de momento haremos el ejercicio de
calcular el valor de las razones trigonométricas de ciertos ángulos que llamaremos “notables”dada
su recurrente aparición en la geometrı́a plana. Nos referimos, por ejemplo, a los ángulos de 30◦ , 45◦
y 60◦ ó, como los mediremos desde ahora, de π6 , π4 y π3 radianes. Sugerimos al lector que verifique
las reglas de conversión presentadas anteriormente con estos y otros ángulos para que se familiarice
con el sistema radial de medición de amplitudes.

Para calcular las razones trigonométricas de estos ángulos tenemos que encontrar triángulos
rectángulos de dimensiones conocidas cuyos ángulos agudos sean los que queremos estudiar. ¿Re-
conoce usted algún triángulo rectángulo con ángulo interior de π6 radianes y lados de longitud
conocida?

Es claro que, si uno de los ángulos agudos de este triángulo mide π6 el otro tiene amplitud de
π
3, por lo que podrı́a obtenerse a partir de un triángulo equilátero de lado conocido. En efecto,
si trazamos la altura de un triángulo equilátero de lado a, aparece el triángulo rectángulo que
necesitamos y podemos conocer la longitud de todos sus lados, con lo que estarı́amos preparados
para calcular las razones trigonométricas de sus ángulos agudos.

108
Obviamente hemos utilizado resultados geométricos conocidos como el hecho de que la altura
de un triángulo equilátero es transversal de gravedad y el Teorema de Pitágoras, lo que nos permite
aplicar las definiciones estudiadas y calcular las razones que buscamos.
a

3a

π 1 π 3
sen = 2 = , cos = 2
=
6 a 2 6 a 2
√ √
π a
3 π 3a √
tan = √2 = , cot = 2
a = 3
6 3a 3 6 2
2

π a π a 2 3
csc = a = 2, sec = √ =
6 2 6 3a 3
2

Un detalle interesante es que este mismo triángulo conocido nos permite calcular las razones
trigonométricas del ángulo de π3 dado que éste es complementario del anterior.

π 1 π 3
cos = , sen =
3 2 3 2

π 3 π √
cot = , tan = 3
3 3 3

π π 2 3
sec = 2, csc =
3 3 3

Para el ángulo de π4 , ¿cómo encontrar el triángulo rectángulo que necesitamos? Claramente, si


trazamos la diagonal de un cuadrado de lado conocido podemos obtener el siguiente triángulo:

109
π
y calcular las razones trigonométricas del ángulo 4.

π a 2 π
sen =√ = = cos
4 2a 2 4
π a π
tan = = 1 = cot
4 a 4

π 2a √ π
csc = = 2 = sec
4 a 4

Ejercicios.

1. Construya ángulos con cada uno de los siguientes datos y calcule el seno, el coseno y la
tangente, en caso que no se den como datos:
a) sen θ = 0,39 b) tan θ = 0,7 c) cos θ = 0,9 d) sec θ = 2,8 e) cot θ = 0,625
2. Construir un triángulo ABC, rectángulo en C, de hipotenusa 10 cm y tan A = 0,81. Hallar
sen A y cos A.
3. Construir un triángulo rectángulo ABC, a partir de los siguientes datos:

tan A = 0,7 , ∠C = 90◦ , b = 2,8cm

4. Sea ABC es un triángulo rectángulo en A. Se traza BD perpendicular a BC, que corta a la


prolongación de CA en D. Si AB = 12, AC = 16 y BC = 20, encontrar BD y CD.
5. Si ABCD es un cuadrado y C está unido a E, que es punto medio de AD, encontrar todas
las razones trigonométricas del ángulo ∠ECD.
6. Verifique las siguientes relaciones:

a) (sen θ + csc θ)2 + (cos θ + sec θ)2 = tan2 θ + cot2 θ + 7


√ √ √
b) 1 + cot2 θ · sec2 θ − 1 · 1 − sen2 θ = 1
1 1
c) + = 2 sec2 α
1 − sen α 1 + sen α
d ) tan2 α + sec2 β = sec2 α + tan2 β
tan α − cot β cot β
e) =−
cot α − tan β cot α
7. Si sen θ − cos θ = 0, hallar csc θ.
p p cos θ − q sen θ
8. Si cot θ = , hallar el valor de .
q p cos θ + q sen θ
9. Hallar el valor de α si:

a) cos 2α = sen 3α
b) tan α = cot 3α

110
c) sec 5α = csc α

10. Dado el pentágono regular de la figura:

a) Calcule la amplitud (en radianes) de los ángulos interiores del triángulo rectángulo
AM D.
b) Calcule la longitud de los lados del triángulo.
c) Calcule las razones trigonométricas de los ángulos agudos del triángulo.

Sugerencia: Recuerde que si la longitud del lado del pentágono regular es a, su diagonal tiene
longitud φa, donde φ es el número áureo.

4.3. Funciones trigonométricas.

En esta sección explicaremos cómo superar la principal limitación que hemos encontrado hasta
ahora al momento de definir y calcular las razones trigonométricas de ángulos planos: el hecho de
que sólo es posible hacerlo para ángulos agudos de un triángulo rectángulo. La extensión de estas
nociones para ángulos generales implica a su vez, la extensión del concepto de razón trigonométrica
(como cuociente de longitudes de lados de un triángulo) al concepto de función trigonométrica.
Este ejercicio nos permitirá utilizar para el estudio de objetos geométricos algunas herramientas
de la teorı́a de funciones de variable real y del cálculo diferencial e integral, lo que nos permite
enriquecer nuestro punto de vista matemático y el tratamiento de ciertos problemas. En este sentido
deberemos adaptar y generalizar las nociones geométricas de ángulos y amplitudes de modo que
sean compatibles con la teorı́a de funciones de variable real.

Para esto consideraremos conocido el uso del sistema cartesiano para representar puntos del
plano, figuras geométricas y funciones reales. Comenzaremos generalizando la noción de ángulo
plano y su amplitud y discutiremos lo adecuado del sistema radial de medición de amplitudes en
este contexto.

Consideremos un sistema de coordenadas cartesianas rectangulares − →


x O−
→y y una circunferen-
cia de radio 1, centrada en el origen O que se denomina circunferencia goniométrica. Tomaremos
el semieje positivo O−→
x como lado inicial de los ángulos que consideraremos. Llamaremos θ a la
amplitud del ángulo ∠AOP .

111
−−→
Si el punto P se mueve sobre la circunferencia, el rayo OP rota alrededor del punto O y la
amplitud θ del ángulo ∠AOP varı́a. Note que, como la circunferencia tiene radio 1, la amplitud
θ del ángulo, medida en radianes, coincide con la longitud del arco de circunferencia AP . Hasta
ahora hemos pensado sólo en ángulos con amplitud positiva, pero es fácil definir ángulos negativos
si tenemos en cuenta el sentido en que tomamos el ángulo. Si el punto P rota desde A en sentido
contrario a las manecillas del reloj (antihorario) se considera el ángulo positivo. Si el punto P rota
desde A en el sentido de las manecillas del reloj (horario) se considera el ángulo negativo.

Note que, según la definición geométrica que tenı́amos antes, ambos ángulos, al ser semejantes,
medı́an lo mismo, pero que ahora los hemos diferenciado según el sentido en que se rota desde el
lado inicial hasta el lado final.

Aún más: podemos definir ángulos de amplitud mayor a la de un ángulo completo siempre
que consideremos la rotación necesaria para llegar hasta el lado final del ángulo. Por ejemplo: un
ángulo de 3π radianes serı́a aquel que consiste en una vuelta y media en torno al punto O en
sentido antihorario, un ángulo de 9π
2 consistirá en dos vueltas en torno a O más un cuarto de
vuelta adicional, todo en sentido antihorario, mientras que un ángulo de − 5π 2 se obtiene rotando
en sentido horario una vuelta completa y un cuarto de vuelta adicional.

112
Esta manera de considerar los ángulos: medidos en radianes, positivos o negativos según el
sentido de rotación y de cualquier magnitud, permite establecer una correspondencia entre los
ángulos con vértice en O, los puntos de la circunferencia goniométrica y los números reales. Es
decir, todo ángulo está representado por un punto de la circunferencia goniométrica que se obtiene
de la intersección del lado final del ángulo con ésta y a su vez, corresponde a un número real que
indica su amplitud medida en radianes.

Puntos de la
Ángulos de vértice O ←→ Circunferencia ←→ Números Reales
Goniométrica

Una forma gráfica de representar esta correspondencia es considerar una recta, perpendicular
al eje O−→
x y que pase por el punto A. Si se “enrosca.esta recta alrededor de la circunferencia
goniométrica, manteniendo fijo el punto A, entonces el punto P que el ángulo determina sobre la
circunferencia, se corresponde con un punto Q de la recta. Si se representan los números reales sobre
esta recta, colocando el cero en el punto A y tomando hacia arriba el sentido positivo, entonces el
punto Q indica la longitud del arco AP , es decir, la medida en radianes del ángulo ∠AOP .

113
Hay que notar que la correspondencia entre los ángulos y los números reales es biunı́voca,
es decir: a cada ángulo le corresponde uno y sólo un número real dado por la longitud del arco
de circunferencia que recorre el punto P desde A hasta su posición final (incluyendo las vueltas
necesarias si el ángulo es mayor que 2π o menor que −2π).

Sin embargo la correspondencia entre ángulos y puntos de la circunferencia no es biunı́voca.


Hay una cantidad infinita de ángulos que están representados por el mismo punto P sobre la
circunferencia goniométrica. Por ejemplo: los ángulos π2 , 9π 3π
2 , − 2 , entre otros están representados


todos por el punto B donde el semieje positivo de las y intersecta a la circunferencia.

Estos ángulos que terminan en un mismo punto de la circunferencia reciben el nombre de


coterminales y tienen la caracterı́stica de que sus medidas en radianes difieren en un múltiplo de
2π. Esto equivale a que los arcos que estos ángulos recorren difieren en una cantidad entera de
vueltas completas a la circunferencia, que pueden ser en sentido horario o antihorario. Luego todos
los ángulos coterminales representados por un mismo punto P de la circunferencia goniométrica
pueden escribirse por medio de la expresión:
θk = θ + 2kπ
donde k puede ser cualquier número entero y θ es cualquiera de los ángulos que termina en P .

En este punto estamos listos para definir las funciones trigonométricas. Cada ángulo se iden-
tificará con el valor de su amplitud (que, como hemos visto, es un número real), por lo que las
funciones en realidad no están definidas para ángulos (que son objetos geométricos) sino para sus
amplitudes (que son números reales). De este modo las funciones que definiremos a continuación
son funciones reales de variable real.

Utilizaremos la correspondencia entre los números reales que representan las amplitudes de
los ángulos medidas en radianes y los puntos P donde el lado final del ángulo intersecta a la
circunferencia goniométrica.

Veremos primero lo que sucede para ángulos agudos de modo que la definición sea consistente
con la presentada en la Sección anterior. Para un ángulo agudo, el punto P de la circunferencia
que lo representa está en el primer cuadrante del sistema de coordenadas. Cosideremos el triángulo
rectángulo T OP que se forma al trazar las lı́neas coordenadas del punto P .

114
Según las definiciones de razones trigonométricas estudiadas y dado que la hipotenusa del
triángulo es el radio de la circunferencia, que es igual a 1, se tiene que:

TP OT
sen θ = y cos θ =
1 1

luego tenemos que OT = cos θ y que OS = sen θ, por lo que las coordenadas del punto P en este
sistema de coordenadas son P (cos θ, sen θ).

Este análisis servirá de base para definir las siguientes funciones que llamaremos funciones
trigonométricas:

Definición 4.3.1. Dado un ángulo de amplitud θ ∈ R consideremos el punto P que lo representa


sobre la circunferencia goniométrica y sus coordenadas P (x, y) en el sistema cartesiano descrito.
Se definen las siguientes funciones:

sen θ = ordenada de P = y

cos θ = abscisa de P = x

sen θ y
tan θ = =
cos θ x
cos θ x
cot θ = =
sen θ y

1 1
sec θ = =
cos θ x
1 1
csc θ = =
sen θ y

Note que esta definición es válida para ángulos generales, no necesariamente agudos. En la
siguiente figura se muestran los casos en que el punto P , que representa al ángulo θ sobre la
circunferencia goniométrica, se encuentra en los cuadrantes II, III y IV y los correspondientes
triángulos rectángulos que se forman al trazar las lı́neas coordenadas del punto P .

115
Ahora podemos verificar que, para todo k ∈ Z:
π  
 3π
sen (2kπ) = 0 sen + 2kπ = 1 sen ((2k + 1)π) = 0 sen + 2kπ = −1
2 2

π  
 3π
cos (2kπ) = 1 cos + 2kπ = 0 cos ((2k + 1)π) = −1 cos + 2kπ =0
2 2

Ejercicios

1. Calcule:

a) cos(135o ), tan(240o ), sen(150o ), csc(1575o )


b) sen(−1200o ), cot(−855o ), cot(−240o ), sec(138o )
c) cos( 10π 13π 17π
3 ), sen(− 6 ), cos( 4 )

2. Hallar tan(120◦ ) si sen(120◦ ) = 3
2 .

2
3. Hallar tan( 5π 5π 5π
4 ) y sec( 4 ) si sen( 4 ) = − 2 .
12
4. Si cos α = 13 , hallar sen α y tan α.
5. Indique los ángulos coterminales a 105◦ que estén entre −2000◦ y 2000◦ . ¿Cuántos son?
6. Indique cuántos y cuáles son los ángulos coterminales a − 5π
12 que están en el intervalo
[−10π, 10π].
7. Si k es un número entero, escriba todos los ángulos que las siguientes expresiones describen:

a) 45◦ + k · 360◦ , con −10 ≤ k ≤ 10.


b) − 3π
2 + 2kπ, con −10 ≤ k ≤ 10.
c) (2k + 1) π2 , con k ∈ Z.

116
4.4. Propiedades de las funciones trigonométricas.
En esta sección analizaremos algunas de las propiedades más importantes de las funciones
trigonométricas definidas anteriormente, con el objetivo de facilitar el camino para obtener sus
gráficos.

Acotación

Note que de la propia definición, como las coordenadas de P : −1 ≤ x, y ≤ 1, se tiene que para
todo θ ∈ R,
| sen θ| ≤ 1 ; | cos θ| ≤ 1
| csc θ| ≥ 1 ; | sec θ| ≥ 1
tan θ ∈ R ; cot θ ∈ R

Ceros

Por la definición de las funciones y los rangos de cada una de ellas, los ceros de cada una de las
funciones trigonométricas son:

sen θ = 0 ⇒ θ = kπ

cos θ = 0 ⇒ θ = (2k + 1) π2

tan θ = 0 ⇒ θ = kπ

cot θ = 0 ⇒ θ = (2k + 1) π2

csc θ 6= 0 ∀θ ∈ R

sec θ 6= 0 ∀θ ∈ R

Signo y Monotonı́a

Esta definición permite también analizar el signo de las funciones trigonométricas en cada
cuadrante del sistema de coordenadas. Aún más, de la misma manera, cuando el punto P se mueve
alrededor de la circunferencia goniométrica, podemos ver cómo varı́an los valores de las funciones
sen θ, cos θ y tan θ, al pasar de un cuadrante a otro. En la siguiente tabla se resumen los signos y
la variación de las funciones sen θ, cos θ y tan θ.

117
Cuadrante Rango sen θ cos θ tan θ

π positivo positivo positiva


I 0<θ< 2 creciente decreciente creciente

π positivo negativo negativa


II 2 <θ<π
decreciente decreciente creciente

3π negativo negativo positiva


III π<θ< 2 decreciente creciente creciente

3π negativo positivo negativa


IV 2 < θ < 2π
creciente creciente creciente

lo que es válido también para los ángulos coterminales con θ.

Máximos y Mı́nimos

π
Como vimos antes, la función sen θ alcanza su valor máximo 1 para los ángulos θ = 2 + 2kπ,
mientras que el valor mı́nimo −1 se alcanza para θ = 3π
2 + 2kπ.

Por su parte, la función cos θ alcanza su valor máximo 1 para los ángulos θ = 2kπ, mientras
que el valor mı́nimo −1 se alcanza para θ = (2k + 1)π.

Las funciones tan θ y cot θ pueden tomar todos los valores reales y como son siempre crecientes,
no tienen máximos ni mı́nimos.

La csc θ, por ser el recı́proco del seno, alcanza sus mı́nimos en los máximos del seno y sus
máximos, en los mı́nimos del seno. Lo mismo sucede respecto de las funciones sec θ y cos θ.

A continuación analizaremos las propiedades de simetrı́a de las funciones trigonométricas, de


modo que reduciremos el cálculo del valor de una función en un θ ∈ R, al cálculo de su valor en un
ángulo agudo. De esta manera bastará con graficar estas funciones entre 0 y π2 y luego, a través
de simetrı́as, construir el gráfico en todos los reales.

Periodicidad

Una función f se denomina periódica, de perı́odo T si T es el menor número real que satisface
que, para todo x ∈ Domf , x + T ∈ Domf y f (x + T ) = f (x).

De este modo, se puede ver que las funciones sen θ, cos θ, csc θ y sec θ son funciones periódicas
de perı́odo 2π. Por otro lado, las funciones tan θ y cot θ son periódicas de perı́odo π. De esta forma:

118
sen(θ + 2π) = sen θ, cos(θ + 2π) = cos θ, csc(θ + 2π) = csc θ, sec(θ + 2π) = sec θ

tan(θ + π) = tan θ, cot(θ + π) = cot θ

luego, bastarı́a tener el gráfico de una de estas funciones en un intervalo de longitud igual a su
perı́odo y luego repetirlo sucesivamente a la derecha y a la izquierda para obtener todo el gráfico.

Simetrı́a respecto de θ = 0 y θ = π

En la figura vemos que el punto Q es el simétrico de P con respecto al eje −→x . Si P representa
al ángulo θ, entonces Q representa a los águlos −θ ó 2π − θ. Vemos que las lı́neas coordenadas en
azul tienen la misma longitud, pero signo diferente y las lı́neas en rojo tienen la misma longitud y
signo. Por eso:

sen θ = − sen(−θ) = − sen(2π − θ),


cos θ = cos(−θ) = cos(2π − θ)
y
tan θ = − tan(−θ) = − tan(2π − θ)

π 3π
Simetrı́a respecto de θ = 2 yθ= 2

119
En la figura vemos que el punto Q es el simétrico de P con respecto al eje −→
y . Si P representa
al ángulo θ, entonces Q representa al águlo π − θ. Vemos que las lı́neas coordenadas en azul tienen
la misma longitud y signo y que las lı́neas en rojo tienen la misma longitud, pero signo diferente.
Por eso:
sen θ = sen(π − θ),
cos θ = − cos(π − θ)
y
tan θ = − tan(π − θ)

Simetrı́a respecto del origen

En la figura vemos que el punto Q es el simétrico de P con respecto al origen O. Si P representa


al ángulo θ, entonces Q representa al águlo π + θ. Vemos que las lı́neas coordenadas en azul y en
rojo tienen la misma longitud, pero signo diferente. Por eso:
sen θ = − sen(π + θ),
cos θ = − cos(π + θ)
y
tan θ = tan(π + θ)

Ejercicios

1. Indique los cambios de signo y magnitud de:


a) cot θ entre 0◦ y 360◦ .
b) csc θ entre 0 y π.
c) tan θ entre − π2 y − 3π
2 .

2. Exprese como función del ángulo θ:


sen(θ − π2 ) , tan(θ − π) , sec( 3π
2 − θ)
cos(270◦ + θ) , cot(270◦ − θ) , sec(θ − 180◦ )
3. Demuestre que el perı́odo de las funciones tan θ y cot θ es π.
4. Encuentre los ceros, máximos y mı́nimos de la función f (x) = sen x + cos x. ¿Es periódica
esta función?¿Cuál es su perı́odo?

120
4.5. Construcción de gráficos de funciones trigonométricas.
Ahora resulta más fácil obtener los gráficos de las funciones trigonométricas gracias a las
propiedades estudiadas anteriormente.

En este momento se evidencia la necesidad del sistema radial de medición de ángulos. Si quer-
emos obtener el gráfico de las funciones trigonométricas en un sistema de coordenadas cartesianas
rectangulares; lo usual y conveniente es que en ambos ejes (el de la variable independiente an-
gular θ y el de los valores de la función) se tengan las mismas unidades de medidas, es decir:
números reales. Si utilizáramos el sistema sexagesimal, en cada eje se tendrı́an medidas de distinta
naturaleza.

Bastará graficar la función para ángulos 0 ≤ θ < π2 (ángulos agudos). Luego, utilizando la
simetrı́a respecto de θ = π2 , se extiende el gráfico al intervalo [0, π[. Igualmente, por la simetrı́a
de las funciones respecto de θ = π, se extiende al intervalo [0, 2π[ y luego, por la periodicidad se
extiende a todos los reales.

Para graficar la función y = f (θ) = sen θ en el intervalo [0, π2 [, utilizamos la circunferencia


goniométrica y encontramos el punto P que representa al ángulo de amplitud θ. La ordenada del
punto P es el valor de sen θ y luego trasladamos esa longitud al nuevo sistema de coordenadas,
verticalmente en la abscisa de valor θ. Note que la longitud del arco AP (la medida en radianes
del ángulo) coincide con la abscisa θ del punto del gráfico de la función sen θ.

Fı́jese que el punto P tiene coordenadas P (cos θ, sen θ) en su sistema de coordenadas, mientras
que las coordenadas de Q en su sistema son Q(θ, sen θ). De este modo, como ya calculamos

el valor

del seno para algunos ángulos notables, sabemos que los puntos A( π6 , 21 ), B( π4 , 22 ) y C( π3 , 23 )
pertenecen al gráfico de la función.

Ahora, por las propiedades de simetrı́a, como sen(π − θ) = sen θ, el gráfico es simétrico con
respecto a la recta θ = π2 . Además, como sen(2π − θ) = − sen θ, el gráfico es simétrico con respecto
al punto (π, 0). Luego, por la periodicidad de la función se extiende el gráfico a todos los reales.
La aplicación de estas simetrı́as se refleja en la siguiente figura. Note que los puntos Qi indicados
tienen coordenadas:
Q0 (θ, sen θ) , Q1 (π − θ, sen θ) , Q2 (π + θ, − sen θ) , Q3 (2π − θ, − sen θ) , Q4 (−θ, − sen θ)

121
Ahora el lector podrá repetir el proceso anterior para el resto de las funciones trigonométricas,
de modo que pueda construir los gráficos de las funciones cos θ, tan θ, cot θ, csc θ y sec θ que se
muestran a continuación:

122
4.6. Funciones trigonométricas inversas.
El problema directo es: dado un ángulo α determinar el número a = sen(α), b = cos(α)
ó c = tan(α). Suponga ahora que inversamente, dado el número a quiere determinar un ángulo α
que verifique que sen(α) = a. A continuación mostramos como construir el ángulo, dado el número,
en los casos a > 0 y a < 0.

123
Trazamos una circunferencia unitaria y copiamos un segmento de longitud a sobre el eje x cuan-
do a es positivo y bajo el eje x cuando a es negativo. Trazamos la paralela al eje x y determinamos
ası́ en ambos casos un ángulo cuyo seno es exactamente el número a.

Notamos que este ángulo existe sólo en el caso que −1 ≤ a ≤ 1 y que en este caso es un ángulo
− π2 ≤ α ≤ π2 .

Decimos entonces que α = Arc sen (a). Hemos definido entonces la función arcseno arcsin o
seno inverso Arc sen .

Definición 4.6.1. Sea a un número cualquiera, con −1 ≤ a ≤ 1. El ángulo α = Arc sen (a) si y
sólo si sen(α) = a y − π2 ≤ α ≤ π2 .

Sea ahora b un número, construiremos un ángulo β que verifique cos(β) = b.

Copiamos desde el origen la distancia b hacia el lado positivo del eje x si b > 0 y hacia el
negativo si b < 0. levantamos la perpendicular en el extremo y al cortar a la circunferencia se
determina el ángulo β cuyo coseno es la medida b.

Como en el caso anterior, notamos que este ángulo existe sólo en el caso que −1 ≤ b ≤ 1 y es
un ángulo 0 ≤ β ≤ π.

Decimos entonces que β = Arc cos (b). Hemos definido entonces la función arcocoseno arc cos
o coseno inverso Arc cos .

124
Definición 4.6.2. Sea b un número cualquiera, con −1 ≤ b ≤ 1. El ángulo β = Arc cos (b) si y sólo
si cos(β) = b y 0 ≤ β ≤ π.

Finalmente dado un número c construiremos un ángulo γ tal que tan(γ) = c.

Copiamos desde el punto A un segmento de longitud c tangente a la circunferencia, hacia el


lado positivo del eje y si c > 0 y hacia el lado negativo si c < 0. Al unir el extremo con el origen
se determina el ángulo γ cuya tangente tiene medida c.

Notemos que a diferencia de los dos casos anteriores este ángulo existe para todo número real
c y es un ángulo − π2 < γ < π2 .

Decimos entonces que γ = Arc tan (c). Hemos definido entonces la función arcotangente arctan
o tangente inversa Arc tan .

Definición 4.6.3. Sea c un número cualquiera, El ángulo γ = Arc tan (c) si y sólo si tan(γ) = c y
− π2 < γ < π2 .

Las funciones trigonométricas no son biyectivas, el lector puede verificar que al trazar una recta
paralela al eje x corta en más de un punto a la gráfica de la función, luego hay puntos distintos
del dominio con la misma imagen y la función no es inyectiva. Las restricciones anteriores de los
dominios hacen que sea posible invertirlas.

Los nuevos dominios y recorridos que se escogen para conseguir funciones biyectivas son:

sen x : [− π2 , π2 ] → [−1, 1], cos x : [0, π] → [−1, 1] y tan x :] − π2 , π2 [→ R,

125
Arc sen (x) : [−1, 1] → [− π2 , π2 ], Arc cos (x) : [−1, 1] → [0, π] y Arc tan (x) : R →] − π2 , π2 [.

Ejemplo 1

Probar que p
sen( Arc cos (x)) = cos( Arc sen (x)) = 1 − x2 .
Solución:

Sea α = Arc cos (x) entonces 0 ≤ α ≤ π. Sabemos que sen2 (α) √ + cos2 (α) = 1, y como
2 2
cos( Arc cos (x)) = x tenemos que sen (α) = 1 − x de donde sen(α) = ± 1 − x2 . Pero la función

126

seno es positiva en el primer y segundo cuadrantes, luego sen( Arc cos (x)) = + 1 − x2 . La otra
igualdad es análoga, invitamos al lector a completarla.

Ejemplo 2

Resuelva la ecuación
Arc sen (x) = Arc cos (x).
Solución:

Aplicando la√función sen() en ambos lados de la ecuación y ocupando el problema anterior,


obtenemos x = 1 − x2 elevando al cuadrado obtenemos x = ± √12 . Reemplazando en la ecuación
original obtenemos que sólo x = √12 es solución.

Ejemplo 3

Sea α = Arc cos ( 31 ), determine α en términos de Arc sen y de Arc tan .

Solución:

Considere el triángulo de la figura


2 2
√ √
Por Pitágoras x = 3 . Luego α = Arc sen ( 2 3 2 ) = Arc tan (2 2).

Ejercicios

1. Calcular

a) Arc sen (0) + Arc sen ( √12 ) · Arc sen (−1).


b) Arc cos (0) + Arc cos ( 12 ) + Arc cos (−1).
c) Arc tan (0) + Arc tan (1) + Arc tan (−1).

2. Probar que Arc sen es una función creciente.


3. Probar que Arc cos es una función decreciente.
π
4. Probar que si c > 1 entonces Arc tan (c) > 4.

5. ¿Es Arc tan una función creciente?

127
6. Calcule el valor de

a) sen( Arc cos ( 35 )).


b) cos( Arc tan ( 12
5 )).
c) sen( Arc tan ( 24 12
7 ) − Arc cos ( 13 )).
d ) sen(2 Arc sen ( 45 ) + 1
2
8
Arc cos ( 17 )).

7. Demuestre que:

a) Arc tan ( 14 ) + Arc tan ( 12


5
) = Arc tan ( 32
43 ).
5
b) Arc sen ( 35 ) + Arc sen ( 13 56
) = Arc tan 33 .
q √
c) Arc cos ( 23 ) − Arc cos ( 26+1 √ ) = π.
3 6

d ) 2 Arc tan ( 18 ) + Arc tan ( 17 ) + 2 Arc tan ( 15 ) = π


4.

8. Resuelva la ecuación Arc cos x − Arc sen x = Arc cos 3x
   
x−1 x+1
9. Resuelva la ecuación Arc tan x−2 + Arc tan x+2 = π4

10. Demuestre que si u, v ∈ R+ , entonces


r   
u v−u
2 Arc tan = Arc cos .
v v+u

11. Desarrolle Arc tan (k + 1) − Arc tan (k − 1) y luego calcule

n
X 2
Arc tan ( ).
k2
k=1

12. Desarrolle Arc tan ( k1 ) − Arc tan ( k+1


1
) y luego calcule

n
X 1
Arc tan ( ).
k2 + k + 1
k=1

4.7. Identidades trigonométricas.

En esta sección revisaremos algunas de las más conocidas identidades trigonométricas. Una
identidad de este tipo es una igualdad algebraica que involucra a funciones trigonométricas de uno
o más ángulos y que es verdadera para todo ángulo que esté en el dominio de definición de la
relación algebraica. Algunas de las más sencillas ya han sido utilizadas anteriormente, por ejemplo:
cos θ
sen2 θ + cos2 θ = 1 , cot θ = , tan θ = tan(θ + π) , cos θ = cos(−θ)
sen θ

128
Note que la primera y la última se cumplen para todo ángulo θ ∈ R, mientras que la segunda
y la tercera son ciertas para todo ángulo θ que no indefina las expresiones correspondientes.

Suma y diferencia de ángulos

Dados dos ángulos α y β, si los representamos en la circunferencia goniométrica, como en la figu-


ra, podemos ver que las coordenadas del punto P son P (cos α, sen α) y las de Q son Q(cos β, sen β).
Si el ángulo ∠P OQ = β − α se traslada a su posición normal (en la figura ∠AOB), vemos que
intersecta a la circunferencia en A(1, 0) y B(cos(β − α), sen(β − α)).

Observemos que la distancia entre P y Q se conserva, d(P, Q) = d(A, B), al igual que sus
cuadrados. Utilizando la fórmula de distancia entre dos puntos y teniendo las coordenadas resulta
que:
(cos β − cos α)2 + (sen β − sen α)2 = (cos(β − α) − 1)2 + sen2 (β − α)
desarrollando:

cos2 β − 2 cos β cos α + cos2 α + sen2 β − 2 sen β sen α + sen2 α =

cos2 (β − α) − 2 cos(β − α) + 1 + sen2 (β − α)

Utilizando la identidad fundamental para los ángulos α, β y β − α, se tiene:

cos(β − α) = cos β cos α + sen β sen α, (1)


para todo par de números reales α, β ∈ R

Ahora es fácil verificar la veracidad de las siguientes identidades trigonométricas, para todo
α, β ∈ R.

cos(β + α) = cos β cos α − sen β sen α, (2)

sen(β − α) = sen β cos α − cos β sen α, (3)

129
sen(β + α) = sen β cos α + cos β sen α, (4)

En efecto, si en la ecuación (2) hacemos cos(β − (−α)), podemos aplicar (1) y desarrollar esta
diferencia. Hay que notar que cos(−α) = cos α y que sen(−α) = − sen α dado que la función coseno
es par y la función seno es impar. Empleando esta última propiedad se obtiene (2).
π 
Para obtener (3) debemos poner sen(β − α) = cos − (β − α) , por ser ángulos complemen-
 π 2 
tarios. Luego reescribimos sen(β − α) = cos − β + α) y utilizamos (2) para desarrollar el
2 π  π 
coseno de esta suma. En el desarrollo aparecen los términos cos − β y sen − β que, al ser
2 2
sustituı́dos por sen β y cos β respectivamente, proporcionan la identidad (3).

Para obtener (4) se repite el argumento utilizado para verificar (2). Como sólo hemos indicado
la idea de la demostración, recomendamos al lector que realice la comprobación detallada de las
identidades (2), (3) y (4), a partir de los argumentos indicados.

Las siguientes identidades se obtienen directamente de las fórmulas presentadas anteriormente.


Indicaremos cómo verificar cada una de ellas y, en algunos casos, dejaremos al lector completar los
detalles.

tan α ± tan β
tan(α ± β) = , (5)
1 ∓ tan α tan β
sen(α ± β) sen β cos α ± cos β sen α
tan(α ± β) = =
cos(α ± β) cos β cos α ∓ sen β sen α

Si se divide el numerador y el denominador por cos α cos β se obtiene (5).

Ángulo Doble y Ángulo Mitad

sen 2α = 2 sen α cos α, (6)

cos 2α = cos2 α − sen2 α, (7)

2 tan α
tan 2α = , (8)
1 − tan2 α

Para probar (6), (7) y (8) hay que hacer β = α en (4), (2) y (5), respectivamente.

r
θ 1 − cos θ
sen = ± , (9)
2 2

130
r
θ 1 + cos θ
cos = ± , (10)
2 2

r
θ 1 − cos θ
tan = ± , (11)
2 1 + cos θ

Para probar (9), (10) y (11) veamos que de la identidad (7) se tiene que:

cos 2α = 2 cos2 α − 1 = 1 − 2 sen2 α


Si ahora hacemos θ = 2α, tenemos:
θ θ
cos θ = 2 cos2 − 1 = 1 − 2 sen2
2 2
de donde se obtienen las identidades buscadas.

Transformación de Productos en Sumas

2 sen α cos β = sen(α + β) + sen(α − β), (12)

2 cos α sen β = sen(α + β) − sen(α − β), (13)

2 cos α cos β = cos(α + β) + cos(α − β), (14)

2 sen α sen β = cos(α − β) − cos(α + β), (15)

Para verificar estas identidades basta desarrollar los miembros derechos de cada igualdad em-
pleando las identidades anteriores.

Transformación de Sumas en Productos


   
γ+δ γ−δ
sen γ + sen δ = 2 sen cos , (16)
2 2
   
γ+δ γ−δ
sen γ − sen δ = 2 cos sen , (17)
2 2
   
γ+δ γ−δ
cos γ + cos δ = 2 cos cos , (18)
2 2
   
γ+δ γ−δ
cos γ − cos δ = 2 sen sen , (19)
2 2

131
Basta hacer, en las identidades (12) a (15), α + β = γ y α − β = δ, de donde se tiene que
γ+δ γ−δ
α= y β= .
2 2

Ejercicios

1. Probar las siguientes identidades

cos (α + β + γ)
a) = 1 − tan α · tan β − tan α · tan γ − tan β · tan γ
cos α · cos β · cos γ
sen(3α) + sen3 (α)
b) = cot(α)
cos3 (α) − cos(3α)
sen β · cos α · (tan α + tan β) sen( α−β
2 )
c) + =1
1 − cos(α + β) cos β · sen( α+β
2 )
d ) cos(45o + α) + sen(α − 45o ) = 0

2. En los siguientes ejercicios suponga α + β + γ = π. Pruebe que:

a) sen α + sen β + sen γ = 4 · cos( α2 ) · cos( β2 ) · cos( γ2 )


b) tan α + tan β + tan γ = tan α · tan β · tan γ

3. Si a cos2 α + b sen2 α = c, probar que


c−a
tan2 α =
b−c

4. Si
n sen α · cos α
tan β =
1 − n sen2 α
Probar que
(1 − n) tan α = tan(α − β)

5. Comprobar las siguientes identidades


β β
a) (cos − sen )2 = 1 − sen β
2 2
α
b) tan = csc α − cot α
2
sen 2β cos 2β
c) − = sec β
sen β cos β
d ) sen 3θ = 3 sen θ − 4 sen3 θ
e) cos 3θ = 4 cos3 θ − 3 cos θ
1 − cos 2α
f) = tan α
sen 2α
g) csc 2γ + cot 2γ = cot γ

132
3 tan δ − tan3 δ
h) tan 3δ =
1 − 3 tan2 δ
tan δ + cot δ
i) = sec 2δ
cot δ − tan δ
sen 3α cos 3α
j) − =2
sen α cos α

4.8. Ecuaciones tigonométricas.


Las ecuaciones trigonométricas básicas son: (1) sen(x) = a, (2) cos(x) = a y (3) tan(x) = a
donde a es un número real.

Analizaremos sus soluciones en cada caso determinando una fórmula que permite solucionar
completamente la ecuación.

Consideremos en primer lugar la ecuación (1) y analicémosla desde el punto de vista gráfico.
Las soluciones se obtienen al intersectar la gráfica de y = sen(x), con y = a.

Como se observa en la gráfica esta pregunta sólo tiene sentido si |a| ≤ 1, puesto que si |a| > 1
la recta no corta a la función sinusoidal. También nos damos cuenta que la ecuación tiene infinitas
soluciones, por la forma de la función. Todos los cortes pueden determinarse a partir de dos
soluciones iniciales sumando múltiplos de 2π. Estas soluciones fundamentales son:
x0 = Arc sen (a) e y0 = π − Arc sen (a). Luego la solución general de la ecuación está dada por

xk = Arc sen (a) + 2kπ o bien xk = π − Arc sen (a) + 2kπ.

Las distintas soluciones que se ven en el gráfico, resultan de hacer variar el parámetro k en el
conjunto de los números enteros Z = {0, ±1, ±2, ....}

Ejemplo 1

133
1
Resolver la ecuación sen(x) = .
2
Solución
Sus soluciones, de acuerdo a la fórmula anterior son
1 1
xk = Arc sen ( ) + 2kπ o bien xk = π − Arc sen ( ) + 2kπ.
2 2
1 π
Como Arc sen ( ) = . lo anterior se simplifica como
2 6
π π
xk = + 2kπ o bien xk = π − + 2kπ.
6 6
Consideremos ahora la ecuación (2) cos(x) = a. Sus soluciones se obtienen al intersectar la
gráfica de y = cos(x), con y = a.

Como antes, esta ecuación sólo tiene sentido si |a| ≤ 1, puesto que si |a| > 1 la recta no corta a
la cosinusoide. También nos damos cuenta que la ecuación tiene infinitas soluciones, por la forma
de la función. Todos los cortes pueden determinarse a partir de dos soluciones iniciales sumando
múltiplos de 2π. Estas soluciones fundamentales son:
x0 = Arc cos (a) e y0 = − Arc cos (a). Luego la solución general de la ecuación está dada por

xk = ± Arc cos (a) + 2kπ.


Las distintas soluciones que se ven en el gráfico, resultan de hacer variar el parámetro k en Z.

Ejemplo 2
1
Resolver la ecuación cos(x) = − .
2
Solución
Sus soluciones, de acuerdo a la fórmula anterior son

1
xk = ± Arc cos (− ) + 2kπ.
2

134
1 2
Como Arc cos (− ) = π. lo anterior se simplifica a
2 3
2
xk = ± π + 2kπ.
3
Finalmente consideremos la ecuación (3) tan(x) = a. Sus soluciones se obtienen al intersectar
la gráfica de y = tan(x), con y = a.

A diferencia de los casos anteriores, esta ecuación tiene solución para cualquier número real a.
Como antes, nos damos cuenta que la ecuación tiene infinitas soluciones, por la periodicidad de la
función. Todos los cortes pueden determinarse a partir de una solución inicial sumando múltiplos
del perı́odo π. La solución fundamental es

x0 = Arc tan (a), luego la solución general de la ecuación está dada por

xk = Arc tan (a) + kπ.


Las distintas soluciones que se ven en el gráfico, resultan de hacer variar el parámetro k en Z.

Ejemplo 3

Resolver la ecuación tan(x) = − 3.

Solución

Sus soluciones, de acuerdo a la fórmula anterior son

135

xk = Arc tan (− 3) + kπ.
√ π
Como Arc tan (− 3) = − . lo anterior se simplifica a
3

π
xk = − + kπ.
3

Ejercicios

1. Resuelva las siguientes ecuaciones en [0, 2π]. Escriba además la solución general.

a) sen x = √1
2
b) cos x = 2
1
c) cos x = 2
1
d ) tan2 x = 3
π 1
e) cos(3x − 4) = 2
f ) cos 3x = cos 2x

2. Resuelva las siguientes ecuaciones:

a) 2 cos2 x + 4 sen2 x = 3
b) (tan x − 1)(tan x + 3) = 2 tan x
c) 4 sen3 x − 2 sen2 x − 2 sen x + 1 = 0

d ) 2 cos x + 2 2 = 3 sec x
e) tan x − cot x = csc x

f ) 6 tan x − 5 3 sec x + 12 cot x = 0
g) 3 sen x + 4 cos x = 5
h) tan( π4 + x) = 1 − sen 2x
i ) tan3 x + cot3 x = 12 + 8 csc3 2x
j ) sec 4x − sec 2x = 2
1
k ) sen4 θ + cos4 θ = 2
l ) cos 2α = cos α − sen α
√ √
m) tan β − 3 cot β + 1 = 3
n) sen 5x + sen x = sen 3x
ñ) 1 + cos y = 2 sen2 y
o) sen 7α = sen 4α − sen α
p) sen(6x − π4 ) = sen(2x + π4 )
q) sen(3x − π6 ) = sen(x + π3 )
r ) cos x + cos 3x + cos 5x + cos 7x = 0

136

2
s) cos x − sen x = 2
t) tan4 x − 4 tan2 x + 3 = 0
5
u) sen4 x + cos4 x = 8

3. Resolviendo la ecuación sen 2x = cos 3x encuentre el valor de sen 18◦



4. Si a cos x + b sen x = c, muestre que a sen x − b cos x = ± a2 + b2 − c2 . Luego resuelva
5 sen x − 2 cos x = 1
5. Elimine α de los siguientes pares de ecuaciones:

a) cos α cos a = a, sen α cos a = b


b) cos(α − a) = a, cos(α + a) = b

4.9. Ángulos y lados de un triángulo.


Sea dado un triángulo rectángulo ABC, recto en C, de catetos a y b, hipotenusa c y ángulos
agudos α y β. Pensemos en la pregunta: ¿cuál es la información mı́nima que necesitamos para
determinar este triángulo completamente? Es decir, de los cinco elementos desconocidos del 4ABC
¿cuántos y cuáles debemos conocer para poder calcular el resto de ellos?

Es claro que es necesario conocer la longitud de, al menos uno de los lados del triángulo, pues de
lo contrario, hay toda una familia de triángulos rectángulos semejantes que comparten los mismos
ángulos.

Si se conoce alguno de los ángulos agudos, como el triángulo es rectángulo, el otro ángulo es
complementario del primero. Si se conocen dos lados, el tercero se puede encontar a través del
Teorema de Pitágoras. Eso quiere decir que para triángulos rectángulos basta conocer la longitud
de un lado y cualquiera de los otros cuatro elementos desconocidos (uno de los dos lados restantes
o uno de los ángulos agudos) para que el triángulo quede completamente determinado. En efecto,
veamos los posibles casos:

Dos lados: Conocidos dos de los lados (no importa cuáles de ellos) el tercero se determina a
través de la relación a2 + b2 = c2 , con lo que se conocen todos. Luego, para determinar cada ángulo
a a π
agudo basta notar que sen α = , por lo que α = Arc sen y β = − α.
c c 2
Un lado y un ángulo agudo: Conocido uno de los ángulos agudos, el otro es complementario
a
con él. Supongamos que se conoce el cateto a, como sen α = , tenemos que c = a sen α y luego se
c
obtiene el otro cateto por Pitágoras. Es fácil ver que si el lado conocido es otro, también se pueden
determinar los elementos que faltan.

Queremos hacer ahora el mismo análisis para triángulos generales, no necesariamente rectángu-
los. Para esto necesitaremos dos resultados muy conocidos:

En ambos casos utilizaremos la notación usual para triángulos


Vértices: A, B, C; Lados: a, b, c; Ángulos: α, β, γ

137
Teorema 4.9.0.56. Teorema del Seno. En un triángulo cualquiera ABC se tiene que:

sen α sen β sen γ


= =
a b c

Demostración:

Basta ver la figura donde está trazada una de las alturas del triángulo, de manera que queda
dividido en dos triángulos rectángulos. En cada uno de ellos se expresa la altura en términos de
los ángulos α, β ó γ y se obtiene la igualdad.
h h
sen α = , sen β =
b a
de donde
b sen α = h = a sen β
sen α sen β
=
a b
Si ahora se traza otra altura del triángulo y se repite el proceso, se obtiene la igualdad restante
del Teorema.

138
Teorema 4.9.0.57. Teorema del Coseno. En un triángulo cualquiera ABC se tiene que:

a2 = b2 + c2 − 2bc cos α
b2 = a2 + c2 − 2ac cos β
c2 = a2 + b2 − 2ab cos γ

Demostración:

Note que, en la figura, la altura h es cateto de los triángulos AHC y BHC, luego:

h2 = b2 − x2 = a2 − (c − x)2

de donde
a2 = b2 + c2 − 2cx
basta escribir x = b cos α para obtener la primera expresión del Teorema. Si se quiere obtener las
otras dos expresiones hay que repetir la demostración trazando las otras dos alturas del triángulo.

Ejercicios

1. El ángulo de elevación de una torre CD desde un lugar A al Sur de ella es 30o y desde un
lugar B hacia el Oeste de A, su ángulo de elevación es de 18o . Si AB = a probar que la altura
de la torre es
a
p √
2+2 5
2. Calcular el ángulo agudo que forman entre sı́ las diagonales de un rectángulo, si las longitudes
de sus lados son 2,2m y 1,4m.
3. En cada caso calcular los demás ángulos y lados con los datos del triángulo que se indican a
continuación:

a) c = 2, α = 75o , β = 60o

139

3 1
b) a = 1, b = 2 ,c = 2
c) b = 3,07, α = 26o , γ = 49o

d ) a = 2, β = 60o , c = 3 + 1

4. Probar que en un triángulo ABC se cumple:


γ α
2 · (a sen2 + c · sen2 ) = a − b + c
2 2

5. Probar que si en un triángulo ABC se tiene que:


sen α + sen β
sen γ =
cos α + cos β
entonces el triángulo es rectángulo en C.
6. Demostrar que en todo triángulo ABC se cumple:
cos β c − b cos α
a) =
cos γ b − c cos α
a sen γ
b) tan α =
b − a cos γ
7. En un triángulo ABC α = 30o , β = 50o probar que los lados del triángulo cumplen la
relación:
c2 = b(a + b)

8. En un triángulo ABC se tiene que γ = 60o y que su perı́metro 2s está dado por 2s = a + b + c
probar que:
1 1 3
+ =
a+c b+c 2s
9. En un triángulo ABC se cumple:

b3 + c3 − a3
= a2
b+c−a
3
y sen β · sen γ = 4 probar que el triángulo es equilátero.
10. Un avión vuela en lı́nea recta a una altura de 1000m. A la 13 horas se encuentra en A y
asciende bruscamente desviándose 30o con la horizontal, manteniendo el movimiento rec-
tilı́neo con rapidez constante. Después de 10 segundos el avión se encuentra en B. Si desde
una torre de observación en la tierra, se tienen ángulos de elevación de 30o y 45o para los pun-
tos A y B respectivamente, indicar cuál es la rapidez del avión suponiendo que las visuales
de los puntos A y B y la trayectoria del avión están en un plano.

140
Capı́tulo 5

Vectores y geometrı́a analı́tica

En este capı́tulo estudiamos los vectores en el plano como en el espacio y sus propiedades.
Introducimos las coordenadas en forma general, lo que nos permite determinar ecuaciones de
rectas y planos en el espacio como también de rectas en el plano. Estudiamos las propiedades
de los productos punto y cruz entre vectores y los aplicamos en las proyecciones y el cálculo de
distancias. Aplicamos los vectores para resolver problemas geométricos.

5.1. Segmentos dirigidos y vectores


Los vectores aparecen asociados a magnitudes estudiadas en la fı́sica. La velocidad y la acel-
eración de un cuerpo, por ejemplo, son elementos que tienen asociada una magnitud y una direc-
ción, tales magnitudes se conocen como “magnitudes vectoriales..Estas magnitudes se diferencian
de aquellas relacionadas con la masa, el volumen y el tiempo que tienen asociadas sólo un valor
real. Estudiaremos a continuación los vectores del punto de vista geométrico tanto en el plano
como en el espacio.

Definición 5.1.1. Segmento dirigido. Dados dos puntos A y B en el plano o en el espacio por (A, B)
denotamos el segmento dirigido con punto inicial A, y punto final B.

Los segmentos dirigidos anteriores tienen una dirección, que está dada por la recta que de-
terminan los puntos A y B, tienen un sentido dado, que identifica en ellos un punto inicial y un

141
punto final. En las figuras se observa que (A, B) 6= (B, A). Los segmentos dirigidos tienen también
asociada un valor, dado por la distancia entre los puntos A y B.

El concepto de vector es algo un poco más general que el de segmento dirigido. En el sentido
que “dos segmentos dirigidos que coincidan en dirección, sentido y magnitud, representan el mismo
vector”. Entender el concepto anterior es clave en el trabajo vectorial. Para acercarse a este enfoque,
el lector puede hacer un paralelo con las fracciones y los racionales.

Ası́ como todas las fracciones 42 , 36 , 48 , 10


5
, ... se identifican con un único número racional 12 , un
vector es el representante de todos los segmentos dirigidos que tienen igual magnitud, dirección y
sentido que él.

Lo anterior se formaliza matemáticamente considerando el conjunto de todos los segmentos


dirigidos del plano o el espacio y estableciendo la siguiente relación entre ellos.

(A, B) ↑ (C, D) si los segmentos dirigidos (A, B) y (C, D) tienen igual magnitud, dirección y
sentido. La relación anterior recibe el nombre de equipolencia de segmentos dirigidos.

Teorema 5.1.0.58. La relación de equipolencia es una relación de equivalencia, esto es: es refleja,
simétrica y transitiva.

Demostración.

Como (A, B) ↑ (A, B) la relación ↑ es refleja. Además como (A, B) ↑ (C, D) implica claramente
que (C, D) ↑ (A, B) esta relación es simétrica.

Si (A, B) ↑ (C, D) y (C, D) ↑ (E, F ) los segmentos dirigidos (A, B) y (E, F ) tendrán claramente
igual magnitud, dirección y sentido, por lo que son equipolentes.

Consideremos un punto O del plano o del espacio que será considerado nuestro origen, pasamos
a definir formalmente los vectores como clases de equivalencia.

Definición 5.1.2. Vector. Sea P un punto del plano, denotamos por p~ al vector asociado a los
puntos O y P, este vector queda definido como p~ = [(O, P )] = {(A, B) : (A, B) ↑ (O, P )}

142
Definición 5.1.3. El vector nulo queda definido por ~0 = [(O, O)]

Denotaremos por k~ak a la magnitud del vector ~a que llamaremos, su norma.

5.2. Operaciones con vectores


Existe una forma de asociar a dos vectores dados, un tercer vector llamado su suma, llamamos
a este proceso adición vectorial.

Consideremos dos vectores ~a y ~b. El vector ~a + ~b es aquél que resulta al unir el origen con el
vértice del paralelógramo formado por ~a y ~b.

Si los vectores son colineales y tienen el mismo sentido, el resultado de la suma es un vector que
tiene el mismo sentido que los vectores anteriores y cuya magnitud es la suma de las magnitudes
de los dos anteriores.

Si los vectores son colineales y tienen sentidos opuestos, el resultado de la suma es un vector
que tiene el sentido del vector de mayor magnitud y como magnitud la diferencia de las magnitudes
de los vectores anteriores.

143
Una segunda operación con vectores es la ponderación por un escalar, que es un número real.
Denotaremos los escalares por α, β, γ, ...

Si ~a es un vector y α es un escalar α~a, es un vector con las siguientes caracterı́sticas:

Si α es positivo, α~a, mantiene el sentido del vector ~a. Si por el contrario α es negativo, α~a tiene
el sentido opuesto al vector ~a.

En general los escalares 0 < α < 1 contraen el vector y los escalares α > 1 lo dilatan.

kα~ak = |α|k~ak

Propiedades de la adición y ponderación de vectores.

El conjunto V de los vectores geométricos tiene las siguientes propiedades respecto a la adición
y ponderación ya definidas:

1. ∀~a, ~b ∈ V : ~a + ~b ∈ V. [Clausura]

2. ∀~a, ~b, ~c ∈ V : (~a + ~b) + ~c = ~a + (~b + ~c). [Asociatividad]


3. ∃~0 ∈ V : ∀~a ∈ V : ~a + ~0 = ~a.[Elemento neutro]
4. ∀~a ∈ V : ∃ − ~a ∈ V : ~a + −~a = ~0.[Opuesto aditivo]

5. ∀~a, ~b ∈ V : ~a + ~b = ~b + ~a. [Conmutatividad]


6. ∀~a ∈ V, λ ∈ R : λ~a ∈ V.[Clausura]
7. ∀~a ∈ V : 1~a = ~a
8. ∀~a ∈ V, λ, µ ∈ R : λ(µ~a) = (λµ)~a.
9. ∀~a ∈ V, λ, µ ∈ R : (λ + µ)~a = λ~a + µ~a.

10. ∀~a, ~b ∈ V, λ ∈ R : λ(~a + ~b) = λ~a + λ~b.

144
Las propiedades anteriores son de suma utilidad porque nos permiten hacer álgebra vectorial,
por ejemplo:

Ejemplo 5.2.1. Simplificar 2~a + 3~b − 2~a + ~b. Conmutando, asociando y utilizando la propiedad del
neutro aditivo, la expresión anterior es igual a 4~b.
Definición 5.2.2. ~a − ~b = ~a + (−~b).

5.3. Sistemas de coordenadas


Asociaremos a cada vector ~v del plano un par de números reales. ~v ↔ (α, β) de la siguiente
forma.

Consideramos un punto fijo O como origen del plano y dos vectores no colineales v~1 y v~2 que
forman nuestra base B. Esto es B = {v~1 , v~2 }.

Utilizaremos los vectores anteriores para realizar la asociación. Trazamos las paralelas a los
vectores por el extremo final de ~v . Determinamos de esta forma puntos de corte P y Q con las
rectas que representan la dirección de los vectores y que pasan por O. Determinamos entonces de
manera única, números reales α y β, de modo que p~ = αv~1 y ~q = β v~2 . Las coordenadas del vector
~v en la base B serán entonces (α, β).

Lo anterior permite definir


R2 = {(α, β) : α, β ∈ R}
Podemos definir una suma y ponderación en R2 , acorde con la suma y ponderación vectorial
definida anteriormente, de la siguiente forma.

~x = (a1 , a2 ), ~y = (b1 , b2 ), α ∈ R se definen ~x + ~y = (a1 + b1 , a2 + b2 ); α~x = (αa1 , αa2 ).

Con la suma y ponderación antes definidas (R2 , +, ·) cumple los postulados de espacio vectorial.
Dejamos esta verificación al lector como ejercicio.

145
El lector estará acostumbrado a utilizar vectores básicos perpendiculares y unitarios v~1 , v~2 . Si
esto ocurre, el sistema se conoce como sistema cartesiano ortogonal, en este caso ı̂ = (1, 0),̂ = (0, 1)
y cualquier vector (x, y) = x(1, 0) + y(0, 1) = xı̂ + y̂.

Una identificación similar realizaremos para los puntos del espacio.

Asociaremos a cada vector ~v del espacio un trı́o de números reales. ~v ↔ (α, β, γ) de la siguiente
forma.

Consideramos un punto fijo O como origen del espacio y tres vectores no colineales y no
coplanares v~1 , v~2 y v~3 que forman nuestra base B. Esto es B = {v~1 , v~2 , v~3 }.

Determinamos entonces de manera única números reales α, β y γ, de modo que las coordenadas
del vector ~v en la base B serán entonces (α, β, γ).

Lo anterior permite definir


R3 = {(α, β, γ) : α, β, γ ∈ R}
Podemos definir una suma y ponderación en R3 , acorde con la suma y ponderación vectorial
definidas anteriormente, de la siguiente forma.

~x = (a1 , a2 , a3 ), ~y = (b1 , b2 , b3 ), α ∈ R se definen ~x + ~y = (a1 + b1 , a2 + b2 , a3 + b3 );


α~x = (αa1 , αa2 , αa3 ).

Con la suma y ponderación antes definidas, el espacio (R3 , +, ·) cumple los postulados de espacio
vectorial. Dejamos esta verificación al lector como ejercicio.

El lector estará acostumbrado a uilizar vectores básicos perpendiculares y unitarios v~1 , v~2 , v~3 . Si
esto ocurre el sistema se conoce como sistema cartesiano ortogonal, en este caso ê1 = (1, 0, 0), ê2 =
(0, 1, 0), ê3 = (0, 0, 1) y cualquier vector (x, y, z) = x(1, 0, 0)+y(0, 1, 0)+z(0, 0, 1) = xê1 +yê2 +zê3 .

En Fı́sica los vectores ê1 , ê2 , ê3 se denotan normalmente por ı̂,̂, k̂.

146
5.4. Propiedades fundamentales

~ = ~b − ~a.
Teorema 5.4.0.59. Si A y B son dos puntos del plano o del espacio entonces AB

Para demostrar lo anterior

~ = ~b. Luego sumando −~a en ambos lados de la


note que directamente de la definición ~a + AB
~ ~
igualdad anterior obtenemos AB = b − ~a.

Ejemplo 5.4.1. En el plano, considere los puntos A = (1, 2) y B = (3, 1), el vector

~ = ~b − ~a = (1, 2) − (3, 1) = (2, −1).


AB

Ejemplo 5.4.2. En el espacio, considere los puntos A = (1, 1, 1) y B = (3, 2, 2), el vector

~ = ~b − ~a = (3, 2, 2) − (1, 1, 1) = (2, 1, 1).


AB

147
Teorema 5.4.0.60. Si A y B son dos puntos fijos del plano o del espacio y P es un punto que está en
AP
el segmento de recta que los une, de modo que = λ, entonces es posible determinar el vector
PB
posición del punto P en términos de los vectores ~a, ~b y la razón de división λ, de modo que

~a + λ~b
p~ =
1+λ

~ = λP~B, luego utilizando el teorema anterior


Para demostrar esto, note que AP

p~ − ~a = λ(~b − p~). Desarrollando p~ − ~a = λ~b − λ~ p = ~a + λ~b,


p, luego p~ + λ~
por lo que
~a + λ~b
(1 + λ)~ p = ~a + λ~b, entonces p~ =
1+λ
Observación 5.4.3. Note que si se impone como condición AP ~ = αAB,
~ entonces se prueba de igual
forma que
p~ = (1 − α)~a + α~b.

148
Tal combinación recibe el nombre de combinación convexa de los vectores ~a y ~b. Si hacemos variar
α entre cero y uno, p~ recorre todos los puntos del segmento entre A y B.

Ejemplo 5.4.4. Determinar el vector posición del punto medio de un trazo en términos de los
vectores posición de los extremos del trazo.

AM a+~b
~
En este caso λ = = 1 por lo que utilizando la fórmula inicial m
~ = 2 .
MB

Ejemplo 5.4.5. Determinar los vectores de posición de los puntos de trisección de un trazo en
términos de los vectores posición de los extremos.

2~a + ~b ~a + 2~b
p~ = ~q = .
3 3

Ejemplo 5.4.6. Determinar las coordenadas de los puntos medios de los lados y del centro de
gravedad G del triángulo con vértices de coordenadas A = (1, 1), B = (3, 2) y C = (3/2, 5).

149
AM
Como M es punto medio de AB, = λ = 1. Utilizando la fórmula anterior
MB

~a + 1~b ~a + ~b (1, 1) + (3, 2)


m
~ = = = = (2, 3/2),
1+1 2 2
~b+~c ~
a+~ c
Del mismo modo ~n = 2 = (9/4, 7/2) y m
~ = 2 = (5/4, 3). Para calcular las coordenadas de G,
AG 2
GB = 1 luego
~a + 2m
~
~g = = (11/6, 3)
3

Ejercicios

1. Utilizar las propiedades de espacio vectorial para simplificar las siguientes expresiones:

a) 8(2~u + ~v ) − 15(~v − 3~u).


b) 6(~u + 8~v − w)
~ − 4(w
~ + ~v ).

2. p~ y ~q son los vectores posición de los puntos P y Q respectivamente y R es el punto cuyo


vector posición es p~ − ~q. Expesar en términos de p~ y ~q los siguientes vectores:
~
a) QP
b) P~Q
~ − RO,
c) RP ~ donde O es el origen.

3. Suponga que los puntos P y Q tienen coordenadas P (1, −1, 3) y Q(3, 1, 0) y el punto R
verifica, ~r = p~ − ~q. Determinar las coordenadas de los siguientes vectores:
~
a) QP

150
~
b) QR
~ − RO,
c) RP ~ donde O es el origen.

4. Considere los puntos A(2, 3) y B(5, −1) del plano. Determinar las coordenadas de todos los
puntos necesarios para dividir el trazo AB en cinco trazos de igual medida.
5. En la figura se muestran dos hexágonos regulares de lado 1cm. Se han dispuesto distintos
sistemas coordenados para cada uno de ellos. Determinar las coordenadas de ~a, ~b, ~c, d,
~ ~e y f~
en cada caso.
(Note que las coordenadas de los vectores dependen de dónde se ubique el origen del sistema.)

6. Probar que el segmento que une los puntos medios de dos lados de un triángulo es paralelo
al tercer lado y mide la mitad de este.
7. Probar que el vector posición del centro de gravedad G de un triángulo ABC está dado por

~a + ~b + ~c
~g =
3

8. Determinar las coordenadas de un punto P en el segmento que une P1 = (1, −1, 4) y P2 =


(−2, 1, 3) tal que kP~P1 k = 4kP~P2 k.
9. Demostrar que al unir los puntos medios de los lados de un cuadrilátero se obtiene un par-
alelógramo.
10. Sean A, B, C, D, E y F , en ese orden, los vértices de un hexágono regular. Demostrar que
~ + AC
AB ~ + AD
~ + AE
~ + AF
~ = 3AD.
~

11. a) Sean P1 , P2 , P3 , P4 , P5 y P6 seis puntos distribuidos a igual distancia en una circunfer-


encia con centro C. Demostrar que:

~ 1 + CP
CP ~ 2 + CP
~ 3 + CP
~ 4 + CP
~ 5 + CP
~ 6 = ~0.

151
b) Demostrar que la afirmación hecha en (a) sigue siendo válida para cualquier conjunto
par de puntos sobre una circunferencia.
c) Demostrar que la afirmación hecha en (a) es cierta para tres puntos.
d ) ¿Es posible que la afirmación hecha en (a) sea cierta para cualquier conjunto finito de
puntos equiespaciados sobre la circunferencia ?

5.5. Ecuación de la recta

Recta determinada por dos puntos

Sea R un punto cualquiera de la recta que pasa por dos puntos fijos A y B.
~ de donde se obtiene ~r = ~a + λ(~b − ~a)
~ = λAB
AR

Recta determinada por un punto y una dirección

Supongamos que conocemos ahora un punto A de la recta y un vector director d~ su ecuación es


~
~r = ~a + λd.

donde λ es un número real cuya variación va entregando los distintos puntos de la recta.

152
Ecuación paramétrica y cartesiana de la recta en 2D

Considere una recta que pasa por el punto P = (a, b), con vector dirección d~ = (d1 , d2 ). Su
~ lo que puede ser escrito como (x, y) = (a, b) + λ(d1 , d2 ), igualando
ecuación vectorial es ~r = p~ + λd,
componentes llegamos a la ecuación paramétrica x = a + λd1 ; y = b + λd2 . Despejando λ e
x−a y−b
igualando, obtenemos la ecuación cartesiana = . La ecuación anterior también puede
d1 d2
d2
ser escrita como y − b = (x − a). El número m = dd21 , se llama la pendiente de la recta y es igual
d1
a la tangente del ángulo que forma la recta con el eje x.

Si la recta pasa por los puntos P (x1 , y1 ), Q(x2 , y2 ) entonces podemos considerar su posición
como P y su dirección es P~Q = ~q − p~ = (x2 − x1 , y2 − y1 ), luego su ecuación paramétrica es
x = x1 + λ(x2 − x1 ), y = y1 + λ(y2 − y1 ), su ecuación cartesiana
y 2 − y1
y − y1 = (x − x1 )
x2 − x1
y2 − y1
La pendiente es entonces m = .
x2 − x1

Ecuación paramétrica y cartesiana de la recta en 3D

Consideremos ahora una recta en el espacio. La recta puede estar determinada como antes, por
dos puntos, o por un punto y un vector dirección.

Si la recta pasa por los puntos A = (x1 , y1 , z1 ), B = (x2 , y2 , z2 ) su dirección d~ = AB ~ =


~b − ~a = (x2 − x1 , y2 − y1 , z2 − z1 ), su ecuación vectorial es ~r = ~a + λd.
~ Reemplazando obtenemos
(x, y, z) = (x1 , y1 , z1 ) + λ(x2 − x1 , y2 − y1 , z2 − z1 ). Igualando componentes obtenemos la ecuación
en la forma paramétrica

x = x1 + λ(x2 − x1 )
y = y1 + λ(y2 − y1 )
z = z1 + λ(z2 − z1 )

153
Si despejamos λ e igualamos obtenemos su ecuación cartesiana.
x − x1 y − y1 z − z1
= = .
x2 − x1 y2 − y 1 z2 − z1
Note que la triple igualdad anterior es un sistema de dos ecuaciones y que puede obtener la
ecuación paramétrica a partir de esta igualando a λ y despejando cada variable.

Si la dirección es d~ = (d1 , d2 , d3 ) y pasa por el punto A(x1 , y1 , z1 ) sus ecuaciones paramétrica


y vectorial son

x = x1 + λd1
y = y1 + λd2
z = z1 + λd3
Si despejamos λ e igualamos, obtenemos su ecuación cartesiana.
x − x1 y − y1 z − z1
= = .
d1 d2 d3
En ambas ecuaciones, dando valores al parámetro real λ, obtenemos los diferentes puntos de la
recta.

Ejemplo 5.5.1. Considere el triángulo de vértices A = (4, 0, 0), B = (0, 2, 0) y C = (0, 0, 2). Deter-
minar las coordenadas de los puntos medios de sus lados, la ecuación de dos de sus transversales
de gravedad y de su centro de gravedad.

154
~
~ = ~a+
Los puntos medios de los lados tienen coordenadas m b
2 = (2, 1, 0), por lo que M = (2, 1, 0).
Del mismo modo N = (0, 1, 1) y P = (2, 0, 1). Vamos ahora por las transversales de gravedad.

~ , como CM
Para la recta CM, ~r = ~c + αCM ~ =m
~ − ~c = (2, 1, −2) la ecuación paramétrica de
esta recta es

x = 0 + 2α
y = 0+α
z = 2 − 2α

Para la recta BP, ~r = ~b + β BP ~ = p~ − ~b = (2, −2, 1) la ecuación paramétrica de esta


~ , como BP
recta es

x = 0 + 2β
y = 2 − 2β
z = 0+β
Para determinar el punto de corte debemos resolver el sistema sobredeterminado

2α = 2β
α = 2 − 2β
2 − 2α = β
2
De las dos primeras ecuaciones obtenemos α = β = 3 que verifica la tercera ecuación, luego las
rectas anteriores se cortan en el punto

155
2 2 2 4 2 2
G = (0 + 2 ∗ , 2 − 2 ∗ , 0 + ) = ( , , ).
3 3 3 3 3 3
~a + ~b + ~c
El lector puede confirmar que este punto coincide con ~g = .
3
El ejercicio anterior sugiere varias preguntas interesantes ¿Cuánto mide la transversal AN ?
¿Qué ángulo forman las rectas AN y BP ? Las respuestas a estas y otras preguntas se desarrollan
en la sección siguiente.

Ejercicios

1. Dos vértices de un triángulo son A(1, −3) y B(5, 1). Encontrar la ecuación del lugar geométri-
co del tercer vértice C(x, y), si la pendiente del lado CA es siempre el triple de la pendiente
del lado BC.
2. Calcular todos los valores de los parámetros (α, β) que hacen que los puntos (1, 4, −1), (2, 3, 5)
y (α, β, 1) sean colineales.
3. Hallar las ecuaciones vectoriales y paramétricas de las siguientes rectas:

a) La recta que pasa por los puntos de coordenadas A(1, 2, 1) y B(3, −1, 2).
b) La recta que es paralela al vector d~ = (1, 2, −1) y que pasa por el punto A(3, −2, 4).
c) La recta que pasa por P (1, 0, 3) y que es paralela a la recta de ecuaciones paramétricas
x = −1 + 2t, y = 1, z = 2 − 5t.
d ) La recta que pasa por P (1, 0, 1) y que corta a la recta de ecuación vectorial ~r =
(1, 2, −1)+λ(2, 1, −2), en los puntos situados a dos unidades de distancia de P (1, 2, −1).

4. Encontrar el punto de intersección, si este existe, de los pares de rectas:

x−4 y−6 z−1


x = 3 + t, y = 1 − 2t, z = 3 + 3t y = = .
2 3 1

5. Encontrar las ecuaciones cartesiana, vectorial y paramétrica de la recta que pasa por los
puntos A(1, −2) y B(−1, −1).

6. Utilizando pendientes, demostrar que las diagonales de un rombo son perpendiculares.


7. Dos vértices opuestos de un rombo son A(1, 2) y C(3, 5). Determinar la ecuación de sus dos
diagonales.
8. Calcular las coordenadas del ortocentro del triángulo de vértices A(0, 0), B(a, 0), C(b, c).

156
5.6. Distancia entre puntos y norma de un vector
Consideremos un vector ~v cuyas coordenadas en la base canónica de R3 son (α, β, γ), como en
la figura,

La norma del vector ~v , que es la distancia d en la primera gráfica, puede calcularse utilizando
2 2 2 2 2 2
el
p teorema de pitágoras puesto que d = x + γ , y como x = α + β , se deduce que d =
2 2 2
α + β + γ . Lo anterior nos conduce a la siguiente definición.

Definición 5.6.1. Sea ~v = (α, β, γ) ∈ R3 se define su norma por


p
k~v k = α2 + β 2 + γ 2 .

Es claro que si el vector está en R2 , su norma se define de manera análoga.

Definición 5.6.2. Sea ~v = (α, β) ∈ R2 se define su norma por


p
k~v k = α2 + β 2 .

Disponer de una norma en un espacio nos permite determinar la distancia entre dos de sus
elementos.

Definición 5.6.3. Sean A = (a1 , a2 ) y B = (b1 , b2 ) dos puntos del plano. Definimos la distancia
entre ellos por
~ = k~b − ~ak = (b1 − a1 )2 + (b2 − a2 )2 .
p
d(A, B) = kABk

Ejemplo 5.6.4. Determinar la medida de la diagonal CD del hexágono de vértices A = (0, 0), B =
(1, 0), C = (3, 1), D = (2, 2), E = (0, λ) y F = (−1, 1). ¿Para qué valores de λ el hexágono se
mantiene convexo?

157
Definición 5.6.5. Sean A = (a1 , a2 , a3 ) y B = (b1 , b2 , b3 ) dos puntos del espacio. Definimos la
distancia entre ellos por
~ = k~b − ~ak = (b1 − a1 )2 + (b2 − a2 )2 + (b3 − a3 )2 .
p
d(A, B) = kABk

5.7. Ángulo entre vectores y producto punto


Consideremos ahora el problema de calcular el ángulo que forman dos vectores dados. Por ejem-
plo, podemos preguntarnos en el caso del ejemplo 1.2.1, ¿Cuál es el ángulo entre las transversales
de gravedad BP y CM ? Para desarrollar una fórmula consideremos dos vectores ~a y ~b como en la
figura.

Aplicamos el teorema del coseno al triángulo y obtenemos

~ 2 = k~ak2 + k~bk2 − 2k~akk~bkcos(α).


kABk
Consideremos el problema en R3 , entonces ~a = (a1 , a2 , a3 ) y ~b = (b1 , b2 , b3 ), note que en R2 el
razonamiento es análogo pero los vectores tienen una coordenada menos. Desarrollando la expresión
anterior obtenemos después de simplificar

k~akk~bkcos(α) = a1 b1 + a2 b2 + a3 b3 ,
La expresión de la derecha tiene un nombre, se llama producto punto entre los dos vectores, y
como se explicará en lo que sigue mide de alguna forma el ángulo entre los vectores.

Definimos entonces el producto punto:

Definición 5.7.1. Considere los vectores ~a y ~b, su producto punto o interno se denota ~a · ~b y se
define por

~a · ~b = a1 b1 + a2 b2 + a3 b3 .

158
Note que si ~a = (a1 , a2 ) y ~b = (b1 , b2 ) son vectores de R2 , su producto punto tiene sólo una
componente menos, esto es:

~a · ~b = a1 b1 + a2 b2 .

Reemplazando obtenemos,

~a · ~b
cos(α) =
k~akk~bk.
Hemos probado entonces el siguiente teorema:

Teorema 5.7.0.61. Ángulo entre dos vectores. Dados dos vectores no nulos ~a y ~b del plano o del
espacio, el ángulo α que ellos forman puede ser calculado resolviendo la ecuación trigonométrica

~a · ~b
cos(α) =
k~akk~bk

Para el lector puede ser familiar la siguiente definición utilizada comunmente en Fı́sica: ~a · ~b =
k~akk~bk cos(α), donde α es el ángulo formado por los vectores.

Ejemplo 5.7.2. Calcular el ángulo entre las transversales de gravedad CM y BP en el triángulo


del ejemplo 1.2.1

~ =m
De los cálculos anteriores CM ~ = p~ − ~b = (2, −2, 1) por lo que si α
~ − ~c = (2, 1, −2) y BP
es el ángulo entre estos vectores

~ · BP
CM ~
cos(α) =
~ kkBP
kCM ~ k

CM~ · BP
~ = 2 · 2 + 1 · (−2) + (−2) · 1 = 0, kCM
~ k = kBP
~ k = 3, por lo que
0 π
cos(α) = 9 = 0 con lo que α = 2 esto es: ambos vectores son perpendiculares y el ángulo entre
las transversales de gravedad es 90◦ .

El caso anterior es un ejemplo de un resultado general.

Teorema 5.7.0.62. Sean ~a y ~b dos vectores no nulos de R2 o R3 , ~a es perpendicular al ~b si y sólo si


~a · ~b = 0.

La demostración es sencilla y se deja al lector como ejercicio.

159
Teorema 5.7.0.63. Considere las rectas del plano y = m1 x + n1 e y = m2 x + n2 . Estas rectas son
paralelas si y sólo si m1 = m2 , y son perpendiculares sólo si m1 m2 = −1.

Para una idea de la prueba note que la forma paramétrica de ambas ecuaciones es (x, y) =
x(1, m1 ) + (0, n1 ) y (x, y) = x(1, m2 ) + (0, n2 ) con lo que sus respectivos vectores directores son
d~1 = (1, m1 ) y d~2 = (1, m2 ). En el caso del paralelismo d~1 = λd~2 conduce a m1 = m2 . En el caso
de la perpendicularidad d~1 · d~2 = 0 que se traduce en 1 + m1 m2 = 0 o m1 m2 = −1.

Contar en un espacio de vectores con un producto punto enriquece sumamente la estructura


del espacio, porque no solamente se pueden calcular distancias utilizando la fórmula (1) sino que
se tiene el concepto de perpendicularidad, de suma importancia para proyectar, lo que a su vez
está relacionado con minimizar.

Teorema 5.7.0.64. Propiedades del producto punto. Sean ~a, ~b y ~c, vectores del plano o el espacio
y α una constante real.

1. ~a · ~a = k~ak2

2. ~a · ~b = ~b · ~a
3. ~a · (~b + ~c) = ~a · ~b + ~a · ~c.

4. α(~a · ~b) = (α~a) · ~b = ~a · (α~b).


1
5. ~a · ~b = (k~ak2 + k~bk2 − k~b − ~ak2 )
2

Las propiedades anteriores complementan el álgebra vectorial de adiciones y ponderaciones con


el producto punto. Ası́, en una igualdad vectorial, podemos hacer producto punto con algún vector
adecuado y obtener interesantes relaciones. Explicaremos lo que acabamos de mencionar con un
interesante ejemplo.

Ejemplo 5.7.3. Probar vectorialmente que las alturas de un triángulo se cortan en un único punto,
su ortocentro.

Consideremos el triángulo ABC de la figura en el que se han trazado las alturas CQ y AP .

160
Denotemos por H el punto de intersección entre estas alturas y consideremos la recta que une
los puntos B y H. Debemos probar que esta recta es perpendicular al lado AC.
~ · AB
Tenemos entonces las siguientes hipótesis CH ~ = 0, AH~ · BC
~ = 0 y debemos probar que
~ ~
BH · AC = 0.

De las dos primeras igualdades obtenemos (~h − ~c) · (~b − ~a) = 0 que es equivalente a

~h · ~b − ~h · ~a − ~c · ~b + ~c · ~a = 0,

también (~h − ~a) · (~c − ~b) = 0 que es equivalente a

~h · ~c − ~h · ~b − ~a · ~c + ~a · ~b = 0,

y note que la tesis es equivalente a

~h · ~c − ~h · ~a − ~b · ~c + ~a · ~b = 0,

igualdad que resulta de sumar las dos igualdades iniciales.

Una de las aplicaciones más importantes del producto punto tiene relación con las proyecciones.

5.8. Proyecciones
Dados vectores ~a y ~b no nulos del plano o el espacio, si ponderamos el vector ~b por un escalar λ,
tenemos un vector p~ = λ~b en la dirección de ~b, como en la figura. El vector proyección del ~a sobre
~b es, entre todos estos vectores ası́ formados, el que verifica que el triángulo OP A es rectángulo en
P.

De la condición P~A · ~b = 0 se tiene que (~a − p~) · ~b = 0. Cambiamos en la expresión anterior p~


~a · ~b ~a · ~b ~
por λ~b y desarrollamos el producto punto para obtener λ = , entonces p~ = b.
k~bk2 k~bk2

Luego llegamos a la siguiente definición:

161
Definición 5.8.1. Proyección de un vector sobre otro. Sean ~b un vector fijo y ~x un vector cualquiera
del plano o del espacio, definimos la proyección de ~x sobre el vector ~b y anotamos P~b (~x) por

~x · ~b ~
P~b (~x) = b
k~bk2

En dimensión tres, la transformación de proyección ası́ definida lleva todos los vectores de R3
a una recta que contiene al vector ~b, esto es: transforma un espacio de dimensión tres en uno de
dimensión uno, como lo muestra la siguiente gráfica en tres dimensiones. Note que de la definición
anterior es inmediato que cualquier vector contenido en un plano por el origen y perpendicular al
vector ~b es llevado por la transformación al vector cero y que la proyección del vector ~b es el mismo
vector ~b.

Ejercicios

1. Encontrar e identificar el lugar geométrico de los puntos que equidistan de A = (−1, 2) y


B = (2, −1).
Encontrar e identificar el lugar geométrico de los puntos del espacio que equidistan de A =
(−1, 2, 0) y B = (2, −1, 0).
2. Calcular las longitudes de las diagonales del cubo de vértices (0, 0, 0), (a, 0, 0), (a, b, 0),
(0, b, 0), (0, 0, c), (a, 0, c), (a, b, c), (0, b, c). Representar gráficamente.
3. Demostrar que las distancias entre la diagonal de un paralelepı́pedo recto, cuyas aristas miden
a, b, c y las aristas que no la cortan son

bc ac ab
√ , √ , √
b2 + c2 a2 + c2 a2 + b2
4. Encuentre todos los puntos de la recta que pasa por (−1, 2, 1) y (1, 1, 3) cuya distancia al
punto (1, 4, 1) es 10.

162
x+2 y z−1 x−3 y−1 z−7
5. Determinar el valor de c para que las rectas = = y = =
2 −3 4 c 4 2
sean concurrentes. Calcular el ángulo entre ellas.
6. Los puntos A(~a), B(~b), C(~c) forman un triángulo con k~ak = k~bk = k~ck. Demostrar que el
vector posición del ortocentro está dado por ~h = ~a + ~b + ~c.
7. Demostrar vectorialmente que si un tetraedro tiene dos pares de aristas opuestas mutuamente
ortogonales entonces el tercer par también lo es.
8. Pruebe las siguientes propiedades de la proyección ortogonal:
a) P~b (~b) = ~b.
b) Si ~a es cualquier vector perpendicular al vector ~b entonces P~b (~a) = ~0.
c) P~b (λ~x) = λP~b (~x).
d ) P~b (~x + ~y ) = P~b (~x) + P~b (~y ).

9. Sean A y B dos puntos con vectores asociados ~a y ~b respectivamente. Pruebe que k~a + ~bk ≤
k~ak + k~bk. Interprete gráficamente.
10. Puebe la ley del paralelógramo, esto es
k~a + ~bk2 + k~a − ~bk2 = 2(k~ak2 + k~bk2 ).

11. puebe que k~a + ~bk = k~a − ~bk si y sólo si ~a es perpendicular a ~b.

5.9. Producto cruz y sus propiedades


Consideremos el siguiente problema: Dados dos vectores no colineales ~a y ~b en el espacio, nos
interesa obtener un tercer vector ~c que cumpla las siguientes condiciones:
1. ~c es perpendicular a ~a.
2. ~c es perpendicular a ~b.
3. k~ck es igual al área del paralelógramo que determinan los vectores ~a y ~b.

163
Sean ~a = (a1 , a2 , a3 ) y ~b = (b1 , b2 , b3 ) y nuestro vector buscado ~c = (x, y, z). Las dos primeras
condiciones anteriores se traducen en

a1 x + a2 y + a3 z = 0
b1 x + b 2 y + b 3 z = 0
Es un sistema de dos ecuaciones con tres incógnitas, luego es de esperar que tenga más de una
solución. Para eliminar z multiplicamos por b3 la primera ecuación y la segunda por −a3 y luego
sumamos para obtener
a3 b2 − a2 b3
x(a1 b3 − a3 b1 ) + y(a2 b3 − a3 b2 ) = 0, ⇒ x =
a1 b3 − a3 b1
Para eliminar la variable x multiplicamos por b1 la primera ecuación y la segunda por −a1 y
luego sumamos para obtener
a1 b2 − a2 b1
y(a2 b1 − a1 b2 ) + z(a3 b1 − a1 b3 ) = 0 ⇒ z = .
a3 b1 − a1 b3
ahora y = y y el sistema tiene infinitas soluciones de la forma
a3 b2 − a2 b3 a1 b2 − a2 b1
(x, y, z) = ( , 1, )y
a1 b3 − a3 b1 a3 b1 − a1 b3
Luego las soluciones están en la recta

(x, y, z) = λ(a2 b3 − a3 b2 , −(a1 b3 − a3 b1 ), a1 b2 − a2 b1 ).


Debemos ahora ajustar el valor de λ para que el vector ~c cumpla la tercera condición, referente
al área del paralelógramo.

A continuación indicamos los pasos a seguir para llegar a la definición exacta del producto que
queremos. El lector interesado puede completar los detalles.

Primero el vector ~c, antes de cumplir la tercera condición, es en principio de la forma ~c =


λ(a2 b3 − a3 b2 , −(a1 b3 − a3 b1 ), a1 b2 − a2 b1 ), donde λ es un parámetro a determinar. El área del
paralelógramo que forman los vectores ~a = (a1 , a2 , a3 ) y ~b = (b1 , b2 , b3 ), está dada por A =
k~akk~bk sin(α), donde α es el ángulo que forman los vectores ~a y ~b. Luego igualar A2 con k~ck2 ,
para obtener k~ak2 k~bk2 − (~a · ~b)2 = λ2 k~ck2 . Finalmente simplificar la igualdad anterior para obtener
λ = ±1. Lo anterior tiene perfecto sentido dado que en principio es claro que hay dos vectores de
direcciones opuestas que verifican la propiedad del área. Escogiendo el valor de λ = 1 obtenemos
el vector ~c que se conoce como ~a × ~b.

Definición 5.9.1. Producto cruz.Dados los vectores ~a = (a1 , a2 , a3 ) y ~b = (b1 , b2 , b3 ), su producto


cruz se denota por ~a × ~b. y puede ser calculado como

~a × ~b = (a2 b3 − a3 b2 , −(a1 b3 − a3 b1 ), a1 b2 − a2 b1 )
que en notación de determinantes es el desarrollo de

164

ı̂ ̂ k̂

a1 a2 a3 = (a2 b3 − a3 b2 , −(a1 b3 − a3 b1 ), a1 b2 − a2 b1 ).

b1 b2 b3

Teorema 5.9.0.65. Propiedades del producto cruz.Dados los vectores ~a = (a1 , a2 , a3 ) y ~b = (b1 , b2 , b3 ),
su producto cruz verifica las siguientes propiedades:

1. ~a × ~b es perpendicular a ~a.
2. ~a × ~b es perpendicular a ~b.

3. k~a × ~bk es igual al área del paralelógramo que determinan los vectores ~a y ~b, esto es:

k~a × ~bk = k~akk~bk sin(α),


donde α es el ángulo que forman los vectores ~a y ~b.

Teorema 5.9.0.66. Identidad de Lagrange. Sean ~a y ~b vectores de R3 , se verifica la siguiente iden-


tidad.

k~ak2 k~bk2 − (~a · ~b)2 = k~a × ~bk2 .

La demostración es directa del análisis anterior.

Ejemplo 5.9.2. Determinar un tercer vector perpendicular a los vectores ~a = (1, 2, 1) y ~b =


(−1, 0, 2). Calcular además el área del paralelógramo que ellos forman.

Para la solución


ı̂
̂ k̂
~

2 1 = (4, −3, 2).
~a × b = 1
−1
0 2

Luego ~c = (4, −3, 2) cumple las condiciones pedidas y k~ck = 29 es el área del paralelógramo
que forman estos vectores.

La dirección del producto cruz puede determinarse por una regla conocida como la regla de la
mano derecha.

Teorema 5.9.0.67. Si el vector ~a × ~b se toma con la mano derecha y los dedos lo rodean desde ~a
hasta ~b recorriendo el ángulo α, entonces el pulgar apunta en la dirección de ~a × ~b.

165
Teorema 5.9.0.68. Propiedades del producto cruz. Sean ~a, ~b y ~c vectores de R3 , y α un número
real. Se verifican las siguientes propiedades:

1. ~a × ~b = −~b × ~a.
2. ~a × ~a = ~0.
3. ~a × (α~b) = (α~a) × ~b = α(~a × ~b).

4. ~a × (~b + ~c) = ~a × ~b + ~a × ~c.

5.10. El plano en R3 .
Existen distintas formas de llegar a la ecuación de un plano y todas ellas nos llevan, en último
término, a idénticos resultados. Partimos presentando la ecuación vectorial del plano.

Definición 5.10.1. Ecuación vectorial del plano. Sean ~a y ~b dos direcciones no paralelas en R3 y
P (~
p) un punto fijo del espacio. La ecuación vectorial del plano que pasa por P, y queda determinado
por las direcciones indicadas es

~r = p~ + α~a + β~b.
los parámetros α y β son números variables en los reales, cuya variación determina los distintos
puntos R(~r) del plano.

166
Ejemplo 5.10.2. Determinar la ecuación del plano que pasa por los puntos A(1, 1, 0), B(1, 0, 2) y
C(0, 1, 2).

Para resolver este problema consideremos como punto fijo del plano el punto A(1, 1, 0) y como
~ = ~b − ~a = (0, −1, 2) y AC
direcciones los vectores AB ~ = ~c − ~a = (−1, 0, 2), luego la ecuación del
plano pedido será

~ + β AC.
~r = ~a + αAB ~

(x, y, z) = (1, 1, 0) + α(0, −1, 2) + β(−1, 0, 2).


Lo anterior nos conduce al sistema

x = 1−β
y = 1−α
z = 2α + 2β.

Despejando de las dos primeras ecuaciones α y β y reemplazando en la tercera obtenemos


2x + 2y + z = 4 que es la ecuación del plano pedido. El lector puede notar que las coordenadas de
los tres puntos verifican la ecuación.

Una segunda forma de determinar la ecuación de un plano es la llamada ecuación normal del
plano. En este caso, los elementos que determinan el plano son un punto P del plano y un vector
~n normal al plano.

Si R(x, y, z) es un punto cualquiera del plano entonces Los vectores P~R y ~n son perpendiculares,
lo que conduce a la ecuación normal

(~r − p~) · ~n = 0.

En coordenadas, ~r = (x, y, z), p~ = (p1 , p2 , p3 ), y el vector normal ~n = (n1 , n2 , n3 ), la ecuación


se transforma en

n1 x + n2 y + n3 z = ~n · p~ = c,

167
donde c es una constante. Luego toda ecuación de un plano es de la forma n1 x + n2 y + n3 z = c
donde los coeficientes de x, y, z son las componentes del vector normal.

Resolvamos el ejercicio anterior con este nuevo procedimiento.

Ejemplo 5.10.3. Determinar la ecuación del plano que pasa por los puntos A(1, 1, 0), B(1, 0, 2) y
C(0, 1, 2).

Para resolver este problema consideremos como punto fijo del plano el punto A(1, 1, 0) y como
~ = ~b − ~a = (0, −1, 2) y AC
direcciones los vectores AB ~ = ~c − ~a = (−1, 0, 2), el vector normal lo
calculamos como


ı̂ ̂ k̂
~ × AC
~ = 0 −1 2 = (−2, −2, −1).

~n = AB
−1 0 2

La ecuación del plano entonces es −2x − 2y − z = c y como pasa por A(1, 1, 0) sus coordenadas
verifican la ecuación, lo que entrega c = −4. La ecuación buscada es entonces 2x + 2y + z = 4.

5.11. Distancias entre punto, recta y plano


Estamos ahora en condiciones de desarrollar fórmulas para el cálculo de distancias.

Consideremos en primer lugar un punto P (x0 , y0 ) y una recta de ecuación l : ax + by + c = 0


en R2 . La distancia del punto P a la recta l es la mı́nima distancia entre puntos de la recta y el
punto P. El punto Q que realiza esta distancia es el que se obtiene bajando la perpendicular por
P a la recta l. ¿Podrı́a el lector justificar esta afirmación? Ver figura.

Una de las formas más directas de determinar dicha distancia es la siguiente. Primero se escribe
la recta l en su forma paramétrica de modo de determinar su posición y dirección, esto es (x, y) =
x(1, − ab ) + (0, − cb ). Luego un punto de la recta es P0 (0, − cb ) y podemos considerar como dirección
d~ = (b, −a). El lector puede notar que un vector normal a la recta es ~n = (a, b). Finalmente la
distancia resultará de proyectar el vector P~0 P en la dirección de la normal ~n.

168
,

luego
P~0 P · ~n | ax0 + by0 + c |
d = kP~n (P~0 P )k =| |= √
knk a2 + b2

Teorema 5.11.0.69. Distancia de un punto a una recta en el plano. La distancia δ del punto (x0 , y0 )
del plano a la recta ax + by + c = 0, puede ser calculada como

| ax0 + by0 + c |
δ= √ .
a2 + b2

Un análisis similar permite probar

Teorema 5.11.0.70. Distancia de un punto a un plano. La distancia δ del punto (x0 , y0 , z0 ) al plano
de ecuación ax + by + cz + d = 0, puede ser calculada como

| ax0 + by0 + cz0 + d |


δ= √ .
a2 + b2 + c2

El producto cruz y el área del paralelógramo conducen a la fórmula para la distancia de un


punto a una recta del espacio.

Teorema 5.11.0.71. Distancia de un punto a una recta en el espacio. La distancia δ del punto
P (x0 , y0 , z0 ) a la recta de ecuación vectorial ~r = p~0 + λd~ está dada por la fórmula

~
kP~0 P × dk
δ= .
~
kdk

169
,

Ejercicios

1. Encuentre el punto de intersección

a) entre la recta que pasa por (1, 2, −1) y (2, 1, 0) y el plano que pasa por (1, 1, 1), (2, 1, −1)
y (3, 2, 3),
b) los tres planos x − y + z = 1, 2x + 3z = −2, x + y + z = 4,
c) del plano 2x + 3y − z + 2 = 0 con cada uno de los ejes.

2. Determine el valor de α para que los planos αx+(α−1)y+(α−2)z+5 = 0 y αx+y+αz+7 = 0,


sean perpendiculares.
3. ¿Qué valor debe tener el número β para que el ángulo formado por los vectores ı̂ + ̂ + k̂ y
βı̂ + 2̂ + β k̂ sea π4 ?
4. Considere el triángulo con vértices A(1, 1, −1), B(2, −1, 3) y C(3, 1, 1).

a) Encontrar la ecuación paramétrica del lado AC.


b) Encontrar la ecuación cartesiana del lado BC.
c) Encontrar la ecuación de la biscetriz del ángulo BAC.
d ) Calcular el valor en grados del ángulo BAC.
e) Determinar la ecuación del plano que forma el triángulo ABC.
f ) Encontrar la ecuación paramética de la recta que es normal al plano del triángulo y que
pasa por su centro de gravedad.
g) Encontrar todos los puntos D tales que el centro de gravedad del tetraedro ABCD sea
el punto G(−1, 7, 5).

5. Determine los valores de A y de D para que la recta x = 3 + 4t, y = 1 − 4t, z = −3 + t


esté contenida en el plano Ax + 2y − 4z + D = 0.
6. Encuentre el punto del plano 3x − 2y + z + 1 = 0 que esté más cercano al punto (1, 1, 3).

170
x+1 y−3 z−1
7. Encuentre el punto de la recta = = , que está más cercano al punto
2 1 3
(1, 1, 1).
8. Dos caras de un cubo están en los planos 2x − 2y + z − 1 = 0 y 2x − 2y + z + 5 = 0. Calcular
el volumen del cubo.
9. Encontrar las ecuaciones de todos los planos que pasan por los puntos (0, 4, −3) y (6, −4, 3),
que no pasan por el origen y que cortan sobre los ejes intersecciones cuya suma es cero.
10. Determinar la ecuación del plano que pasa por la recta de intersección de los planos 5x −
2y − z − 3 = 0 y x + 3y − 2z + 5 = 0 y que es paralelo al vector 7ı̂ + 9̂ + 17k̂.
11. Determinar la proyección perpendicular del punto P (3, −1, 2) sobre el plano x + y − z + 4 = 0,
y las coordenadas del punto P 0 simétrico del punto P respecto del plano.

12. Demuestre que si un plano determina sobre los ejes segmentos a, b, c y dista p del origen,
entonces

1 1 1 1
2
+ 2 + 2 = 2.
a b c p

13. Si los vértices de un tetraedro son (a − b, a − c, a − d), (b − c, b − d, b − a), (c − d, c − a, c −


b), (d − a, d − b, d − c), pruebe que las rectas que unen los puntos medios de aristas opuestas
pasan por el origen.
14. Demostrar que los tres planos x = cy + bz, y = az + cx, z = bx + ay son coaxiales, esto es:
su intersección es una recta si y sólo si a2 + b2 + c2 + 2abc = 1.

5.12. Propiedades geométricas y vectores


Los vectores son una herramienta muy poderosa para demostrar distintas propiedades geométri-
cas, de hecho la independencia lineal implica trivialmente el teorema de Tales. Presentamos a
continuación algunos de estos hechos.

Teorema 5.12.0.72. Independencia Lineal. Sean ~a y ~b dos vectores no colineales distintos de cero.

α~a + β~b = ~0 ⇒ α = β = 0.

171
Realizamos la demostración por reducción al absurdo. Supongamos que alguno de los dos α o
β es distinto de cero. Sin perder generalidad supongamos que α 6= 0. Como α~a = −β~b entonces
β
~a = − ~b por lo que los vectores ~a y ~b son colineales. Esta contradicción prueba el teorema.
α

Teorema 5.12.0.73. Independencia Lineal (3D). Sean ~a, ~b y ~c tres vectores no colineales y no
coplanares distintos de cero.
α~a + β~b + γ~c = ~0 ⇒ α = β = γ = 0.

Ejemplo 5.12.1. Demostrar el teorema de Tales usando vectores.

Supongamos que en el triángulo ABC la recta P Q es paralela a la base AB, que P divide al
lado AC en razón λ y Q divide a BC en razón µ, esto es

~ k
kAP ~
kBQk
= λ, = µ,
kP~Ck ~
kQCk
~a + λ~c ~b + µ~c
De lo anterior p~ = y ~q = . Como las rectas P Q y AB son paralelas, los vectores
1+λ 1+µ
P~Q y AB
~ tienen igual dirección y sentido, luego

P~Q = αAB.
~

172
Desarrollando la igualdad anterior,

~b + µ~c ~a + λ~c
~q − p~ = α(~b − ~a) ⇒ − = α(~b − ~a).
1+µ 1+λ
Colocamos ahora el origen en el punto C con lo que ~c = ~0 y la igualdad anterior queda como
1 1
( − α)~b = ( − α)~a.
1+µ 1+λ
de donde por independencia lineal se deduce que
1 1
= = α ⇒ λ = µ.
1+µ 1+λ

Ejemplo 5.12.2. En un triángulo ABC los puntos M y N dividen a los segmentos BC y AC res-
pectivamente en las razones 3 : 2 y 1 : 3. Determinar el vector posición del punto de corte D de
las rectas AM y CN en términos de los vectores posición de los vértices del triángulo y la razón
en que el punto D divide a los segmentos ası́ determinados.

Consideremos un origen O como se muestra en la figura. Esto determina inmediatamente vec-


tores como el vector m~ y muchos otros como los vectores ~a, ~b, ... que no dibujaremos para no
entorpecer el dibujo.

2~b + 3~c 3~a + ~b


Por lo anterior m
~ = , también ~n = . El punto D se determina al intersectar las
5 4
rectas AM y CN , por lo que escribiremos sus ecuaciones.

Para la recta CN : ~r = ~c + αCN~ = ~c + α(~n − ~c). Reemplazando el valor de ~n, obtenemos la


ecuación vectorial de la recta CN
3α α
~r = ~a + ~b + (1 − α)~c.
4 4

173
~ = ~a + β(m
Para la recta AM : ~r = ~a + β AM ~ − ~a). Reemplazando el valor de m,
~ obtenemos la
ecuación vectorial de la recta AM
2β ~ 3β
~r = (1 − β)~a + b+ ~c.
5 5
Como el punto D está en ambas rectas, el vector asociado a él debe verificar
3α α 2β 3β
d~ = ~a + ~b + (1 − α)~c = (1 − β)~a + ~b + ~c
4 4 5 5
Estamos a punto de encontrar el vector d. ~ Para ello colocaremos el origen en un punto que
simplifique lo más posible la expresión anterior, digamos en A. Como O = A ~a = ~0.

Luego tenemos que


α 2β ~ 3β
d~ = ~b + (1 − α)~c = b+ ~c,
4 5 5
de donde
α 2β ~ 3β
( − )b + (1 − α − )~c = ~0
4 5 5
Como los vectores ~b y ~c no son colineales, por independencia lineal lo anterior implica que α y
β verifican el sistema

− 2β
α
4 5 = 0
1 − α − 3β
5 = 0
8 5
de donde α = 11 yβ= 11 luego
6 2 3
d~ = ~a + ~b + ~c
11 11 11
AD
Para las razones, si DM ~
= λ entonces d = ~
a+λm
~
y como de la ecuación inicial d~ = ~a +β(m−~
~ a) =
1+λ
6 5 6~
a+5m
~ AD 5
11 ~
a+ 11 m
~ = 11 entonces DM = 6. Cálculos similares, que invitamos a completar al lector,
CD
prueban que DN = 38 .

174
Ejercicios

1. En el triángulo ABC, el punto M divide al segmento BC en la razón 1 : 3 y S divide al


segmento AB en la razón 1 : 3. P es el punto de intersección de AM y CS. Determinar la
razón en que P divide al segmento AM y a BQ donde Q es el punto de intersección de BP
con AC.

2. Demostrar que si por un punto de uno de los lados de un triángulo se traza una paralela a
otro de los lados, se forman segmentos proporcionales.
3. En el lado AD y en la diagonal AC del paralelógramo ABCD se han tomado los puntos M
y N de forma que AM = 15 AD y AN = 61 AC. Demuestre que los puntos B, N y M son
colineales. ¿En que razón divide el punto N al segmento M B?
4. Demostrar que si se traza una paralela a la base de un triángulo, la recta que une al vértice
con la intersección de las diagonales del trapecio ası́ formado, dimidia a la base del triángulo.
5. Demostrar que las diagonales de un paralelepı́pedo se intersectan en un único punto que las
dimidia.

175
Capı́tulo 6

Secciones cónicas

En este capı́tulo estudiamos las secciones cónicas utilizando sus ecuaciones centradas. Se dis-
cuten los elementos principales de estas curvas y se introduce el concepto de excentricidad de una
cónica.

6.1. La circunferencia
Definición 6.1.1. Por circunferencia entendemos el Lugar Geométrico de todos los puntos del plano
que equidistan de un mismo punto llamado centro.

Supongamos que consideramos, como en la figura, una circunferencia de centro O(h, k) y radio
R. Si imponemos la condición descrita al comienzo y consideramos un punto P (x, y) cualquiera en
la circunferencia
p
(x − h)2 + (y − k)2 = R,
elevando al cuadrado, obtenemos la forma canónica de la ecuación de la circunferencia.

(x − h)2 + (y − k)2 = R2 ,

176
Note que si el centro es el origen y el radio es uno, tenemos la conocida circunferencia go-
niométrica tan útil en trigonometrı́a.
x2 + y 2 = 1.
Ejemplo 6.1.2. Determinar la ecuación de la circunferencia con centro en el punto O(1, 2) y que
pasa por el punto P (1, 0).

Respuesta:

~ k = 2, luego la forma canónica


Como tenemos su centro, calculamos su radio como R = kOP
de la ecuación de la circunferencia es

(x − 1)2 + (y − 2)2 = 4.

Si desarrollamos la ecuación anterior, obtenemos x2 + y 2 − 2x − 4y + 1 = 0. En general al


desarrollar (x − h)2 + (y − k)2 = R2 obtenemos una ecuación de la forma x2 + y 2 − 2hx − 2ky +
h2 + k 2 − R2 = 0, esto es una ecuación como

(∗) x2 + y 2 + Dx + Ey + F = 0,

los cálculos anteriores muestran que una circunferencia siempre está representada por una ecuación
como (∗), la que se conoce como forma general de la ecuación de una circunferencia.

Un pregunta interesante es ¿para qué valores de los parámetros D, E, F la ecuación (∗) repre-
senta realmente una circunferencia?
Ejemplo 6.1.3. ¿Qué representa la ecuación x2 + y 2 − 6x − 4y − 12 = 0?

Respuesta:

Debemos completar cuadrados para tratar de llegar a la forma canónica.

x2 + y 2 − 6x − 4y − 12 = 0 ⇔ x2 − 6x + 9 + y 2 − 4y + 4 = 12 + 9 + 4 ⇔ (x − 3)2 + (y − 2)2 = 25

luego es una circunferencia de centro (3, 2) y radio R = 5.

Para responder a la pregunta anterior completamos cuadrados como antes pero ahora en la
ecuación x2 + y 2 + Dx + Ey + F = 0.

D 2 E D2 E2
x2 + y 2 + Dx + Ey + F = 0 ⇔ (x + ) + (y + )2 = + − F,
2 2 4 4
El siguiente teorema resume los resultados obtenidos.

Teorema 6.1.0.74. La ecuación x2 + y 2 + Dx + Ey + F = 0, representa


r
D2 E2
1. una circunferencia de centro (−D/2, −E/2) y radio R = + − F si D2 +E 2 −4F > 0.
4 4
2. el punto (−D/2, −E/2) si D2 + E 2 − 4F = 0.

177
3. ningún lugar geométrico real si D2 + E 2 − 4F < 0.

Ejemplo 6.1.4. Encontrar la ecuación de la recta tangente a la circunferencia x2 + y 2 − 2x + 2y = 0


en el origen.

Respuesta:

Note que la circunferencia anterior pasa por el origen, porque al reemplazar x = 0, y = 0 se


verifica la ecuación. Calculamos entonces su centro y su radio completando cuadrados.

x2 + y 2 − 2x + 2y = 0 ⇔ (x − 1)2 + (y + 1)2 = 2,

luego tiene su centro en el punto (1, −1) y radio 2.

−1 − 0
La pendiente entre los puntos (0, 0) y (1, −1) es m1 = = −1 luego la pendiente de la
1−0
recta perpendicular es m2 = 1. Finalmente la ecuación de la tangente buscada es y − 0 = 1(x − 0).
o bien y = x.

Ejemplo 6.1.5. Encontrar la ecuación de las tangentes trazadas desde el punto (2, 2) a la circun-
ferencia de ecuación x2 + y 2 − 2x + 2y = 0.

Respuesta:

Del ejercicio anterior sabemos que el centro y el radio de la circunferencia son (1, −1) y R = 2.
Geométricamente es claro que desde un punto exterior a una circunferencia existen dos rectas
tangentes.Describimos ahora una de las formas de determinarlas.

La ecuación de una recta cualquiera que pasa por el punto (2, 2) con pendiente m es y − 2 =
m(x − 2), o bien mx − y + 2 − 2m = 0. La recta tangente debe cumplir que la distancia del centro
de la circunferencia a la recta sea igual al radio. Lo que nos conduce a

| m + 1 + 2 − 2m | √
√ = 2,
m2 + 1

178
esta ecuación con valor absoluto es equivalente a
√ p
| 3 − m |= 2 m2 + 1.

Elevando al cuadrado llegamos a la ecuación cuadrática m2 +6m−7 = 0 que tiene por soluciones
m = 1 o m = 7. Luego las ecuaciones de las rectas tangentes son y = x e y = −7x + 16.

Ejercicios

1. Hallar la ecuación de la circunferencia que cumple en cada caso las condiciones indicadas:

a) Tiene como diámetro el trazo que une los puntos A(1, 3) y B(2, 4).
b) Pasa por los puntos (2, 3), (3, 6) y es tangente a la recta de ecuación 2x + y − 2 = 0.
c) Pasa por el punto (2, 6) y tiene su centro en el punto (−1, 2).
d ) El centro es el punto (1, −1) y la recta de ecuación 5x − 12y + 9 = 0 es tangente a la
circunferencia.
e) Pasa por (4, 3), (3, −3) y (−1, 2).
f ) Pasa por el punto (2, 1) y es tangente a las rectas 2x + y − 5 = 0 y 2x + y + 15 = 0.

2. Calcular la máxima y la mı́nima distancias desde el punto (10, 7) a la circunferencia de


ecuación

x2 + y 2 − 6x − 4y − 3 = 0.

3. Encontrar la ecuación de la circunferencia que pasa por los puntos de intersección de las
circunferencias x2 + y 2 − 6x − 4y − 3 = 0, x2 + y 2 − 2 = 0 y es tangente a la recta de ecuación
x + 3y − 14 = 0.
4. Determinar la ecuación de la circunferencia que pasa por los puntos de intersección de las
circunferencias x2 + y 2 − 8x − 4y + 11 = 0, x2 + y 2 − 4x + 4y − 8 = 0. y tiene su centro en la
circunferencia de ecuación x2 + y 2 + 2x + 10y + 22 = 0.

179
5. Dadas las rectas de ecuaciones y = x + 9, x + 2y − 24 = 0, encontrar la ecuación de la recta
que pasa por la intersección de las anteriores y determina

en la circunferencia de ecuación
x2 + y 2 − 4x + 4y + 7 = 0 una cuerda de longitud 23 .
6. Hallar la ecuación de la circunferencia que tiene su centro en la recta de ecuación y = −2x
y que es tangente a las rectas de ecuaciones: 4x − 3y + 10 = 0, 4x − 3y − 30 = 0.
7. Calcular los valores de c para los que la recta de ecuación 4x + 3y = c es tangente a la
circunferencia de centro (2, 1) y radio unitario.

8. Dados los puntos A(2, 2) y B(4, −2) hallar en el eje x un punto C de modo que el ángulo
ACB sea recto.
9. Demostrar que las tangentes comunes a las circunferencias de ecuaciones x2 + y 2 + 2x = 0 y
x2 + y 2 − 6x = 0 forman un triángulo equilátero.

10. Hallar las ecuaciones de las rectas tangentes a la circunferencia de ecuación x2 + y 2 + 2x −


8y + 12 = 0 que pasan por el punto (−2, 7).
11. Hallar en la circunferencia de ecuación 16(x2 + y 2 ) + 48x − 8y − 43 = 0, el punto P más
próximo a la recta de ecuación 8x − 4y + 73 = 0 y calcular la distancia de este punto P a la
recta mencionada.
12. Encontrar la ecuación del lugar geométrico de los puntos medios de las cuerdas determinadas
por las secantes trazadas desde el punto P0 (x0 , y0 ) a la circunferencia de ecuación x2 + y 2 +
Dx + Ey + F = 0.
13. Dada la circunferencia de ecuación x2 + y 2 − 2ax − 3a2 = 0, Se trazan O(h, k) circunferencias
cuyos perı́metros quedan dimidiados por la circunferencia dada. Determinar el LG de los
centros de tales circunferencias.
14. Demostrar que el lugar geométrico de un punto del plano que se mueve de modo que la
suma de los cuadrados de sus distancias a los cuatro lados del cuadrado es constante, es una
circunferencia con centro en el centro del cuadrado.
15. Demostrar que el lugar geométrico de los puntos medios de las cuerdas de una circunferencia
que pasan por un punto fijo es una circunferencia que tiene por diámetro al trazo que une el
centro de la circunferencia dada con el punto dado.
16. Hallar el lugar geométrico de los puntos, cuya suma de los cuadrados de sus distancias a los
puntos (−3, 0) y (3, 0) es igual a 50.
17. Desde cada punto de la circunferencia x2 + y 2 + 4x + 4y − 8 = 0 se traza una perpendicular al
diámetro paralelo al eje x (y sólo hasta el diámetro). Hallar e identificar el lugar geométrico
de los puntos medios de estas perpendiculares.
18. Determinar el lugar geométrico de los puntos P (x, y) del plano que se encuentran a la misma
distancia del punto (4, 0) y de la circunferencia x2 + y 2 = 4.

180
6.2. La Elipse
Definición 6.2.1. Una elipse es el Lugar geométrico de todos los puntos del plano cuya suma de
distancias a dos puntos fijos, llamados focos es constante, la que suele denotarse por 2a.

La figura muestra la construcción geométrica de los puntos de una elipse de focos F1 y F2 en la


que se traza una circunferencia con centro en uno de los focos y de radio igual a la suma constante
2a. El punto Q es variable en la circunferencia y a medida que este se mueve, el punto P genera la
elipse.

El eje que pasa por los focos de la elipse se llama eje focal. Como en este momento no hemos
fijado el sistema de ejes coordenados, tenemos libertad para hacerlo. Para una descripción más
sencilla de la ecuación de la elipse haremos coincidir el eje x con el eje focal y el eje y será la
simetral del segmento F1 F2 .

En resumen, tenemos ahora la suma constante 2a, donde a es un real positivo, las coordenadas
de los focos son F1 (−c, 0) y F2 (c, 0). La condición del Lugar Geométrico conduce a

kP~F1 k + kP~F2 k = 2a

181
Utilizando las fórmulas de distancia, lo anterior es equivalente a
p p
(x + c)2 + y 2 + (x − c)2 + y 2 = 2a
Para obtener una ecuación sin radicales la escribimos como
p p
(x + c)2 + y 2 = 2a − (x − c)2 + y 2
y elevamos al cuadrado ambos lados de la ecuación anterior, simplificando y elevando nuevamente
al cuadrado, invitamos al lector a completar los detalles, obtenemos

(a2 − c2 )x2 + a2 y 2 = a2 (a2 − c2 )


Dado que por la desigualdad triangular a > c podemos introducir la constante positiva b que
verifica b2 = a2 − c2 y obtenemos finalmente

x2 y2
+ =1
a2 b2
Damos a continuación la interpretación geométrica de la ecuación anterior.

a es el semieje mayor y b el semieje menor. Existe un número asociado a la forma de la elipse


llamada su excentricidad e = ac que es un número entre cero y uno.

Si c es pequeño comparado con a, la excentricidad e está cerca de cero, lo que indica que la
elipse se parece a una circunferencia. Si ahora c y a son parecidos, caso en que e está cerca de uno,
los focos están muy cerca del borde y la elipse es más achatada.

182
Supongamos ahora que fijamos el sistema de coordenadas de modo que la ecuación de la elipse
depende fundamentalmente de la posición de los focos y del valor de la suma constante 2a.

Alalizaremos primero el caso en que la elipse está centrada y los focos están en el eje y. Es decir
F1 (0, c) y F2 (0, −c).

El lector puede seguir el razonamiento del caso inicial que lo conducirá a la ecuación

x2 y2
+ = 1,
b2 a2
donde a > b y a2 = b2 + c2 . La excentricidad sigue siendo e = ac .

x2 y2
Ejemplo 6.2.2. Graficar la elipse de ecuación + = 1, determinando las coordenadas de sus
9 25
focos, vértices y excentricidad.
a2 = 25, b2 = 9, a2 = b2 + c2 de donde c = 4. Los focos son F1 (0, 4) y F2 (0, −4), los vértices
V1 (3, 0), V2 (0, 5), V3 (−3, 0), V4 (0, −5). Su excentricidad e = 54 = 0,8

183
Tangente y normal a la Elipse en un punto

Por recta tangente a una elipse en un punto P de ella, entendemos una recta que pasa por P
e intersecta a la elipse sólo en el punto P.

Como no suponemos conocimientos de cálculo diferencial describiremos un método para deter-


minar la ecuación de la recta tangente. Invitamos al lector entusiasta a completar los detalles.

En primer lugar consideremos un punto P (x0 , y0 ) de la elipse de ecuación

x2 y2
2
+ 2 = 1,
a b
Una recta cualquiera de pendiente m que pasa por P tiene ecuación y − y0 = m(x − x0 ),
intersectamos esta ecuación con la elipse y obtenemos

(b2 + a2 m2 )x2 + 2a2 m(y0 − mx0 )x + a2 (y0 − mx0 )2 − a2 b2 = 0.


La ecuación anterior es una ecuación cuadrática en x que entrega los puntos de corte para
distintos valores de m. En el caso de la tangencia el discriminante de esta cuadrática debe ser cero.
Igualando a cero el discriminante obtenemos una ecuación para m.

m2 (x20 − a2 ) − 2x0 y0 m + y02 − b2 = 0.


x0 y0
Si resolvemos la cuadrática notamos que tiene una única solución m = . Finalmente
x20 − a2
reemplazamos este valor de m para obtener.

x2 y 2
Teorema 6.2.0.75. La ecuación de la recta tangente a la elipse 2 + 2 = 1, en un punto P (x0 , y0 )
a b
xx0 yy0
de ella es 2 + 2 = 1.
a b

Ejemplo 6.2.3. Determinar las ecuaciones de las rectas tangentes y normal a la elipse de ecuación
2x2 + 3y 2 = 5 en el punto (1, −1).

Respuesta:

184
La ecuación de la tangente es 2xx0 + 3yy0 = 5, cambiando x0 = 1, y0 = −1, obtenemos la
ecuación de la tangente 2x − 3y = 5. La pendiente de esta recta es m1 = 32 por lo que la pendiente
de la normal es m2 = − 32 , y su ecuación y + 1 = − 23 (x − 1).

Propiedad Focal de la Elipse

Una de las propiedades más importantes y origen de diversas aplicaciones de la elipse es la


llamada propiedad focal, esta establece lo siguiente

Teorema 6.2.0.76. La recta normal a la elipse en cualquiera de sus puntos es bisectriz del ángulo
formado por los radiovectores del punto.

Dejamos esta demostración al lector como ejercicio.

Ejercicios

1. Los semiejes de una elipse miden 3 cm. y 5 cm. Hallar la distancia entre los focos y la
excentricidad.
2. O es el centro, F es un foco y B uno de los vértices correspondientes al eje menor de una
elipse. Demostrar que la excentricidad está dada por e = cos(∠OF B).
3. Determinar las coordenadas de los vértices y focos, las longitudes de los ejes mayor y menor
y la excentricidad de la elipse correspondiente. Hacer un gráfico aproximado

a) 9x2 + 4y 2 = 36.
b) 16x2 + 25y 2 = 400.
c) x2 + 3y 2 = 6.

185
d ) 4x2 + 9y 2 = 36.

4. Demostrar que una circunferencia es un caso particular de una elipse cuya excentricidad vale
cero.
5. Una elipse, cuyos ejes miden 8 cm y 10 cm tiene su centro en el origen y sus focos sobre el
eje OY. Hallar su ecuación.

6. Demostrar que la normal a una elipse en un punto que no sea alguno de los vértices del eje
mayor, corta a este eje en un punto comprendido entre los focos.
7. Calcular el área del triángulo determinado por la tangente a la elipse 2x2 + 3y 2 = 14 en el
punto (1, 2) con los semiejes positivos.

8. Demostrar que la longitud del Latus rectum (Cuerda que pasa por el foco perpendicular al
x2 y2
eje focal), de la elipse 2 + 2 = 1, con a > b está dada por cualquiera de las expresiones
a b
2b2 p
; 2b 1 − e2 ; 2a(1 − e2 ).
a
9. La Tierra se mueve al rededor del Sol en una órbita elı́ptica, uno de cuyos focos es el Sol.
Si la máxima y la mı́nima distancias de la Tierra al Sol están en razón 29 : 30 ¿Cuál es la
excentricidad de su órbita?
10. Los ejes de una elipse que pasa por los puntos (4, 1) y (2, 2) son los ejes coordenados. Hallar
su ecuación.
x2 y2
11. Hallar la ecuación de la circunferencia tangente a la elipse 2 + 2 = 1, en los extremos de
a b
su Latus rectum.
12. Demostrar que el producto de las distancias desde una tangente cualquiera de una elipse a
sus focos es constante.
13. La normal a una elipse en un punto P corta a los ejes en Q1 y Q2 . Probar que el producto
de las distancias P Q1 · P Q2 es igual al producto de los rayos focales F1 P · F2 P.
14. La base de un triángulo es fija y el producto de las tangentes de los ángulos basales es
constante. Hallar el Lugar Geométrico del vértice.

6.3. La parábola
Definición 6.3.1. Una parábola es el lugar geométrico de todos los puntos del plano que equidistan
de una recta fija llamada directriz y de un punto también fijo llamado foco que no pertenece a la
directriz.

186
En la figura anterior se describe cómo determinar puntos de una parábola utilizando simetrales,
aparecen también el Eje Focal, que es la recta que pasa por el foco perpendicular a la directriz, y
el vértice que es el punto donde más se curva la parábola.

Como no hemos establecido un sistema coordenado, tenemos la libertad para escogerlo de


manera conveniente, ası́, escojamos como eje x el eje focal y como eje y la perpendicular al eje
focal por el vértice. Supongamos entonces que el foco tiene coordenadas F (p, 0) y la directriz en
consecuencia tiene ecuación x = −p.

La condición del lugar geométrico se traduce ahora en una ecuación que relaciona las coorde-
nadas x e y de los puntos P (x, y) de la parábola.
p
(x − p)2 + y 2 =| x + p |,

187
Elevando al cuadrado ambos lados de la ecuación anterior y simplificando obtenemos

Teorema 6.3.0.77. Cualquier parábola del plano tiene ecuación y 2 = 4px si se escoge el sistema
coordenado adecuadamente.

Supongamos ahora que fijamos el sistema coordenado y cambiamos la posición del foco y la
directriz, la ecuación de la parábola cambiará también, pero seguirá siendo cuadrática en una de
las variables.

El siguiente esquema representa todos los casos posibles en que la parábola tiene vértice en el
origen y directriz paralela a uno de los ejes coordenados.

188
Ejemplo 6.3.2. Hacer un gráfico aproximado de la parábola de ecuación y 2 = −x. Determinar las
coordenadas de su foco y la ecuación de su directriz.

Respuesta:

Comparando con y 2 = 4px, tenemos que 4p = −1 con lo que p = −1/4. El foco tiene coorde-
nadas (−1/4, 0) y la directriz es x = 1/4.

Tangente y normal a la parábola en uno de sus puntos.

El lector puede probar con un argumento similar al hecho con la elipse el siguiente resultado.

Teorema 6.3.0.78. Dada la parábola de ecuación y 2 = 4px la ecuación de la recta tangente en un


punto (x0 , y0 ) de ésta es yy0 = 2p(x + x0 ).

Teorema 6.3.0.79. Dada la parábola de ecuación y 2 = 4px la ecuación de la recta Normal en un


−y0
punto (x0 , y0 ) de ésta es y − y0 = (x − x0 ).
2p

Teorema 6.3.0.80. Dada la parábola de ecuación x2 = 4py la ecuación de la recta tangente en un


punto (x0 , y0 ) de ésta es xx0 = 2p(y + y0 ).

189
Propiedad focal de la parábola.

Teorema 6.3.0.81. La normal a la parábola en un punto cualquiera P de ésta, es bisectriz del


ángulo formado por el radio vector de P0 , el punto P y la recta que pasa por P paralela al eje de
la parábola.

En la figura anterior hay que probar que α = β, lo que claramente es equivalente a probar que
el triángulo QF P es isósceles.
−y0
Para ello note que la ecuación de la recta normal es y − y0 = (x − x0 ), para determinar
2p
el punto Q la intersectamos con y = 0 obteniendo Q(x0 + 2p, 0). Calculamos ahora las distancias
kF~Qk2 = (x0 + p)2 y kP~Qk2 = (x0 − p)2 + y02 pero como y02 = 4px0 , reemplazamos en esta última
expresión para obtener kP~Qk2 = (x0 − p)2 + 4px0 = (x0 + p)2 lo que prueba el teorema.

190
La propiedad anterior es la base de todas las aplicaciones prácticas de la parábola porque en
resumen, establece que cualquier rayo paralelo al eje de la parábola rebotará en ella para dirigirse
al foco.

Ejercicios

1. Determinar el foco y la directriz de las siguientes parb́olas haciendo un gráfico aproximado

a) y 2 = 4x.
b) y = x2 .
c) 3y 2 − 5x = 0.
d ) y = −2x2 .

2. Hallar la ecuación de la parábola sabiendo que su foco es F (2, −1) y que su directriz es la
recta de ecuación x − y = 1.
3. Dado el vértice V (6, −3) y la ecuación 3x − 5y + 1 = 0 de la directriz de la parábola se pide
encontrar las coordenadas de su foco y la ecuación de la parábola.
4. Hallar la ecuación de la circunferencia tangente a la parábola de ecuación y 2 = 4x en los dos
extremos de su Latus rectum.
5. Determinar la ecuación de la parábola cuyo eje de simetrı́a coincide con el eje x, su vértice
está en el origen y pasa por el punto P (−2, 4). Encontrar las coordenadas de su foco y la
ecuación de su directriz.
6. Determinar la longitud de la cuerda focal de la parábola x2 = 8y que es paralela a la recta
de ecuación 3x + 4y − 7 = 0.
7. Demostrar que la longitud del radio focal de cualquier punto P1 (x1 , y1 ) de la parábola y 2 =
4px es | x1 + p | .
8. Encontrar e identificar el lugar geométrico de los puntos medios de las cuerdas focales de una
parábola.
9. Demostrar que las rectas tangentes a una parábola en los extremos del Latus rectum son
perpendiculares.
10. Demostrar que cualquier tangente a una parábola, excepto la tangente en el vértice, corta
a la directriz y a la recta que contiene al Latus rectum en puntos que son equidistantes del
foco.
11. Sea N el punto en que la normal a la parábola en un punto R de ella, diferente del vértice,
corta al eje. Demostrar que la proyección ortogonal sobre el eje del segmento RN es igual al
semiparámetro. (En y 2 = 4px el parámetro es p.)
12. La tangente a una parábola en un punto P de ella corta a la directriz en el punto L. Probar
que el segmento LP subtiende un ángulo recto al foco.

191
13. El lugar geométrico de un punto que se mueve de modo que la diferencia de las pendientes de
las rectas que lo unen a los puntos (a, 0) y (−a, 0) es una constante no nula, es una parábola
que pasa por los puntos dados. ¿Cuáles son su eje y su vértice ?
14. Pruebe que el lugar geométrico de un punto que se mueve de forma que su distancia a una
circunferencia dada es igual a su distancia desde una recta dada que pasa por el centro de
la circunferencia son dos parábolas congruentes, cuyos focos coinciden con el centro de la
circunferencia y cuyos ejes son perpendiculares a la recta fija.
15. Probar que la cuerda x+by = 4a subtiende un ángulo recto al vértice de la parábola y 2 = 4ax.
16. Una circunferencia variable pasa por un punto fijo A y es tangente a una recta fija L; AP
es diámetro de la circunferencia. Demostrar que el lugar geométrico de P es una parábola;
hallar su foco, vértice y directriz.
17. Demostrar que el lugar geométrico de los puntos medios de las cuerdas de una parábola que
pasan por su vértice es otra parábola.
18. Una cuerda variable de una parábola subtiende un ángulo recto al vértice. Demostrar que
pasa por un punto fijo.
19. Demostrar que las parábolas y 2 = ax, x2 = by se cortan segun el ángulo
1 1
3a 3 b 3
arctan( 2 2 ).
2(a + b 3 )
3

6.4. La hipérbola

Definición 6.4.1. Una hipérbola es el lugar geométrico de todos los puntos del plano que verifican
que el valor absoluto de la diferencia de sus distancias a dos puntos fijos, llamados focos, es igual
a una cantidad constante positiva, mayor que la distancia entre los focos. Se acostumbra denotar
esa constante por 2a.

192
La figura anterior muestra cómo construir puntos de la hipérbola utilizando una circunferencia
de radio 2a con centro en uno de los focos y con la ayuda de simetrales. Toda hipérbola está con-
formada por dos ramas infinitas disconexas. El eje que pasa por ambos focos, como antes, recibe
el nombre de eje focal. Una cuerda que pase por el foco se llama cuerda focal. Si la cuerda focal
además es perpendicular al eje focal se conoce, como en las otras cónicas, como Latus rectum.

Para determinar una ecuación sencilla de la hipérbola, hacemos coincidir el eje x con el eje
focal y el eje y con la simetral del segmento F1 F2 . Denotemos por (c, 0) y (−c, 0) las coordenadas
de los focos.

La condición del lugar geométrico se traduce entonces en


p p
(x + c)2 + y 2 − (x − c)2 + y 2 = ±2a
Pasando una de las raı́ces al lado derecho de la ecuación anterior y elevando al cuadrado en dos
oportunidades hasta eliminar los radicales, obtenemos

(c2 − a2 )x2 − a2 y 2 = a2 (c2 − a2 ).


Como c > a, c2 − a2 es una cantidad positiva que podemos denotar por b2 . Sustituyendo en la
ecuación anterior b2 = c2 − a2 , obtenemos

x2 y2
− = 1.
a2 b2
que es la ecuación de la hipérbola.

Ası́ntotas de la hipérbola.

Una recta y = mx + n es ası́ntota de una curva en general y = f (x) si verifica que la distancia
entre la curva y la recta se va a cero cuando x crece indefinidamente.

193
Informalmente lo que ocurre es que la curva se pega a la recta al crecer el valor de la variable.
La elipse y la parábola no tienen ası́ntotas, La hipérbola tiene un par de rectas que cumplen la
propiedad descrita anteriormente. Describiremos a continuación cómo encontrarlas.

x2 y2
Supongamos que consideramos un punto P (x, y) en la hipérbola de ecuación + = 1.
a2 b2
Si consideramos el punto en el primer cuadrante podemos despejar su coordenada y en términos
de su coordenada x,

b2 x2 − b2 a2 b 1 p
y = , por lo que y − x = ( b2 x2 − b2 a2 − bx). Racionalizando la expresión
a a a
anterior obtenemos

b b2 a
| y − x |= √ .
a b2 x2 − b2 a2 + bx
la expresión del lado derecho claramente tiende a cero cuando x aumenta su valor.
√ Análogamente
b b2 x2 − b2 a2
podemos probar que la recta y = − a x, es ası́ntota de la rama negativa y = − . De lo
a
anterior concluimos:

x2 y2
Teorema 6.4.0.82. Las rectas y = ± ab x son ası́ntotas de la hipérbola de ecuación + = 1.
a2 b2
c
Como en la elipse, la excentricidad e = a > 1 es una medida de la forma de la hipérbola.

Ejemplo 6.4.2. Trazar la gráfica de la hipérbola de ecuación 9x2 − 4y 2 = 36. Determinar sus focos,
vértices, las ecuaciones de sus ası́ntotas y su excentricidad.

Respuesta:

194
Dividiendo por 36, tenemos que
x2 y2
− =1
4 9
luego a2 = 4 ⇒ a = 2, b2 = 9 ⇒ b = 3, recuede que estas constantes son positivas por
√ √ √
definición.c2 = a2 +b2 = 13 luego c = 13. e = 213 , sus ası́ntotas y = ± 23 x. Los focos: F1 (− 13, 0)

y F2 ( 13, 0). Los vértices: V1 (−2, 0) y V2 (2, 0).

Si fijamos el sistema de coordenadas, la ecuación asociada a la hipérbola cambiará de acuerdo


a dónde ubiquemos los focos y al valor de la constante 2a. Si los focos están ubicados en el eje y,
con coordenadas (0, c) y (0, −c) con a2 + b2 = c2 , la ecuación es ahora

y2 x2
2
− 2 = 1.
a b
Las ası́ntotas cambian a y = ± ab x.

Ejemplo 6.4.3. Trazar la gráfica de la hipérbola de ecuación 4y 2 − 2x2 = 1. Determinar sus focos,
vértices, las ecuaciones de sus ası́ntotas y su excentricidad.

Respuesta:

Reescribiendo la ecuación tenemos que


y2 x2
− =1
1/4 1/2
1 1 1
√ √
luego a2 = 4 ⇒ a = 2, b
2
= 2 ⇒ b = √12 . c2 = a2 + b2 = 43 luego c = 3/2. e = 3, sus
√ √ √
ası́ntotas y = ± 22 x. Sus focos: F1 (0, − 3/2) yF2 (0, 3/2). Sus vértices: V1 (0, −1/2) y V2 (0, 1/2).

195
Tangente a la Hipérbola en un punto de ella

Un argumento similar al dado para el caso de la elipse permite probar el siguiente teorema

x2 y2
Teorema 6.4.0.83. La ecuación de la recta tangente a la hipérbola 2 − 2 = 1, en un punto
a b
xx0 yy0
P (x0 , y0 ) de ella es 2 − 2 = 1.
a b
Ejercicios

1. Si la excentricidad de una hipérbola es 2 y su eje real es 3 calcular la medida de su eje


conjugado.

2. Determinar la ecuación de una hipérbola de excentricidad 2 cuyos focos distan 4 entre sı́.
2
3. Demostrar que las ası́ntotas de la hipérbola Ax√ − By 2√= C, en que las constantes A, B y C
son positivas, están dadas por las ecuaciones x A ± y B = 0.
4. Hallar la ecuación de la hipérbola cuyas ası́ntotas forman un ángulo de 60◦ con el eje x y
cuyos vértices son (1, 0) y (−1, 0).

5. Probar que las pendientes de las ası́ntotas de la hipérbola x2 /a2 − y 2 /b2 = 1, son ± e2 − 1.
6. La pendiente de una ası́ntota de la hipérbola anterior es 3/4 calcular la excentricidad.
7. Si a = b la hipérbola se dice equilátera. Hallar la ecuación de la hipérbola equilátera, cuyos
focos tienen distancia 1 al centro.

196
8. Hallar la ecuación de la normal a la hipérbola

x2 y2
− =1
25 144
en los extremos del Latus rectum del primer cuadrante.
9. Probar que la suma de los cuadrados de los recı́procos de las excentricidades de las hipérbolas
conjugadas

x2 y2 x2 y2
− = 1, y − = −1
9 16 9 16
es igual a la unidad.

10. Encontrar la ecuación de una hipérbola cuyas ası́ntotas son las rectas 2x + y = 0, 2x − y = 0
y que pasa por el punto (1, 1).
11. Las proyecciones de un punto P de una hipérbola sobre los ejes real y conjugado, respec-
tivamente, son P1 y P2 . La tangente en P corta a estos ejes en T1 y T2 respectivamente.
Demostrar que OP1 · OT1 = a2 y que OP2 · OT2 = −b2 , en que O es el centro de la hipérbola
y a, b son los semiejes.
12. Una recta se mueve de modo que el área del triángulo que forma con dos rectas perpendic-
ulares dadas es constante. Hallar el lugar geométrico del punto medio del segmento de esta
recta comprendido entre estas perpendiculares.
x2 y2
13. Considere la hipérbola a2 − b2 = 1, de centro O y sea P un punto de ella. Sea OP = r.
2 2
Considere además la hipérbola yb2 − xa2 = 1, de centro O y sea P 0 un punto de ella. Sea
OP 0 = r0 . Si OP es perpendicular con OP 0 demostrar que

1 1 1 1
2
+ 02 = 2 + 2 .
r r a b
14. Si las ası́ntotas de una hipérbola de excentricidad e forman un ángulo de 2ω, probar que

e2 − 1 p
sin(ω) = , sec(ω) = e, tan(ω) = e2 − 1.
e
x2 y2
15. Discutir la variación de la fórma de la hipérbola a2 − b2 = k, para 0 < k < 1.

197
Capı́tulo 7

Transformaciones de coordenadas

En este capı́tulo presentamos los cambios de sistema coordenado realizados a través de una
traslación o una rotación de los ejes. Hacemos el estudio de la ecuación general de segundo grado
en dos variables Ax2 + Bxy + Cy 2 + Dx + Ey + F = 0. Describimos y estudiamos en forma
geométrica y matricial, las transformaciones geométricas del plano.

7.1. Cambio de sistema de coordenadas


7.1.1. Traslación de ejes coordenados

Supongamos que tenemos una curva en el plano, como hemos visto antes, al colocar un sistema
de ejes ortogonales, las coordenadas de sus puntos verifican cierta relación.

Si cambiamos el sistema coordenado y colocamos nuevos ejes la relación anterior cambia algunos
de sus coeficientes. Es muy importante que el lector se dé cuenta que la ecuación es sólo una
representación de la curva y que cambia de acuerdo a como estén ubicados los ejes coordendos.

Partiremos analizando las relaciones entre dos ejes coordenados en que los ejes son rectas
paralelas.

198
De acuerdo a la figura inicial
~ = OO
OP ~ 0 + O~0 P
o equivalentemente (x, y) = (h, k) + (u, v) de donde obtenemos las fórmulas de traslación

(∗) x = u + h, y = v + k.
Ejemplo 7.1.1. Eliminar los términos de primer grado en la ecuación:
2x2 + 2xy + y 2 + 5x + 6y − 8 = 0.
Respuesta:

Hacemos una traslación y ajustaremos el centro (h, k) para que esto suceda.
x = u + h, y = v + k.
Reemplazando en la ecuación obtenemos:

2(u + h)2 + 2(u + h)(v + k) + (v + k)2 + 5(u + h) + 6(v + k) − 8 = 0,


Desarrollando la ecuación anterior obtenemos:

2u2 + 2uv + v 2 + (4h + 2k + 5)u + (2h + 2k + 6)v + 2h2 + 2hk + k 2 + 5h + 6k − 8 = 0,


luego podemos pedir que

2h + 2k + 6 = 0
,
4h + 2k + 5 = 0
1 −7
el sistema anterior entrega las soluciones h = , k = . En los nuevos ejes u, v la ecuación
2 2
inicial se transforma en:
69
2u2 + 2uv + v 2 − =0
4

199
Teorema 7.1.1.1. Si en la ecuación Ax2 + Bxy + Cy 2 + Dx + Ey + F = 0 hacemos una traslación
de ejes en la forma x = u + h, y = v + k. los términos cuadráticos permanecen invariantes, además
si B 2 − 4AC 6= 0 siempre es posible eliminar los términos de primer grado dejando sólo el término
libre. Es decir existen h y k de forma que la ecuación se transforma en: Au2 + Buv + Cv 2 + F 0 = 0.

Demostración:

Reemplazando en la ecuación inicial obtenemos

A(u + h)2 + B(u + h)(v + k) + C(v + k)2 + D(u + h) + E(v + k) + F = 0,

que al ser desarrollada se transforma en

Au2 + Buv + Cv 2 + (2Ah + Bk + D)u + (Bh + 2Ck + E)v + Ah2 + Bhk + Ck 2 + Dh + Ek + F = 0.

Para eliminar los términos lineales, debemos resolver el sistema

2Ah + Bk = −D
,
Bh + 2Ck = −E

Si 4AC − B 2 6= 0 el sistema tiene solución única para h, k en términos de los demás parámetros,
luego es posible eliminar los términos de primer grado. Lo anterior prueba el teorema.

También es posible eliminar uno de los términos lineales y el término libre resolviendo el sistema
apropiado.

Ejercicios

Hacer la traslación que elimine los términos indicados en los siguientes ejercicios:

1. x2 − 3xy + 5y 2 + 8x − 26y + 38 = 0; términos de primer grado.


2. 3x2 − xy + 4y 2 + 5x − 8y − 3 = 0; términos de primer grado.
3. x2 + 7xy − 2y 2 − 2x − 14y − 3 = 0; términos de primer grado.
4. 7xy − 2x − 14y − 3 = 0; términos de primer grado.
5. y = x3 − 9x2 + 24x + 3; constante, término en x.
6. y = x3 − 6x2 + 11x − 8; constante, término en x2 .
7. x2 y − 2x2 + 2xy + y − 4x − 6 = 0; términos de segundo grado.
8. Demuestre que cualquier traslación en Ax2 + Bxy + Cy 2 + Dx + Ey + F = 0 deja invariantes
los términos A, B, C.
9. Realice una traslación en x2 + 5y 2 + 8x − 26y + 38 = 0; de dos formas: Primero completando
cuadrados, luego utilizando las fórmulas de traslación. Elimine en ambos casos los términos
lineales.

200
7.1.2. Rotación de ejes coordenados
Analizaremos ahora una segunda forma de modificar los ejes. Esta vez consideraremos un nuevo
sistema de ejes u, v que resulta de rotar los ejes antiguos x, y en un ángulo α en sentido antihorario.

Sean ı̂,̂ los vectores de la base del sistema antiguo x, y. Sean k̂, ŝ la base del sistema nuevo u, v.
Luego p~ = xı̂ + y̂, p~ = uk̂ + vŝ.Las relaciones son las siguientes

k̂ = cos(α)ı̂ + sin(α)̂
ŝ = − sin(α)ı̂ + cos(α)̂.

Reemplazando obtenemos,

p~ = u(cos(α)ı̂ + sin(α)̂) + v(− sin(α)ı̂ + cos(α)̂),


reordenando,

p~ = (u cos(α) − v sin(α))ı̂ + (u sin(α) + v cos(α))̂ = xı̂ + y̂,


De lo anterior obtenemos las fórmulas de cambio entre los dos sistemas coordenados:

x = u cos(α) − v sin(α)
y = u sin(α) + v cos(α).
En el sistema anterior podemos despejar las variables u y v y obtenemos

u = x cos(α) + y sin(α)
v = −x sin(α) + y cos(α).
Si una curva tiene una ecuación F (x, y) = 0 en el sistema XY las fórmulas anteriores permiten
determinar su ecuación en un sistema U V que se ha obtenido del anterior rotando los ejes en un
ángulo α. Obtenemos entonces otra representación G(u, v) = 0 de la misma curva, que relaciona

201
las coordenadas en el otro sistema coordenado y viceversa. Explicaremos lo anterior a través de
algunos ejemplos.

Ejemplo 7.1.2. Determinar la ecuación asociada a xy = 1 en un sistema de ejes rotado en 45◦


respecto al sistema original. Identificar la curva.

Respuesta:

1
Como α = 45◦ tanto cos(α) como sin(α) valen √ . La ecuaciones de cambio quedan:
2
x = u−v √
2

y = u+v √ .
2
  
u−v u+v
xy = 1 ⇔ √ √ =1
2 2
u2 v2
Luego la ecuación es − = 1, por lo que la curva es una hipérbola de ası́ntotas v = ±u
2 2
que coinciden con los ejes antiguos.

En ocaciones es de suma utilidad eliminar el término rectángulo xy, en una expresión de la


forma Ax2 + Bxy + Cy 2 + Dx + Ey + F = 0. El teorema siguiente nos indica cómo hacerlo.

Teorema 7.1.2.1. Siempre es posible eliminar el término rectángulo en una ecuación de la forma
Ax2 + Bxy + Cy 2 + Dx + Ey + F = 0 rotando en un ángulo α que verifique
A−C
cot(2α) = .
B

202
Demostración:

Reemplazamos las fórmulas de rotación en la ecuación y obtenemos:

A(u cos(α) − v sin(α))2 + B(u cos(α) − v sin(α))(u sin(α) + v cos(α)) +


C(u sin(α) + v cos(α))2 + D(u cos(α) − v sin(α)) + E(u sin(α) + v cos(α)) + F = 0.

Agrupamos ahora los productos que involucran al factor uv.

(−2A cos(α) sin(α) + B(cos2 (α) − sin2 (α)) + 2C cos(α) sin(α)).


Simplificamos e igualamos a cero:
A−C
(C − A) sin(2α) + B cos(2α) = 0 ⇒ cot(2α) = .
B
Luego, de la ecuación trigonométrica final, obtenemos el ángulo que elimina el término rectángu-
lo.

Ejemplo 7.1.3. Determinar el cambio de coordenadas que elimina el término rectángulo en la


ecuación x2 + 4xy − 2y 2 − 6 = 0.

Solución:
3 3
El ángulo de rotación debe verificar cot(2α) = , luego cos(2α) = . Para obtener las fórmulas
4 5
de rotación utilizamos el hecho que
r r
1 + cos(2α) 1 − cos(2α)
cos(α) = , sin(α) =
2 2
2 1
Por lo que cos(α) = √ y sin(α) = √
5 5
Para eliminar el término rectángulo debemos girar los ejes en un ángulo de α ≈ 63, 5◦ y los
cambios son:
2u−v
x = √
5

u+2v
y = √ .
5

Al sustituir en la ecuación original obtenemos:

2u2 − 3v 2 = 6.

203
Ejercicios

1. Determine la nueva representación algebraica de la curva cuya ecuación respecto a un sistema


cartesiano ortogonal x, y está indicada, respecto a nuevos ejes u, v que se han girado en un
ángulo indicado:

a) 2x + y = 5; α = arctan(3/2).
b) x2 − 2xy + y 2 + x − 2y = 5; α = 45◦ .

c) 31x2 + 10 3xy + 21y 2 − 196 = 0; α = 30◦ .

d ) 11x2 − 50 3xy − 39y 2 + 576 = 0; α = 60◦ .
1
e) 4x2 + 3xy + 54 = 0; α = 2 arctan( 43 ).

2. Elimine el término xy de la ecuación dada mediante una rotación de ejes adecuada



a) 2x2 − 3xy + 3y 2 = 2
b) 5x2 + xy + 10y 2 − 3x + 2y = 6
c) 2x2 + xy + y 2 = 5.
d ) 8x2 − xy + 2y 2 − x + 2y = 6.

3. Pruebe que el indicador B 2 − 4AC no cambia su valor al efectuar una rotación en Ax2 +
Bxy + Cy 2 + Dx + Ey + F = 0, cualquiera sea el valor de α.

7.2. Ecuación General de Segundo Grado.

En secciones anteriores estudiamos la ecuación general de la recta Ax + By + C = 0 que, en su


expresión algebraica, contiene como máximo términos de primer grado en ambas variables.

Lo que estudiaremos en esta sección es la ecuación general de segundo grado en dos variables
y los lugares geométricos que ella puede representar. Se entiende por ecuación general de segundo
grado a aquella expresión algebraica del tipo:

Ax2 + Bxy + Cy2 + Dx + Ey + F = 0


donde al menos uno de los tres coeficientes de la parte cuadrática (A, B o C) es distinto de cero.

Si las coordenadas cartesianas (x, y) de los puntos de un lugar geométrico satisfacen una
ecuación de este tipo, ¿qué caracterı́sticas tiene el lugar geométrico dependiendo de los valores
que toman los coeficientes de la ecuación?

Demostraremos en esta sección que los lugares geométricos cuyas ecuaciones tienen esta estruc-
tura algebraica no son otros sino las cónicas que estudiamos más arriba, con la precisión de que
los ejes de estas cónicas no necesariamente son paralelos a los ejes de coordenadas y sus centros
no tienen que coincidir con el origen del sistema de coordenadas. De este modo, bastará realizar

204
transformaciones de coordenadas convenientes, de modo que la ecuación del lugar geométrico en
el nuevo sistema adopte una de las formas que vimos anteriormente y que se denominan formas
canónicas de las cónicas.

La ecuación general de segundo grado puede reducirse, mediante transformaciones de coorde-


nadas, a su forma canónica. Mostraremos que bastará con una traslación y una rotación, como
máximo, para lograr este objetivo.

El primer paso consiste en diagonalizar la parte cuadrática de la ecuación, de modo que de-
saparezca el término cuadrático mixto xy. Para esto aplicamos el proceso descrito en la sección
anterior que consiste en relizar una rotación del sistema de coordenadas con un ángulo conveniente.
De este modo, la ecuación original

Ax2 + Bxy + Cy2 + Dx + Ey + F = 0

se ha transformado en

A0 u2 + C0 v2 + D0 u + E 0 v + F = 0

Mientras que el término independiente F no se altera al rotar el sistema de coordenadas.

El segundo paso consiste en hacer un completamiento cuadrático o equivalentemente una


traslación en la ecuación, de modo que desaparezcan los términos lineales siempre que sea posible.

En estas nuevas coordenadas la ecuación nos queda:

A0 t2 + C 0 s2 + F 0 = 0

Debemos precisar que es posible que no se pueda realizar el completamiento cuadrático en


ambas variables. Esto podrı́a ocurrir si, después de la rotación, alguno de los coeficientes A0 o C 0
es igual a cero, lo que implicarı́a que no aparece el término cuadrático en alguna de las variables.

Ejemplificaremos lo anterior en casos particulares.

Ejemplo 7.2.1. Sea dada la ecuación general de segundo grado:

3x2 + 2xy + 3y 2 + 2x − 10y + 3 = 0

Reduzca esta ecuación a su forma canónica, determine el lugar geométrico que ella representa
y describa geométricamente las transformaciones de coordenadas realizadas.

Si queremos reducir esta ecuación a su forma canónica, dado que aparece el término cuadrático
mixto xy, debemos hacer una rotación primero. En todas las alternativas necesitamos conocer el
seno y el coseno del ángulo de rotación, por lo que esta primera parte es común a todas las variantes
de solución.

205
Como A = 3, B = 2 y C = 3 tenemos que el ángulo de rotación satisface:
3−3 π π
cot 2θ = =0 ⇒ 2θ = ⇒ θ=
2 2 4
de este modo:
√ √
2 2
sen θ = y cos θ =
2 2
El cálculo algebraico necesario debe realizarse para cada ecuación:

El cambio de coordenadas es:


 √
2
 x =

2 (u − v)


 y 2
= 2 (u + v)

De este modo podemos hacer la sustitución en la ecuación general:

3x2 + 2xy + 3y 2 + 2x − 10y + 3 = 0

"√ #2 √ √ "√ #2 √ √
2 2 2 2 2 2
3 (u − v) + 2 (u − v) (u + v) + 3 (u + v) + 2 (u − v) − 10 (u + v) + 3 = 0
2 2 2 2 2 2

3 3 √ √
(u − v)2 + (u − v)(u + v) + (u + v)2 + 2(u − v) − 5 2(u + v) + 3 = 0
2 2
√ √
4u2 + 2v 2 − 4 2u − 6 2v + 3 = 0

Esta es la ecuación de la curva en el sistema rotado. Note que no aparece el término cuadrático
mixto.

Para hacer la traslación (completamiento cuadrático), podemos reagrupar los términos anteri-
ores.
 √   √ 
4 u2 − 2u + 2 v 2 − 3 2v + 3 = 0

por lo que el cambio de coordenadas necesario es:


 √
2
 t = u−

2


 s = v− 3 2
2

y sustituyendo en la ecuación obtenemos:


 √   √ 
4 u2 − 2u + 2 v 2 − 3 2v + 3 = 0

206
√ !2 √ !2
2 3 2 1 9
4 u− +2 v− +3−4 −2 =0
2 2 2 2

4t2 + 2s2 − 8 = 0
que es la forma canónica de la ecuación de segundo grado.
Note que si dividimos por 8 la ecuación, se tiene:

t2 s2
√ + =1
( 2)2 (2)2

que se reconoce fácilmente como la ecuación de una elipse


√ con eje mayor en la coordenada s, de
tamaño 2 y eje menor por la coordenada t, de longitud 2.

Se pueden describir gráficamente los cambios de coordenadas realizados. Note que, en primer
π
lugar, se realizó unarotación de
 los ejes coordenados en un ángulo θ = 4 y posteriormente, una
√ √
2 3 2
traslación de vector 2 , 2 .

En la siguiente figura se muestran ambos cambios de coordenadas y se aprecia cómo, al realizar


este proceso, la cónica queda centrada en el origen y con sus ejes paralelos a los ejes coordenados
(forma canónica de la cónica).

Ejemplo 7.2.2. Sea dada la ecuación general de segundo grado:

16x2 − 24xy + 9y 2 − 320x − 260y + 25 = 0

Reduzca esta ecuación a su forma canónica, determine el lugar geométrico que ella representa
y describa geométricamente las transformaciones de coordenadas realizadas.

Como A = 16, B = −24 y C = 9 tenemos que el ángulo de rotación satisface:

207
16 − 9 7
cot 2θ = =− <0
−24 24
16 − 9 7
cos 2θ = − p =−
2
(−24) + (9 − 16) 2 25
s s
7 7
r r
1 − 25 9 3 1 + 25 16 4
cos θ = = = , sen θ = = =
2 25 5 2 25 5

El cambio de coordenadas es:


 1
 x = 5 (3u − 4v)

1
y = 5 (4u + 3v)

De este modo:

16x2 − 24xy + 9y 2 − 320x − 260y + 25 = 0

 2  2
1 1 1 1 1 1
16 (3u − 4v) −24 (3u−4v) (4u+3v)+9 (4u + 3v) −320 (3u−4v)−260 (4u+3v)+25 = 0
5 5 5 5 5 5

16(3u − 4v)2 − 24(3u − 4v)(4u + 3v) + 9(4u + 3v)2 − 1600(3u − 4v) − 1300(4u + 3v) + 625 = 0

25v 2 − 400u + 100v + 25 = 0

Para hacer la traslación reagrupamos los términos. Note que sólo se puede trasladar en la
variable v, porque luego de la rotación no apareció el término cuadrático en u.

25 v 2 + 4v − 400u + 25 = 0


por lo que el cambio de coordenadas necesario es:



 t = u

s = v+2

Sustituyendo en la ecuación obtenemos:


2
25 (v + 2) − 400u + 25 − 100 = 0

25s2 − 400t − 75 = 0
que es la forma canónica de la ecuación de segundo grado.

208
Note que si dividimos por 400 la ecuación, se tiene:

s2 3
25s2 − 400t − 75 = 0 ⇒ t= −
16 80
que se reconoce fácilmente como la ecuación de una parábola con eje en la coordenada t y vértice
3
en el punto de coordenadas (t, s) = (− 80 , 0).

Se pueden describir gráficamente los cambios de coordenadas realizados. Note que, en primer
lugar, se realizó una rotación de los ejes coordenados en un ángulo θ ≈ 0,9273 y posteriormente,
una traslación de vector (0 , 2).

En la siguiente figura se muestran ambos cambios de coordenadas y se aprecia cómo, al realizar


este proceso, la cónica queda con sus ejes paralelos a los ejes coordenados (forma canónica de la
cónica).

Ejemplo 7.2.3. Sea dada la ecuación general de segundo grado:

2x2 + 24xy − 8y 2 + 3x − 2y − 1 = 0

Como A = 2, B = 24 y C = −8 tenemos que el ángulo de rotación satisface:


2+8 5
cot 2θ = = >0
24 12
2+8 5
cos 2θ = p =
2
(24) + (2 + 8) 2 13
s
5
r √ s
5
r √
1 + 13 18 3 13 1 − 13 8 2 13
cos θ = = = , sen θ = = =
2 26 13 2 26 13

El cambio de coordenadas es:

209
 √
13
 x =

13 (3u − 2v)


 y 13
= 13 (2u + 3v)

De este modo:

2x2 + 24xy − 8y 2 + 3x − 2y − 1 = 0

√ √
2 24 8 3 13 2 13
(3u − 2v)2 + (3u − 2v)(2u + 3v) − (2u + 3v)2 + (3u − 2v) − (2u + 3v) − 1 = 0
13 13 13 13 13
√ √
2 5 13
2 12 13
10u − 16v + u− v−1=0
13 13

Para hacer la traslación reagrupamos los términos.


√ ! √ !
13 3 13
10 u2 + u − 16 v 2 + v −1=0
26 52
por lo que el cambio de coordenadas necesario es:
 √
13
 t = u+

52


 s = v+ 3 13
104

Sustituyendo en la ecuación rotada:


√ !2 √ !2
13 3 13 5 9
10 u + − 16 v + −1− + =0
52 104 104 52
7
10t2 − 16s2 −=0
8
que es la forma canónica de la ecuación de segundo grado.
Note la anterior ecuación puede reescribirse como:

t2 s2
 √ 2 −  √ 2 = 1
35 14
20 16

que se reconoce

fácilmente como la ecuación de

una hipérbola con eje mayor en la coordenada t ,
de tamaño 2035 y eje menor en s, de longitud 1614 .

Se pueden describir gráficamente los cambios de coordenadas realizados. Note que, en primer
lugar, se realizó una rotación
 √ de √ ejes coordenados en un ángulo θ ≈ 0, 588 y posteriormente,
los 
13 3 13
una traslación de vector 52 , 104 .

210
En la siguiente figura se muestran ambos cambios de coordenadas y se aprecia cómo, al realizar
este proceso, la cónica queda centrada y con sus ejes paralelos a los ejes coordenados (forma
canónica de la cónica).

Podemos también comenzar realizando la traslación como explicamos en el siguiente ejemplo.

Ejemplo 7.2.4. Determinar centro, vértices, focos y ası́ntotas, si es que existen, de la curva de
ecuación x2 − 3xy + y 2 + 8x − 2y + 5 = 0.

Respuesta:

En primer lugar eliminamos los términos de primer grado a través de una traslación de ejes
coordenados.

x = u + h, y = v + k. Reemplazando e igualando a cero los términos de primer grado, tenemos


el sistema:

2h − 3k = −8

−3h + 2k = 2.
El sistema anterior tiene soluciones h = 2, k = 4. Lo anterior indica que el cetro de la cónica
es el (2, 4) y las ecuaciones de traslación son x = u + 2, y = v + 4. En el nuevo sistema U V la
ecuación de la curva es:

(u + 2)2 − 3(u + 2)(v + 4) + (v + 4)2 + 8(u + 2) − 2(v + 4) + 5 = 0,


que simplificada se reduce a:
u2 − 3uv + v 2 + 9 = 0.
Estamos ahora en condiciones de realizar la rotación en nuestro ejemplo para eliminar el término
rectángulo en u2 − 3uv + v 2 + 9 = 0.

211
π π
cot(2α) = 0 ⇒ 2α = ⇒α=
2 4
1
Luego, tanto cos(α) como sin(α) valen √ . La ecuaciones de cambio quedan:
2
u = t−s√
2

t+s
v = √ .
2
Reemplazamos ahora en la ecuación:
t−s t−s t+s t+s
( √ )2 − 3( √ ) √ + ( √ )2 + 9 = 0
2 2 2 2
Simplificamos para obtener

t2 s2
− 18 = 1.
18 5
√ q
18
La ecuación representa ası́ una hipérbola con los siguientes elementos: a = 18, b = 5
q
c = a2 + b2 luego c = 108
2
5 .

Primero obtenemos las coordenadas canónicas en el sistema t, s y para obtenerlas en los otros
sistemas utilizamos las fórmulas de cambio:
t−s
u = √
2

t+s
v = √ ,
2
y
x = u + 2, y = v + 4.

Sistema t, s u, v x, y

Centro (0, 0) (0, 0) (2, 4)

√ √
Vértices ( 18, 0), (− 18, 0) (3, 3), (−3, −3) (5, 7), (−1, 1)

√ √
5 3+ 5
Ası́ntotas t = ± √s5 v=( 3
2 +√2 )u y =4+ 2√ (x − 2)
v= ( 32 − 25 )u y =4+ 3− 5
2 (x − 2)

q q q q q
Focos ( 108 , 0) (3 65 , 3 65 ) (2 + 3 65 , 4 + 3 65 )
q5 q q q q
(− 108
5 , 0) (−3 65 , −3 65 ) (2 − 3 65 , 4 − 3 65 )

212
El siguiente es el gráfico estimativo de la curva:

Ejercicios

1. Trace y describa la gráfica de las siguientes ecuaciones determinando sus elementos princi-
pales:

a) 4x2 + y 2 − 8x + 6y + 9 = 0.
b) y 2 − 8x + 4y + 28 = 0 .
c) 7x2 + 6xy − y 2 − 32 = 0.
√ √
d ) 4x2 + 4xy + y 2 + 8 5x − 16 5y = 0.
e) 2x2 + xy + x − y 2 + 7y − 10 = 0.

2. Se puede verificar que dos secciones cónicas tienen cuando más, cuatro puntos en común pero
2x2 + xy − y 2 + 3y − 2 = 0 y 2x2 + 3xy + y 2 − 6x − 5y + 4 = 0 tienen cinco puntos en común
(1, 0), (−2, 3), (5, −4), (−6, 7), (10, −9). ¿Por qué ?
3. Determine la ecuación de la parábola con foco F (3, 4) y directriz la recta 2x−y = 4. Revierta
el proceso partiendo de la ecuación y determinando los elementos principales.
4. Considere las ecuaciones de segundo grado de las cónicas estudiadas en el Capı́tulo 6. Calcule
el discriminante de cada una y demuestre que se tiene la siguiente caracterización:

a) Elipse ⇒ ∆ < 0.
b) Parábola ⇒ ∆ = 0.
c) Hipérbola ⇒ ∆ > 0.

213
7.3. Transformaciones en un mismo sistema coordenado
Una transformación del plano en sı́ mismo es una operación geométrica que mueve los puntos de
un lugar a otro, según ciertas reglas. Una transformación geométrica se le aplica individualmente
a cada uno de los puntos del plano o del espacio por lo que, en el caso de figuras geométricas más
complejas, éstas se transforman según lo hagan cada uno de los puntos que la conforman.

Entre las transformaciones geométricas hay dos tipos que, por su importancia en la resolución
de problemas y el estudio de propiedades de figuras geométricas, son de especial interés. Primera-
mente nos referiremos a las transformaciones conformes que tienen la propiedad de conservar los
ángulos. Es decir, si dos rectas forman un determinado ángulo y se les aplica una transformación
conforme, sus imágenes formarán el mismo ángulo. Por otro lado están las isometrı́as que tienen
la caracterı́stica de que si le aplicamos la transformación a cualquier segmento, el resultado es otro
segmento con la misma longitud del original. Es fácil verificar que cualquier isometrı́a es una trans-
formación conforme. Estudiaremos cuatro transformaciones conformes: la Traslación, la Rotación,
la Reflexión y la Homotecia. Las tres primeras son isometrı́as, mientras que la última no lo es.

Sea Π el conjunto de todos los puntos del plano. Consideraremos algunas transformaciones
fundamentales del conjunto Π sobre sı́ mismo.

7.3.1. Traslación en el plano


−−→ −−→
Dado un segmento dirigido AB del plano definimos la traslación T (AB) del plano Π sobre
sı́ mismo como la transformación que transporta cada punto P del plano al punto P 0 tal que
−−→ −−→
el segmento P P 0 es paralelo, de igual longitud y con el mismo sentido que el segmento AB. El
−−→
segmento dirigido AB se denomina vector de la traslación.

−−→ −−→
La traslación T (AB) no tiene puntos invariantes. La inversa de la traslación T (AB) es la
−−→ −→
traslación T (BA). Para cualquier punto A, la traslación T (AA) es la identidad.

7.3.2. Rotación en el plano


Dado un punto fijo O del plano y un ángulo θ definimos la rotación R(O, θ) del plano Π sobre
sı́ mismo como la transformación que lleva cada punto P del plano al punto P 0 tal que OP 0 = OP
y ∠P OP 0 = θ. El punto O se llama centro de rotación y θ es el ángulo de rotación.

214
Por convención se toma el sentido antihorario como el sentido de rotación y el ángulo θ entre
0◦ y 360◦ . La rotación R(O, θ) sólo deja invariante al punto O para cualquier θ 6= 0◦ , 360◦ . La
transformación inversa de la rotación R(O, θ) es la rotación R(O, −θ) (el ángulo negativo se entiende
como rotación en sentido horario). Las rotaciones R(O, 0◦ ) y R(O, 360◦ ) coinciden con la identidad.

7.3.3. Reflexión en el plano


Sea ` una recta fija del plano. Definimos la reflexión R(`) del plano Π sobre sı́ mismo como
la transformación que lleva cada punto P del plano al punto P 0 tal que ` sea la mediatriz del
segmento ¬P P 0 . La recta ` se llama eje de la reflexión.

La reflexión R(`) es involutiva. Toda la recta ` es invariante por la reflexión R(`).

7.3.4. Homotecia en el plano


Dado un punto fijo O del plano y un número real distinto de cero k definimos la homotecia
H(O, k) del plano Π sobre sı́ mismo como la transformación que lleva cada punto P del plano al
punto P 0 tal que OP 0 = kOP . El punto O se llama centro de la homotecia y k es la razón de la
homotecia.

215
Para cualquier k 6= 1, el punto O es el único invariante por la homotecia H(O, k). La transfor-
mación inversa de la homotecia H(O, k) es la homotecia H O, k1 . La homtecia H(O, 1) coincide
con la identidad.

Ejercicios

1. Demuestre las siguientes propiedades de las transformaciones geométricas anteriores.


a) R(O, 180◦ ) = H(O, −1).
−−→ −−→ −−→ −→ −→
b) T (AB)T (BC) = T (BC)T (AC) = T (AC).
c) R(O, θ1 )R(O, θ2 ) = R(O, θ2 )R(O, θ1 ) = R(O, θ1 + θ2 ).
2. Demuestre que una traslación es una isometrı́a.
3. Demuestre que la reflexión es una isometrı́a.
4. Demuestre que una rotación es una isometrı́a.
−−→ −−−→
5. Si AB se transporta por una rotación a A0 B 0 , encontrar el centro de rotación.
6. Demuestre que si O 6= O0 , entonces R(O0 , −θ)R(O, θ) es una traslación.
7. Demuestre que si θ es el ángulo entre dos rectas secantes `1 y `2 , entonces R(`1 )R(`2 ) =
R(O, 2θ).
−−−→
8. Demuestre que si A1 A2 es la distancia dirigida entre dos rectas paralelas `1 y `2 , entonces
−−−→
R(`1 )R(`2 ) = T (2A1 A2 ).
9. ¿Dónde está un punto P si su imagen por la homotecia H(O1 , k1 ) coincide con su imagen
por la homotecia H(O2 , k2 ), con k1 6= k2 ?

216
7.4. Matrices.
Para el estudio de transformaciones del plano introduciremos al estudiante a lo que se conoce en
matemáticas como matrices. Estos objetos matemáticos permiten expresar relaciones algebraicas
de manera más compacta y sus propiedades facilitarán el análisis cualitativo de lugares geométicos
que queremos describir.

Una matriz es un arreglo bidimensional de números reales dispuestos entre paréntesis. Las
dimensiones de una matriz se refieren a la cantidad de filas (lı́neas horizontales) y columnas (lı́neas
verticales) del arreglo. Por ejemplo,
 
a11 a12 · · · a1m
 a21 a22 · · · a2m 
 
 .. .. .. .. 
 . . . . 
an1 an2 ··· anm

es una matriz de n filas y m columnas (matriz de n × m).

Una matriz cuadrada de 2 × 2 es un arreglo de 4 números reales dispuestos ordenadamente en


la forma:
 
a11 a12
A=
a21 a22

Estas matrices pueden considerarse como un arreglo de dos  vectores


 (a11 , a12 ) y (a21 , a22 )
a11 a12
dispuestos en filas (horizontalmente); o de otros dos vectores y dispuestos en
a21 a22
columnas (verticalmente). En lo adelante haremos la distinción entre un vector fila y un vector
columna dependiendo de si lo consideramos una matriz de 1 × 2 (una fila y dos columnas) o una
matriz de 2 × 1 (dos filas y una columna). En general consideraremos siempre a los vectores como
vectores columna (dispuestos verticalmente o, en el nuevo lenguaje, como matrices de 2 × 1).

Pueden definirse operaciones entre matrices de la siguiente forma: dado un número real α ∈ R
y dos matrices    
a11 a12 b11 b12
A= y B=
a21 a22 b21 b22

Se define la suma de matrices A + B como:


     
a11 a12 b11 b12 a11 + b11 a12 + b12
A+B = + =
a21 a22 b21 b22 a21 + b21 a22 + b22

Se define el producto de un número real por una matriz α A como:


   
a11 a12 α a11 α a12
αA=α =
a21 a22 α a21 α a22

217
   
1 2 5 6
Como ejemplo, sean dadas las matrices A = yB= , tenemos que:
3 4 7 8
       
1 2 5 6 1+5 2+6 6 8
A+B = + = =
3 4 7 8 3+7 4+8 10 12
y      
1 2 3·1 3·2 3 6
3·A=3· = =
3 4 3·3 3·4 9 12

La operación de transposición de una matriz intercambia filas con columnas, es decir: la i-ésima
fila de la matriz original se convierte en la i-ésima columna de la matriz transpuesta:
 t    t  
1 2 1 3 2 t −1
= , = (2, 3) , (−1, 0) =
3 4 2 4 3 0
en general:
 t  
a11 a12 ··· a1m a11 a21 ··· an1
 a21 a22 ··· a2m   a12 a22 ··· an2 
 =
   
 .. .. .. .. .. .. .. .. 
 . . . .   . . . . 
an1 an2 ··· anm a1m a2m ··· anm

Note que la transpuesta de una matriz de n × m es una matriz de m × n y que la transposición


transforma vectores fila en vectores columna y viceversa.

También se puededefinir
 el producto entre dos
 matrices o entre una
 matriz y un
 vector. Sean

− x a11 a12 b11 b12
dados el vector x = y las matrices A = yB= .
y a21 a22 b21 b22

Se define el producto de un vector fila por una matriz −



x t A, como:
 

− a11 a12
x t A = (x, y) = (xa11 + ya21 , xa12 + ya22 )
a21 a22

Se define el producto de una matriz por un vector columna A −→


x , como:
     
a11 a12 x xa11 + ya12
A− →
x = =
a21 a22 y xa21 + ya22

Se define el producto de dos matrices A · B, como:


     
a11 a12 b11 b12 a11 b11 + a12 b21 a11 b12 + a12 b22
A·B = · =
a21 a22 b21 b22 a21 b11 + a22 b21 a21 b12 + a22 b22

Note que en cada caso, para obtener el término ubicado en la i-ésima fila y la j-ésima columna
del resultado basta obtener el producto escalar del i-ésimo vector fila del factor de la izquierda por
el j-ésimo vector columna del factor de la derecha.

218
De este modo, el producto de dos matrices C = A · B, tendrá tantas filas como el factor de la
izquierda (A) y tantas columnas como el factor de la derecha (B).
   
1 2 5 6
Como ejemplo, sean dadas las matrices A = y B = , y el vector −

x =
  3 4 7 8
2
tenemos que:
3
       
1 2 5 6 1·5+2·7 1·6+2·8 19 22
A·B = · = =
3 4 7 8 3·5+4·7 3·6+4·8 43 50
 

− 1 2
x t · A = (2, 3) · = (2 · 1 + 3 · 3 , 2 · 2 + 3 · 4) = (11, 16)
3 4
       

− 1 2 2 1·2+2·3 8
A· x = · = =
3 4 3 3·2+4·3 18

La transposición de matrices satisface la siguiente propiedad respecto del producto de matrices:

(A · B)t = B t · At

Ejercicios

   
1 2 −1 5
1. Dadas las matrices A = yB= calcule:
−1 −2 1 −11

219
a) A + B
b) 3B
c) A − 2B
d ) At
     
1 −1 1 e π
2. Dados el vector →

u = y las matrices A = yB= :
−1 2 −2 π −e

a) Calcule A · B y B · A. Verifique que, en general A · B 6= B · A.


b) Calcule −

u t A, −

u t B, A−

u , B−→
u, −
→u t−

u.
c) Verifique la propiedad de la transposición con respecto al producto para las matrices A
y B.

3. Demuestre que, para un vector cualquiera →



x de R2 : −

x t−

x = k−

x k2 .
4. Encuentre una matriz X de orden 2 × 2, que resuelva la ecuación:
   
1 2 −1 0
X =
2 5 0 1
5. Sea A la matriz  
1 1
0 1
encuentre A2 , A3 y conjeture una fórmula para An . Demuéstrela por inducción.
6. Si A y B son matrices de 2 × 2, pruebe que las matrices At A y B t B son matrices simétricas.
7. Encuentre una matriz A de 2 × 2 que cumpla
 
−1 0
A2 =
0 −1

7.5. Transformaciones del Plano.


En capı́tulos anteriores estudiamos las transformaciones del plano como funciones del plano
sobre sı́ mismo. En esta sección retomaremos este análisis con un enfoque más algebraico, utilizando
para esto el lenguaje matricial y las coordenadas cartesianas, de modo que las trataremos como
funciones de R2 en R2 .

Supongamos que tenemos un sistema de coordenadas cartesianas en el plano, describiremos la


acción de la transformación sobre los puntos del plano de acuerdo a la manera en que cambian sus
coordenadas en el sistema.
 
1
Por ejemplo, para describir una traslación T en la dirección del vector , veremos cómo
2
 
x
cambian las coordenadas de un punto arbitrario bajo esta transformación. Note que al
y

220
trasladar el punto, su primera coordenada aumenta en 1, mientras que la segunda aumenta en 2.
Esto indica que la traslación cambia las coordenadas del punto de la siguiente manera:

x → x+1 , y → y+2

Esta situación puede describirse de la forma:


       
x x+1 x 1
T = = +
y y+2 y 2



− en la dirección del vector b = (a, b)t puede describirse de
De manera general, la traslación T→
b
la siguiente manera: dado un punto del plano − →x = (x, y)t la traslación actúa de modo que
       

− x x+a x a →

T→
− [ x ] = T→
− = = + =− →x + b
b b y y+b y b

En este mismo sentido, existe otro tipo de transformaciones, cuya acción sobre los puntos del
plano puede expresarse algebraicamente a través de las matrices que introdujimos en la sección
anterior.

Utilizando esta notación definimos una transformación lineal del plano como aquella cuya
acción sobre un punto del plano puede expresarse a través del producto por una matriz cuadrada
de 2 × 2, es decir: T es una transformación lineal si

T [−

x] =A−→
x
       
x a11 a12 x xa11 + ya12
T = =
y a21 a22 y xa21 + ya22

Esto indica que la transformación lineal cambia las coordenadas del punto de la siguiente
manera:
x → xa11 + ya12 , y → xa21 + ya22

221
es decir, cada una de las nuevas coordenadas se obtiene como combinación lineal de las coordenadas
originales (de ahı́ su nombre). Note, por ejemplo, que una traslación no es una transformación lineal.

En este punto y conociendo la acción de algunas de las transformaciones que estudiamos en


capı́tulos anteriores, podremos expresarlas en el lenguaje matricial.

Por ejemplo recordemos la acción de una reflexión con respecto al eje −



x . Esta transformación,
como se puede ver en la figura, cambia las coordenadas de un punto (x, y)t de la siguiente manera:

x → x , y → −y

De esta manera podemos ver que si tomamos en la matriz que determina la transformación
lineal:

a11 = 1 a12 = 0 a21 = 0 a22 = −1


podremos expresar la acción de la reflexión en la forma:

R[−

x] =A−→
x
       
x 1 0 x x
R = =
y 0 −1 y −y
Haremos lo mismo a continuación con una rotación respecto del origen. Si vemos la figura se
puede determinar, conociendo el ángulo de la rotación, cómo cambian las coordenadas de cada
punto (Ver Ejercicio 3). De esta forma sabremos cuáles son los coeficientes de la matriz que la
representa como transformación lineal.

222
Esta transformación, como se puede ver, cambia las coordenadas de un punto (x, y)t de la
siguiente manera:

x → x cos θ + y sen θ , y → −x sen θ + y cos θ

De esta manera podemos ver que si tomamos en la matriz que determina la transformación
lineal:

a11 = cos θ a12 = sen θ a21 = − sen θ a22 = cos θ


podremos expresar la acción de la rotación en la forma:

G[−→
x] =A−→x
       
x cos θ sen θ x x cos θ + y sen θ
G = =
y − sen θ cos θ y −x sen θ + y cos θ

Para el caso de una Homotecia con centro en el origen y factor k, si vemos la figura se puede
determinar fácilmente cómo cambian las coordenadas de cada punto. De esta forma sabremos
cuáles son los coeficientes de la matriz que la representa como transformación lineal.

Esta transformación, como se puede ver, cambia las coordenadas de un punto (x, y)t de la
siguiente manera:
x → kx , y → ky

223
De esta manera podemos ver que si tomamos en la matriz que determina la transformación
lineal:

a11 = k a12 = 0 a21 = 0 a22 = k


podremos expresar la acción de la homotecia en la forma:

H[−

x] =A−→
x
       
x k 0 x kx
H = =
y 0 k y ky

Si combinamos traslaciones y transformaciones lineales obtenemos las denominadas transfor-


maciones afines del plano, que actúan sobre los puntos del plano en la forma:


F [−

x ] = A−

x + b
         
x a11 a12 x a xa11 + ya12 + a
F = + =
y a21 a22 y b xa21 + ya22 + b

Ejercicios

1. Encuentre la matriz que representa como transformación lineal a la reflexión respecto del eje


y.
2. Encuentre la matriz que representa como transformación lineal a una reflexión respecto de
una recta que pasa por el origen y de ecuación general ax + by = 0.
3. Demuestre que una rotación de ángulo θ, con centro en el origen, cambia las coordenadas de
la siguiente manera:

x → x cos θ + y sen θ , y → −x sen θ + y cos θ

4. Encuentre la forma matricial de la transformación afı́n para una reflexión respecto de una
recta arbitraria de ecuación general ax + by + c = 0.

224
Bibliografı́a

[1] Charles H. Lehmann, Geometrı́a Analı́tica, Limusa, (2004).


[2] Douglas F. Riddle, Geometrı́a Analı́tica, Thomson, (1996).
[3] Fernando Arenas, Gonzalo Masjuan, Felipe Villanueva, Geometrı́a Elemental, Ediciones Uni-
versidad Católica de Chile, (1993).
[4] I, Shariguin Problemas de Geometrı́a, Editorial Mir, Moscú, (1989).
[5] W.Keith Nicholson Algebra Lineal con aplicaciones, Mc Graw Hill, (2003).
[6] Howard Eves Estudio de las Geometrı́as, UTEHA, (1963).
[7] Hall & Knight Trigonometrı́a Elemental, UTEHA, (1961).

225

Vous aimerez peut-être aussi